Sunteți pe pagina 1din 94

Anul XV, Nr.

Ianuarie Iunie 2013

RECREAII
MATEMATICE
REVIST DE MATEMATIC PENTRU ELEVI I PROFESORI

e i = 1

Asociaia Recreaii Matematice


IAI - 2013

Semnificaia formulei de pe copert:


i
ntr-o form concis, formula e = 1 leag cele patru ramuri fundamentale

ale matematicii:

ARITMETICA
GEOMETRIA
ALGEBRA
ANALIZA MATEMATIC

reprezentat
reprezentat
reprezentat
reprezentat

de
de
de
de

i
e

Redacia revistei :
Petru ASAFTEI, Temistocle BRSAN, Alexandru CRUU, Constantin CHIRIL,
Eugenia COHAL, Adrian CORDUNEANU, Mihai CRCIUN (Pacani), Paraschiva
GALIA, Paul GEORGESCU, Gheorghe ILIE, Gheorghe IUREA, Gabriel MRANU,
Gabriel POPA, Dan POPESCU (Suceava), Maria RACU, Neculai ROMAN (Mirceti), Ioan
ERDEAN (Ortie), Dan TIBA (Bucureti), Marian TETIVA (Brlad), Lucian TUESCU
(Craiova), Adrian ZANOSCHI, Titu ZVONARU (Comneti)
Materialele se trimit la una dintre adresele: t_birsan@yahoo.com, profgpopa@yahoo.co.uk
Pagina web a revistei RecreaiiMatematice: http://www.recreatiimatematice.ro
COPYRIGHT 2011, ASOCIAIA RECREAII MATEMATICE
Toate drepturile aparin Asociaiei Recreaii Matematice. Reproducerea integral sau
parial a textului sau a ilustraiilor din aceast revist este posibil numai cu acordul prealabil
scris al acesteia. Se consider c autorii materialelor trimise redaciei revistei sunt, n mod
implicit, de acord cu publicarea lor, i asum responsabilitatea coninutului lor i cedeaz
Asociaiei Recreaii Matematice dreptul de proprietate intelectual asupra acestora.
TIPRIT LA BLUE SIM&Co IAI
Bd. Carol I, nr. 3-5
Tel. 0332 111021, 0721 571705
E-mail: simonaslf@yahoo.com
ISSN 1582 - 1765

Anul XV, Nr. 1

Ianuarie Iunie 2013

RECREAII
MATEMATICE
REVIST DE MATEMATIC PENTRU ELEVI I PROFESORI

e i 1
Revist cu apariie semestrial

EDITURA RECREAII MATEMATICE

IAI - 2013

Academicianul RADU MIRON la 85 de ani


La 3 octombrie 2012, Academicianul prof.
dr. docent RADU MIRON a mplinit
85 de ani dintre care 65 i-a dedicat slujirii nv
at
am
antului si cercetarii n domeniul matematicii, ca profesor la Universitatea
,,Alexandru Ioan Cuza din Iasi, precum si
ca membru al Academiei Romane.
Evenimentul a fost marcat prin organizarea n cadrul Zilelor Universit
atii ,,Alexandru Ioan Cuza (Dies Academici) din luna
octombrie 2012 a unui simpozion aniversar
intitulat ,,Academicianul Radu Miron la 85
de ani la care au participat rectorul Universitatii, presedintele Filialei Iasi a Academiei
Romane, membrii ai Academiei Romane si
ai Academiei de Stiinte a Moldovei, profesori
universitari din Iasi si din alte centre universitare, profesori de la licee din Iasi. Academicianului Radu Miron i s-a nmanat
Diploma de excelent
a a Universit
atii ,,Alexandru Ioan Cuza din Iasi pentru ntreaga
activitate didactica si stiintic
a desfasurata la Universitatea ,,Alexandru Ioan Cuza
din Iasi, pentru introducerea conceptelor de spatii Lagrange si Hamilton si studiul lor
si crearea Scolii romanesti de cercetare a acestor spatii.
Simpozionul a fost un prilej excelent de evocare a realizarilor Acad. Radu Miron
n domeniul stiintei si educatiei.
S-a nascut la 3 octombrie 1927 n comuna Codaesti, judetul Vaslui, ntr-o familie
numeroas
a din clasa de mijloc a vremii. De mic a dorit sa e inginer, ca unul dintre fratii sai. Dupa clasele primare n Codaesti, urmeaza un liceu tehnic n Barlad.
La absolvire s-a hotar
at sa studieze un an la Facultatea de matematica a Universitatii pentru a-si completa cunostintele de matematica si apoi sa treaca la Facultatea
de constructii de la Institutul Politehnic din Iasi. Aici, sub inuenta profesorilor Al.
Myller si Octav Mayer, personalitati ilustre ale matematicii romanesti, si-a descoperit
talentul pentru matematica si a ramas sa studieze matematica. In anul al treilea de
studii a fost numit preparator, iar la sfarsitul studiilor, n 1952, a fost ncadrat ca
asistent la Facultatea de matematica a Universitatii iesene si cercetator la Institutul
de Matematica din Iasi al Academiei Romane. Cariera domniei sale va inseparabil
legata de aceste doua institutii. Obtine titlul de doctor n matematici n 1956 cu o
teza referitoare la aplicatii ale geometriei n Mecanica. Teza de doctorat de o valoare
deosebita i-a deschis drumul spre pozitii superioare n cercetare si nvatamant, astfel
ca n 1969 este numit profesor titular la Facultatea de Matematica si Sef de sector
la Institutul de Matematica. Obtine titlul de doctor docent n 1972. Conduce, ca
decan, Facultatea de Matematica ntre anii 1972 si 1976. Intre timp participa si tine
comunic
ari stiintice foarte apreciate la numeroase manifestari stiintice nationale si
1

internationale. Contactele cu Scoala japoneza de geometrie Finsler au fost hotaratoare


n decizia acad. Radu Miron de a studia n profunzime geometria spatiilor Finsler.
Studiul acestor spatii l-a condus la introducerea spatiilor Lagrange n anii 1984-1985 si
a spatiilor Hamilton, n anii 1987-1988. Lucrarile, memoriile si monograile publicate
asupra acestor spatii n reviste si edituri internationale i-au adus consacrarea deplina
pe plan national si international precum si recunoasterea nationala cea mai important
a, alegerea ca membru corespondent n 1991 si ca membru titular al Academiei
Romane n 1993. S-a pensionat si a devenit profesor consultant n 1998.
Acad. Radu Miron s-a armat pentru prima oara n mediul stiintic prin teza de
doctorat n care rezolva o problema considerata insolubila de catre celebrul geometru
francez Elie Cartan. Dupa publicarea tezei de doctorat continua studiile de geometrii
cu grup fundamental ale predecesorilor sai Al. Myller si O. Mayer. Aprofundeaza o
conguratie geometrica numita de d-sa configuratie Myller, devenita clasica, asupra
careia publica o monograe de referinta, premiata de Academia Romana n 1968.
In 1974 comunic
a prof. Makoto Matsumoto, conducatorul scolii japoneze de geometrie Finsler, constructia unei clase de repere ortonormate pe o varietate Finsler si
ecuatiile lor de miscare, denumite de savantul japonez Repere Miron.
In 1980 organizeaza primul Seminar National de Geometrie Finsler la Universitatea din Brasov, cu scopul de a mobiliza geometrii romani la studiul acestei geometrii.
In acest Seminar, aat la a 16-a editie, acad. Radu Miron a introdus si a propus ca
obiect de studiu spatiile Lagrange, spatiile Hamilton de ordin 1 si de ordin superior.
Ideile si rezultatele sale au fost preluate si dezvoltate de participantii din toate centrele
universitare, constituindu-se un grup de cercetare cunoscut azi ca S
coala rom
aneasc
a
de geometrie Finsler si Lagrange.
Colaborarea acad. Radu Miron cu geometrii japonezi s-a extins mai ales dupa
Colocviul rom
ano-japonez de geometrie Finsler din august 1984 la care au participat
14 matematicieni japonezi. In aceasta colaborare, care dainuie si astazi, 10 geometri
japonezi si 3 din Ungaria, Italia si Vietnam au obtinut titlul de doctor n matematica
sub conducerea acad. Radu Miron.
In cadrul Seminarului de geometrie Finsler si Lagrange s-a elaborat, sub conducerea acad. Radu Miron, prima monograe dedicata bratelor vectoriale, spatiilor
Lagrange si aplicatiilor n Relativitate; publicata de Editura Academiei, premiata de
Academia Roman
a n 1987, a carei versiune englezeasca a aparut apoi n 1997.
Dup
a 1990, studiile de geometrie Finsler si Lagrange sunt orientate de catre acad.
Radu Miron cu precadere spre aplicatii n Mecanica si Fizica Teoretica, ndeosebi n
Teoria Relativitatii. Aceasta linie este ilustrata de monograa n limba engleza elaborata n colaborare cu autorul acestor randuri, publicata de Editura Kluwer Academic
Publishers n 1994. Ecoul acestei monograi n lumea specialistilor a impus geometria
spatiilor Lagrange n randul matematicienilor, zicienilor si mecanicienilor.
Eforturile de cercetare si de organizare a scolii de geometrie Finsler si Lagrange
ale acad. Radu Miron au fost conjugate cu preocuparea de a face cunoscute rezultatele obtinute. A publicat cca. 250 de articole stiintice pe langa 28 de carti si de
monograi. A participat si a prezentat comunicari si conferinte la zeci de manifestari
stiintice din tar
a si de peste hotare. Plin de energie, a calatorit mult si a conferentiat
la numeroase universit
ati de seama din Europa, Japonia, America de Nord, Egipt,
2

India. A stabilit relatii de colaborare cu matematicieni de pretutindeni. A sprijinit


tinerii pentru a face studii de doctorat. Peste 30 de geometri romani si straini au
obtinut titlul de doctor n matematici sub conducerea d-sale si sunt astazi profesori
universitari sau cercetatori cunoscuti.
Activitatea stiintic
a bogata a acad. Radu Miron a fost recunoscuta pe plan
national si international n diferite forme de catre colegii sai prin folosirea si citarea
lucrarilor d-sale. Rezultate ale d-lui acad. Radu Miron sunt incluse n monograi de
specialitate cu notiuni si metode care-i poarta numele si care au fost preluate de catre
cercetatorii n domeniu. Includerea unor idei, concepte si rezultate ale d-sale ntr-un
tratat adresat studentilor, elaborat de trei importanti matematicieni americani arata
caracterul esential al acestora si perenitatea lor. Mentionam recunoasteri ociale prin:
Premiul Ministerului
Inv
at
am
antului din 1956, Premiul ,,Gh. T
iteica al Academiei
Rom
ane si recunoasterea suprema, alegerea d-sale ca membru al Academiei Rom
ane.
Adaugam ca universit
atile din Constanta, Craiova, Bacau, Galati, Oradea, Tiraspol
i-au acordat titlul de Doctor Honoris Causa. A primit diplome de excelent
a cu diferite
ocazii de la Ministerul Educatiei si Cercetarii, Universitatea ,,Alexandru Ioan Cuza
unde este si profesor emeritus, de la Universitatea ,,Petre Andrei. In 2003 a primit
premiul Opera Omnia al CNCSIS din Romania pentru ntreaga activitate de cercetare.
In acelasi an a primit Premiul ,,V. Pogor al Primariei Municipiului Iasi.
Acad. Radu Miron a mp
art
asit cu generozitate experienta si dragostea sa pentru matematica la peste 50 de serii de studenti de la Facultatea de Matematica a
Universit
atii din Iasi. Intr-o jumatate de veac de activitate didactica a predat cursuri de geometrie superioara la toate nivelurile. Totdeauna adaptate la cunostintele
studentilor, prezentate viu, atractiv, cu eleganta, lectiile d-sale au fost extrem de apreciate de catre studentii sai si pentru multi dintre ei profesorul Radu Miron a devenit
un model pe care l-au urmat n cariera lor de profesori sau cercetatori. Claritatea si
uenta ideilor, date de propozitii scurte, percutante si notatii matematice adecvate,
caracterizeaz
a si cele peste 20 de manuale si carti adresate studentilor, profesorilor
de liceu si elevilor cu nclinatie pentru geometrie, publicate de acad. Radu Miron la
edituri romanesti. S-a implicat n toate activitatile cerute de munca cu studentii si s-a
preocupat constant de evolutia ulterioara a absolventilor Facultatii de matematica.
Stimat de fostii sai studenti, a fost invitat sa tina conferinte pentru profesori si elevi n
mai toate liceele importante din Moldova. A condus multi ani Filiala Iasi a Societatii
de Stiinte Matematice. A sprijinit si sprijina Asociatia ,,Recreatii Matematice atat
ca membru de onoare al ei, cat si prin articole publicate n Recreatii Matematice,
revista de matematica elementar
a editata de aceasta asociatie.
La cei 85 de ani, purtati cu optimism, acad. Radu Miron este o personalitate
proeminent
a a nv
at
am
antului si cercetarii romanesti, autor al unor concepte si rezultate intrate n patrimoniul matematicii mondiale, creator de scoala stiintica recunoscuta pe plan international, profesor model pentru numeroase generatii de studenti.
La aceasta aniversare, colaboratorii si elevii Domniei Sale sunt onorati a-i aduce
un profund omagiu de recunostint
a pentru opera stiintica, pentru generozitate si
ncredere si a-i ura multi ani de viat
a, cu sanatate, cu putere de munca si creatie.

Prof. univ. dr. Mihai ANASTASIEI


3

Profesorul TRAIAN COHAL


la aniversarea a 80 de ani
Personalitate marcant
a a nvatamantului preuniversitar iesean, Profesorul Traian Cohal a slujit
timp de patru decenii scoala romaneasca. A contribuit
n mod decisiv la educarea si formarea caracterelor
multor generatii de elevi carora le-a fost Dascal. Cu
talent didactic si munc
a neobosita a ridicat multe serii
de elevi la un nivel superior de ntelegere a matematicii, de competitivitate si de performanta. Sub
ndrumarea sa, un num
ar mare de elevi au obtinut
rezultate remarcabile la olimpiade si concursuri de
matematica nationale sau internationale.
Traian Cohal s-a nascut la 12 august 1932 n
orasul Dorohoi, a absolvit Colegiul National ,,Grigore
Ghica din localitate n 1951 si apoi cursurile Facult
atii de Matematic
a si Fizic
a la Universitatea din Iasi
n 1955. A fost retinut ca asistent la aceeasi facultate, unde a functionat n perioada
1956-1958. Fiind privit ca o persoana cu ,,origine sociala nesanatoasa, a fost nlaturat
din nv
at
am
antul superior. Dupa o pauza de doi ani de inactivitate, este ncadrat ca
profesor la Liceul ,,Alexandru Ioan Cuza din Iasi n 1960. Incepand din anul 1966
si pan
a la data pensionarii, n anul 1997, a fost profesor de matematica la Colegiul
National ,,Costache Negruzzi. In slujba acestei ultime institutii Profesorul Traian
Cohal a pus ntrega sa capacitate de munca si talentul de dascal, de aceasta scoala
si-a legat rostul vietii. Intr-un interviu arma: ,,Tat
al meu spunea c
a nt
ai este tara,
apoi familia si apoi restul. C
and fac inventarul vietii mele, nt
ai este tara, iar apoi
Negruzzi.
Ca unul dintre fostii sai elevi, mi aduc aminte de atentia deosebita pe care Profesorul Cohal o acorda corectitudinii si claritatii lectiilor, pana la cele mai mici
detalii. Conducea lectiile n mod magistral, elevii l ascultau cu rasuarea taiata.
O tras
atur
a denitorie era dorinta sa de a atinge perfectiunea n tot ceea ce facea.
Aceasta exigent
a s-a transmis si elevilor sai, care au primit astfel un atu hotarator
pentru reusita n viat
a. Era un dascal cu o personalitate complexa. Nu e de mirare ca
fostii sai elevi l descriu n mod diferit, ecare observand o alta latura a personalitatii
sale coplesitoare. Desigur, la aceasta a contribuit si grija dascalului pentru ecare
elev n parte, ambition
andu-l si oferindu-i raspunsurile de care avea nevoie.
Acorda o atentie deosebita si celorlalte aspecte ale vietii tinerilor. Acestora, cauta
sa le induca o dezvoltare sanatoasa si echilibrata. Nu rareori citea si cate o poezie n
orele de matematica. In momente libere, organiza excursii cu ntreaga clasa, plimbari
n Gradina Botanica, vizionarea de lme artistice etc. Cu timpul, catre ultimii ani de
liceu, raporturile sale cu tinerii elevi se transformau n adevarate legaturi de prietenie.
De-a lungul ntregii sale activitati, s-a aplecat cu o atentie deosebita si cu grija
parinteasc
a asupra elevilor nevoiasi, care aveau o situatie materiala precara. Pentru
4

multi dintre acesti elevi capabili ce proveneau din mediul rural a reprezentat factorul
decisiv al reusitei lor n viat
a. Desigur, ei si amintesc cum Profesorul i-a ajutat cu
pachete consistente de alimente n cursul acelor ani marcati de lipsuri din perioada
comunist
a sau a intervenit la conducerea liceului pentru a li se asigura un loc n
caminul internat. Unii dintre acesti elevi sunt astazi cercetatori recunoscuti pe plan
international sau profesori la universitati din tara sau strainatate.
Profesorul T. Cohal are o bogata activitate editoriala, ndreptata spre aceeasi
nalitate: sustinerea nv
at
am
antului matematic romanesc. Intre anii 1990 si 2007
a publicat 12 carti reprezent
and culegeri de probleme ce acopera programele pentru
gimnaziu si liceu, carti foarte apreciate pentru continutul variat si bogat, ordinea si
gradarea problemelor, frumusetea problemelor incluse, precum si pentru acuratetea
si eleganta stilului n care sunt redactate.
Se spune ca bucuria cea mai mare a unui dascal este aceea de a urmari reusita
profesional
a a elevilor sai. Din acest punct de vedere profesorul Cohal poate unul
dintre cei mai fericiti si mpliniti dascali. Dintre elevii sai care se disting n domeniul
matematicii vom aminti (cerem scuze celor care nu sunt mentionati aici, desi ar
meritat) pe Horia Neculai Teodorescu, profesor la Univ. Tehnica din Iasi si membru
corespondent al Academiei Romane, Octavian P
astr
avanu, profesor la Univ. Tehnica
din Iasi si membru al Academiei de Stiinte Tehnice, Dan Tiba, cercetator principal la
I.M.A.R (Bucuresti) si pe profesorii universitari Gheorghe Anicul
aesei (Iasi), Ovidiu
C
arj
a (Iasi), Constantin Z
alinescu (Iasi), Gabriel Turinici (Paris).
Profesorul T. Cohal a participat si la treburile comunitatii matematice iesene;
n perioada 1959-1964 a fost secretar al Societatii de Stiinte Matematice, liala Iasi.
Cea mai buna dovad
a a faptului ca domnul profesor a reusit sa insue elevilor sai
nu doar cunostinte matematice ci si o atitudine de responsabilitate si generozitate
este instituirea, ncep
and cu anul 2003, a burselor ,,Traian Cohal la Colegiul ,,C.
Negruzzi. Un grup de fosti elevi: Ioan Macinca, Ioan Sauciuc, Mariana Olaru,
Ovidiu T
an
asuc
a, George Rotaru, Manuel Savin si Romi Ciubotariu, ce lucreaza acum
n strain
atate, s-a ntrunit si a decis sa nanteze anual ntretinerea a 4-8 elevi silitori
ce provin din mediul rural. Bursierii primesc bani de haine, de cazare, carti, alimente
si tot ce le este necesar. ,,Bursa a ap
arut ca un omagiu adus unui om care ne-a ajutat
s
a ne croim un drum prin viat
a. Ne-am angajat s
a sustinem aceast
a burs
a p
an
a la
sf
arsitul vietii noastre. E o progresie geometric
a, cei pe care i ajut
am noi i vor ajuta
pe altii si tot asa..., explica fostul elev Ioan Sauciuc care lucreaza la Intel, SUA.
Pentru realizarile deosebite n activitatea sa, Statul roman i-a acordat mai multe
distinctii: Medalia Muncii (n 1965 si 1983), Ordinul Muncii clasa a III-a (1972) si
Ordinul National ,,Serviciu credincios n gradul de Cavaler (2000).
Toti cei care l-au cunoscut pe Profesorul Traian Cohal l pretuiesc ca pe un
trunchi viguros cu seva san
atoas
a care a hranit si a crescut multe ramuri tinere pentru
care matematica a devenit un viitor si un destin.
La acest moment aniversar, fostii sai elevi, animati de admiratie si afectiune,
adreseaz
a Profesorului Traian Cohal un simplu si calduros La multi ani!
Ad multos annos!

Conf. dr. Mircea BIRSAN


5

Matematica
ntre mituri culturale si certitudine
De fapt, ce este matematica? Intrebarea ne descumpaneste deoarece, aparent, toti
stim raspunsul; gandul zboara imediat la orele de matematica din scoala, la temele
de acasa cu fractii, ecuatii, triunghiuri si mai tarziu cu derivate si integrale. Desigur,
toate acestea sunt matematica, dar o denitie comprehensiva ne vine mai greu n
minte. De fapt, orice denitie am ncerca (stiinta calculului, tehnica manipularii
logice a simbolurilor, teorie a structurilor organizate axiomatic etc.), vom constata ca
este e incompleta, e prea vaga pentru a descrie exact aceasta stiinta.
Matematica este o stiinta care se bazeaza esential pe logica clasic
a, dar nu este
reductibila totusi la logica, asa cum credeau logicienii de la nceputul secolului al
XX-lea. Daca ar asa, ea s-ar reduce, cum observa Henri Poincar
e, la o uriasa
tautologie de tipul a = a. Ea contine intrinsec n mecanismul sau intim o trasatura
care face ca un adevar banal de tipul a = b si b = c implica a = c sa conduca prin
rationament logic la adevaruri de mare profunzime, altfel de nepatruns chiar si pentru
o inteligent
a sclipitoare. Dar vom reveni mai ncolo asupra acestui fapt. Matematica,
sau, mai precis, rationamentul matematic (am inclus aici si calculul matematic) are
aceasta virtute surprinzatoare de a nobila si mbogati adevarul primar, adaugandu-i
pe parcurs noi informatii si dimensiuni. Aceasta este, de fapt, arta matematicianului
profesionist, aseman
atoare ntr-un fel cu cea a artistului care modeleaza din materia
primara si amorfa forme cu valoare estetica. Am evidentiat doar un aspect al stiintei
matematice ca instrument de cunoastere, dau suntem departe de a contura o denitie
a ei.
Matematica intervine n procesul de cunoastere si descoperire prin intermediul
unui model care reproduce n termeni simbolici mai mult sau mai putin del un
anume fenomen sau proces zic. In felul acesta, miscarea mecanica se descrie prin intermediul legilor newtoniene ca o ecuatie diferentiala de ordinul doi (de fapt o relatie
ntre acceleratie, viteza si pozitie), dinamica particulelor subatomice se reduce la o
ecuatie cu derivate partiale (ecuatia Schrodinger), campul electromagnetic la ecuatile
lui Maxwell, iar dinamica uidelor la ecuatiile Navier-Stokes. Aceste modele sunt
foarte exacte n cazul proceselor zice si tehnice, dar aproximative n cazul stiintelor
sociale si chiar n unele stiinte ale naturii cum ar biologia. Opera literara scapa
oricum oricarei ncerc
ari de a o explica serios folosind modele matematice si nici nui rau ca se nt
ampl
a asa. Ar , desigur, tragic daca am putea cuantica maestria
artistica si am putea pune n ecuatie opera literara, deoarece aceasta ar nsemna ca
misterul si orul creatiei artistice este doar o combinatie de cifre. Sub inuenta structuralismului s-au facut numeroase ncercari n acest sens, dar rezultatele au fost pur
speculative. Inadecvarea matematicii n descrierea fenomenelor sociale, spirituale si
artistice provine din nsusi specicul limbajului sau care exclude exprimarile ambigue
si conceptele imprecise, desi structurile sale moderne pot descrie destul de nuantat si
fenomene sociale sau din sfera gandirii.
Totusi, sa nu subestimam matematica ca disciplina fundamentala n istoria culturii. Marii loso greci au ncoronat-o ca pe o adevarata regina a stiintelor si aceasta
6

a ramas peste secole ca un model al perfectiunii n gandirea umana. Galileo Galilei


a armat chiar ca natura este scrisa n limbaj matematic. Sa mai presupunem ca
matematica a fost o creatie a geniului Greciei antice. Ea nu s-a nascut n Babilonia, India sau n Egiptul antic (acolo s-au inventat doar cifrele, calculul si tehnica
masuratorilor geometrice), ci n centrele culturale ale lumii grecesti, odata cu fundamentarea rationamentului deductiv si arta demonstratiei geometrice. Numai un popor
nzestrat cu imaginatie si nclinat spre meditatie metazica putea sa-si dea seama
ca cele mai multe adevaruri cantitative referitoare la marimi geometrice (unghiuri,
distante, pozitie) sau numere, pot deduse logic (o alta descoperire greceasca) dintrun set minimal de adevaruri elementare pe care le-am numit axiome. Atunci s-a
nascut matematica, candva, cam cu cinci secole nainte de Hirstos.
Odata cu matematica s-a nascut si spiritul stiintific, adica interpretarea critica
a observatiilor empirice si a miturilor si ncadrarea lor ntr-un sistem ordonat care
evidentia ideea de repetabilitate si, n ultima instanta, de legitate.
Mai mult decat oricare alt domeniu al cunoasterii, matematica a intrat n contiinta
cultural
a cu aura de infailibilitate pe care i-o confera aparenta rigoare si precizie a
argumentelor si rezultatelor sale. Mai ntotdeauna cand matematica intra n discutie i
se asociaza atribute precum: exacta, precisa (se spune chiar ca ar ,,partea exacta a
stiintei) si este unanim vazut
a ca stiinta logico-deductiva producatoare de adevaruri
irefutabile si complete.
Nu este intentia noastra aici de a stirbi miturile acestei stiinte care este, fara
ndoiala, una dintre cele mai importante creatii ale geniului uman, dar am dori sa
nuantam totusi unele dintre ideile cele mai vehiculate si, din pacate, nu tocmai exacte
despre matematica.
Sa ncepem cu sintagma atat de frecvent utilizata: Matematica stiint
a logicodeductiv
a. Desigur, cu totii stim ca rationamentul matematic se bazeaza pe principiile
si mecanismele logicii formale si este, probabil, cea mai remarcabila ilustrare a acesteia. Gandirea si exprimarea logica sunt parte a oricarui discurs articulat, dar numai
n matematica acestea sunt obligatii absolute. Matematica este un sistem formalizat
construit pe un num
ar restrans (si cum vom vedea minimal) de adevaruri elementare
(axiome) din care se obtin, prin principiile logicii formale, adevaruri din ce n ce mai
complexe. Marele mister si miracol al matematicii este capacitatea sa de a construi
din adevaruri triviale (unele simple tautologii) adevaruri de mare profunzime. Asa
cum am subliniat mai sus, procesul de descoperire matematica nu este pur si simplu o
operatie logica, adica o simpla nsiruire de silogisme, ci un act complex n care logica
joaca doar rolul de instrument. Logicismul fundamentat si promovat de Bertrand
Russel la nceputul secolului pretindea n esenta ca matematica este o manipulare
formala de simboluri, iar programul n care a crezut pana la sfar;situl vietii marele
matematician german David Hilbert si propunea fundamentarea matematicii pe
baza unui sistem axiomatic complet si necontradictoriu. A fost, desigur, o tentativa
utopica n care matematicienii de bun simt, am citat anterior pe Henri Poincar
e, nu
au crezut chiar de la bun nceput si care va inrmata ulterior n urma demonstrarii
de catre K. G
odel a principiului incompletitudinii: matematica nu este complet formalizabil
a. Acest rezultat aparent suprinzator avea, desigur, sa deziluzioneze pe cei
care au gandit matematica ca un ediciu perfect sustinut prin soliditatea si infaili7

bilitatea logicii, dar de fapt adevarata forta si ecacitatea matematicii provin tocmai
din aceea ca rationamentul matematic este neformalizabil si impredictibil. Este si
ceea ce a condus pe zicianul E. Wigner la sintagma celebra ,,irezonabila ecienta
a matematicii.
In felul acesta se naruie si mitul exprimat de Matematic
a stiint
a exact
a. Cele
mai multe dintre adevarurile matematice sunt desigur exacte si neechivoce pentru
standardele stiintei si tehnologiei actuale, dar gradul lor de precizie este totusi relativ.
Nefunction
and ca sistem complet formalizat, matematica nu poate da un raspuns (de
tip da sau nu) pentru orice propozitie, fara riscul de a produce contradictii logice. In
ne, teoriile matematice dau solutii doar n cadrul unor anumite criterii de exigenta si
rigoare. De exemplu, miscarea browniana se poate descrie matematic doar n termeni
probabilistici si este de nedenit n cadrul matematicii clasice. In aceeasi categorie
intr
a procesele dinamice care manifesta haos si care nu admit nici n prezent o descriere matematica exacta. Desigur, n timp matematica poate dezvolta teorii care sa
nglobeze aceste manisfestari rebele prezente n sistemul naturii sau n propiul sau
corp, dar existenta lor ilustreaza tocmai faptul subliniat anterior: ca stiinta, matematica nu este nici exacta si nici fara repros ca producatoare de adevaruri infailibile;
este ns
a un instrument ecient de cunoastere care datoreaza mult altor discipline si
este deschis altor tipuri de rationament si de practici stiintice. Aceasta nu nseamna
ca matematica, ca stiint
a, produce rezultate discutabile si nici ca discursul sau este
doar o variant
a mai pretentioasa a celui losoc sau artistic. Ceea ce le desosebeste
fundamental ns
a este caracterul univoc al tezelor si rigoarea conceptelor matematice.
M. Heidegger observa undeva ca rigoarea limbajului losoc obliga n discursul lor losoi sa ajunga la concluzii asemanatoare. Aceeasi observatie se aplica
si matematicienilor si din acelasi motiv deriva unitatea matematicii si caracterul
neechivoc al rezultatelor sale.

Acad. Viorel BARBU

Cercuri, valori medii si diviziuni armonice


Temistocle B
IRSAN

Abstract. Considering an angle and a circle tangent to its sides as a starting point, other six
circles are considered and the properties of the resulting configuration are studied.
Keywords: circle, homothecy, inversion.
MSC 2010: 51M04.

1. In aceasta Nota ne propunem sa punem n evidenta structura armonica a unei


anumite conguratii geometrice prin mijloace simple. La baza conguratiei stau un
unghi arbitrar cu varful notat A si un cerc C(I, r) tangent la laturile unghiului.
Se considera succesiv urmatoarele cercuri:
C(I1 , r1 ) si C(I6 , r6 ) tangente la laturile unghiului dat, si la cercul C(I, r), iar
pozitia centrelor pe bisectoarea AI a unghiului sa respecte ordinea AI1 I I6 ;
C(I2 , r2 ) si C(I5 , r5 ) tangente la laturile unghiului, cu centrele pe cercul C(I, r)
si respectand ordinea A I2 I I5 ;
C(I3 , r3 ) si C(I4 , r4 ) tangente la laturile unghiului, trecand prin centrul I al
cercului dat si cu ordinea A I3 I I4 pentru centrele lor.
Conguratia obtinuta prin aceasta constructie ,,ascunde anumite legaturi ntre
partile ce o compun, legaturi care surprind armonia ei si care vor dezvaluite n
limbaj matematic mai jos.
2. Cercurile din conguratia de mai sus vor numite, pe scurt, C, Ck , k = 1, 6.
Vom determina nt
ai razele cercurilor Ci n functie de elementele de baza.
Propozitia 1. Razele cercurilor Ci sunt date n functie de r si A (=m
asura
unghiului dat) de formulele:
1 sin A2
,
1 + sin A2

A
,
r2 = r 1 sin
2
r
r3 =
,
1 + sin A2

r1 = r
(1)

r4 =

r
,
1 sin A2

r5 = r 1 + sin
r6 = r

A
,
2

1 + sin A2
.
1 sin A2

Demonstratie. Aceste formule se stabilesc n acelasi mod.


Pentru prima dintre ele, conditia de tangenta a cercurilor C1 si C revine la I1 I =
r1 + r. Pornind de la relatia evident
a
AI = AI1 + I1 I,
1 Prof.

dr., Univ. Tehnic


a ,,Gh. Asachi, Iasi

obtinem succesiv:
r
r1
=
+ r1 + r,
A
sin 2
sin A2
A
r = r1 + (r1 + r) sin ,

A
A
r1 1 + sin
= r 1 sin
2
2
si, de aici, prima formul
a din (1). Similar se obtine a sasea formula din (1).
Pentru a doua formul
a (si similar pentru a cincea) se pune conditia I2 I = r si se
pleaca cu egalitatea AI = AI2 + I2 I.
Pentru a treia, conditia este I3 I = r, iar egalitatea de pornire este AI = AI3 + I3 I
(similar pentru egalitatea a patra).
Observatie. Prin consideratii geometrice sau prin utilizarea formulelor (1), suntem condusi la urmatoarea ordonare a razelor:
(2)

r 1 < r 2 < r 3 < r < r5 < r 4 < r 6 .

Mention
am ,,anomalia r5 < r4 , fapt intuitiv.
O masur
a a armoniei interne a conguratiei este data de rezultatul urmator. Stabilirea acestuia se face imediat, prin calcul, pe baza egalitatilor (1).

I5

.
I6
.

..
.J . I
.
.
. I2 I3
I1
A
Propozitia 2. Au loc relatiile:
(3)

2r = r2 + r5 ,

(4)

r2 = r1 r6 = r2 r4 = r3 r5 ,
2
1
1
=
+ .
r
r3
r4

(5)

10

I4

Observatii. 1) Relatia (3) spune ca raza cercului C este media aritmetica a


razelor cercurilor C2 si C5 ce au centrele pe C (amintim, toate cercurile ce intervin
sunt tangente la laturile unghiului). Altfel spus, cercurile C2 si C5 sunt tangente.
2) Relatiile (4) arata ca raza cercului C este media geometrica a razelor situate pe
pozitii egal departate de extreme n sirul (2).
3) In sfarsit, relatia (5) arma ca raza cercului C este media armonica a razelor
cercurilor C3 si C4 ce trec prin centrul sau.
Este util sa mp
artim sirul (2) n ,,jumatatile:
(6)

r1 < r2 < r3 < r si r < r5 < r4 < r6 .

Proprietati de tipul celor din Propozitia 2, dar antrenand separat aceste ,,jumatati
sunt enuntate n propozitia urmatoare. Lasam cititorului demonstrarea (simpla!) si
interpretarea noilor relatii:
Propozitia 3. Au loc relatiile:
(7)

rr1 = r2 r3 , rr6 = r4 r5 ;
3
r1 + r2 + r3
1
r2 =
1
3
+
r4
r5 +
r1 + r3
2
r2 =
1
1 .
2
+
r5
r6

(8)
(9)

1
r6

3. Un studiu asupra pozitiilor relative ale punctelor I si Ik , k = 1, 6, pe bisectoare


va oferi noi informatii despre conguratia de cercuri.
Din faptul ca
(10)

AI =

r
rk
, AIk =
(k = 1, 6)
A
sin 2
sin A2

si tinand seama de formulele (1), deducem:


1 sin A2
r

,
sin A2 1 + sin A2
r
A
AI2 =
1 sin
,
2
sin A2
r
1
AI3 =

,
A
sin 2
1 + sin A2

r
1

,
sin A2 1 sin A2
r
A
AI5 =
1 + sin
,
2
sin A2
1 + sin A2
r
AI6 =

.
sin A2
1 sin A2

AI1 =

(11)

AI4 =

Sa facem observatia simpla ca au loc apartenentele: C I2 , I5 ; C1 I2 ; C3 I, I1 ;


C4 I, I6 ; C6 I5 ; unele deriva direct din denitia cercurilor C si Ck , iar altele
utilizand (11) si (1) (de exemplu, pentru I1 C3 se scrie I1 I3 = AI3 AI1 si cu (11)
1sin

se obtine I1 I3 = r 1+sin A2 , iar cu (1) avem I1 I3 = r1 ).


2

Fie J punctul de tangent


a a cercurilor C2 si C5 , adica C2 C5 = {J}.
11

Propozitia 4. Urm
atoarele perechi de puncte sunt conjugate armonic:
(i) (A, I2 ) si (I1 , I);
(ii) (A, J) si (I2 , I5 );
(iii) (A, I) si (I3 , I4 );
(iv) (A, I5 ) si (I, I6 ).
Demonstratie. Armatiile se demonstreaza la fel, pe baza formulelor (11). Vom
AI1
I2 I1
dovedi numai pe prima. Asadar, sa aratam ca
=
. Intr-adevar, avem:
AI
I2 I
1 sin A2
AI1
=
,
AI
1 + sin A2
I2 I1
AI2 AI1
=
=
I2 I
AI AI2

A
1 sin
2

1 sin A2

1 + sin A2

: 1 1 sin

A
2

1 sin A2
,
1 + sin A2

din care rezulta egalitatea de rapoarte dorita.


De fapt, se constata usor ca toate rapoartele ce intervin n armatiile (i)-(iv) au
valoarea =

1sin
1+sin

A
2
A
2

. Sa retinem numai pe acelea care contin varful A; avem:


AI2
AI3
AI
AI1
=
=
=
= .
AI
AI5
AI4
AI6

(12)

Fin (12) rezulta usor urmatorul rezultat:


Propozitia 5. Omotetia H de centru A si raport transform
a cercurile C, Ck
dup
a cum urmeaz
a: C1 C, C2 C5 , C3 C4 si C C6 . Cercurile surs
a si imagine
prin H sunt tangente exterior: C1 C = {I2 }, C2 C5 = {J}, C3 C4 = {I} si
C C6 = {I5 }.
Observatie. A este punctul de omotetie directa pentru toate cuplurile de cercuri
mentionate, iar centrele lor de omotetie inversa sunt tocmai aceste puncte de tangenta.
Armatiile din Propozitia 4 decurg si din rezultatul urmator: centrele de omotetie
sunt conjugate armonic cu centrele cercurilor.
In conformitate cu (10), mpartind relatiile din (4) cu factorul sin2
(13)

A
2,

vom obtine

AI 2 = AI1 AI6 = AI2 AI4 = AI3 AI5 ,

care permite sa armam:


Propozitia 6. Inversiunea de pol A si putere AI transform
a centrele cercurilor
Ck dup
a cum urmeaz
a: I1 I6 , I2 I4 , si I3 I5 , iar centrul I este punct fix.
Observatie. In nal, constatam ca proprietatile conguratiei considerate sunt
rezultatul interactiunii celor doua transformari introduse mai sus omotetia si inversiunea , ce apar n mod resc.
Bibliograe
1. T. Lalescu Geometria triunghiului, Editura Tineretului, Bucuresti, 1958.
12

O proprietate a patrulaterelor convexe


Cornelia-Livia BEJAN1
Abstract. The main aim of this Note consists in showing that equation (1) holds.
Keywords: area, convex quadrangle, cyclic quadrangle.
MSC 2010: 51M04, 51M25.

Fie ABCD un patrulater convex si P un punct n interiorul sau. Sa observam ca


dupa pozitia punctului P fat
a de diagonala BD, avem: i) daca P si A sunt de aceeasi
parte fat
a de BD, atunci patrulaterul P DAB este concav iar P BCD este convex; ii)
daca P si C sunt de aceeasi parte fata de BD, atunci P DAB este patrulater convex,
iar BP CD este concav; iii) daca P (BD), atunci aceste patrulatere degenereaza n
triunghiurile DAB si, respectiv, BCD.
Sa notam aria unui patrulater U V XY cu SU V XY . Un rezultat cunoscut, util n
ceea ce urmeaza, este dat de urmatoarea
Lem
a. Aria patrulaterului U V XY , convex sau concav, este dat
a de formula
SU V XY =

1
U X V Y sin ,
2

unde = m(U
X, V Y ) (i.e., jum
atate din produsul dintre lungimile diagonalelor si
sinusul unghiului dintre ele).
Rezultatul principal al acestei note este enuntat n
Propozitia 1. Fie ABCD un patrulater convex si punctele P (AC) si Q
(BD). Atunci, are loc relatia:
(1)

P A2 SP BCD + P C 2 SP DAB QB 2 SQCDA QD2 SQABC =


= S (P A P C QB QD),

unde S noteaz
a aria patrulaterului ABCD.

Demonstratie. Notand I punctul de intersectie a diagonalelor si = m(AIB)


si tinand seama de Lema, relatia (1) se scrie:
D
(2) P A2 P C BD + P C 2 P A BD QB 2 QD AC
QD2 QB AC = AC BD(P A P C QB QD).

I
PA
QB
si s =
rapoartele n care P si Q
PC
QD
P
mpart diagonalele (AC), respectiv (BD). Rezulta ca
B
A
k
1
s
1
PA =
AC, P C =
AC, QB =
BD si QD =
BD si prin nlocuire
k+1
k+1
s+1
s+1
n (2) se ajunge la o egalitate evidenta. Asadar, relatia (1) este stabilita.
Fie k =

1 Prof.

dr., Departamentul de Matematic


a si Informatic
a, Univ. Tehnic
a ,,Gh. Asachi, Iasi

13

Corolarul 1. Fie I punctul de intersectie a diagonalelor patrulaterului convex


I1 I2 I3 I4 . Atunci:
4
X

(3)

(1)k1 IA2k Sk = S (IA1 IA3 IA2 IA4 ),

k=1

unde Sk este aria triunghiului opus v


arfului Ak , k = 1, 2, 3, 4.
Corolarul 2. Dac
a patrulaterul I1 I2 I3 I4 este inscriptibil (deci convex) si I este
punctul de intersectie a diagonalelor, atunci
4
X

(4)

(1)k1 IA2k Sk = 0.

k=1

Demonstratie. In (3), membrul al doilea este nul, caci produsele din paranteza
re[rezint
a puterea punctului I fata de cercul circumscris patrulaterului.
Corolarele pot extinse n spatiu; facem acest lucru numai pentru Corolarul 1.
conditiile si cu notatiile din Corolarul 1, dac
Propozitia 2. In
a M este un punct
n spatiu, atunci are loc relatia
4
X

(5)

(1)k1 M A2k Sk = S (M A1 M A3 M A2 M A4 ).

i=1

Demonstratie. Vectorial, relatia (5) se scrie:


4
X


(1)k1 (M I + IAk )2 Sk = S (M A1 M A3 M A2 M A4 )

i=1

sau
(6)

4
X

(1)k1 (M I 2 +IA2k 2M IIAk cos M


IAk )Sk = S(M A1 M A3 M A3 M A4 ).

i=1

Scaz
and din (6) relatia (3) si utilizand faptul ca avem: S1 + S3 = S2 + S4 = S,

cos M IA3 = cos M


IA1 si cos M
IA4 = cos M
IA2 , obtinem succesiv:
(1)

4
X

(1)k1 IAk cos M


IAk Sk = 0,

k=1

(7)

(IA1 S1 IA3 S3 ) cos M


IA1 = (IA2 S2 IA4 S4 ) cos M
IA2 .

1
1
1
Cum S1 = IA3 A2 A4 sin , S2 = IA4 A1 A3 sin , S3 = IA1 A2 A4 sin ,
2
2
2
1
S4 = IA2 A1 A3 sin , prin nlocuire n (7) obtinem 0 = 0, ceea ce ncheie stabilirea
2
egalitatii (5).
Observatie. Propozitia 2 a fost obtinuta anterior, pe alta cale, de I. Radu n
articolul O aplicatie a relatiei lui Stewart (G.M.-10-2009, Teorema 2, p. 469).
14

Observatii asupra cercurilor seminscrise


Neculai ROMAN

Abstract. In this Note, a couple of particular positions for the A-mixtilinear circle associated
to a triangle are considered, and the possible conditions for these positions to occur are established.
Keywords: incenter, circumcenter, mixtilinear center.
MSC 2010: 51M04.

1. Fie ABC un triunghi oarecare si O, I centrele cercurilor circumscris si nscris


lui. Vom nota n mod obisnuit razele acestor cercuri cu R si r.
Un cerc tangent dreptelor AB si AC si tangent interior cercului circumscris se
numeste A-seminscris (sau A-mixtiliniar nscris) triunghiului ABC. Analog se introduc cercurile B-seminscris si C-seminscris.
In aceasta Nota ne propunem sa vedem n ce conditii aceste cercuri au anumite
pozitii particulare. Ne vom referi numai la cercurile A-seminscrise, carora le vom
spune, pe scurt, seminscrise. Vom nota cu C(I1 , r1 ) cercul seminscris avand centrul
I1 si raza r1 . Punctele de tangent
a cu AB si AC vor notate cu B1 , respectiv C1 ,
iar punctul de tangent
a cu cercul circumscris cu T . Vom mai nota cu A1 mijlocul
ce nu contine varful A. Evident, punctele I, I1 si A1 se aa pe bisectoarea
arcului BC
b si avem limitarile: r < r1 < R si AI < AI1 < AA1 .
unghiului A
Cercurile seminscrise au fost studiate n lucrarile [1], [2], [3], [4] s.a. Amintim
cateva proprietati ale cercului seminscris C(I1 , r1 ):
r
r1 =
(formula fundamencos2 A2
tala);
r1
r
AI1 =
=
;
sin A2
sin A2 cos2 A2
I este mijlocul coardei B1 C1 a
cercului C(I1 , r1 ).
punctele O, I1 , T sunt coliniare.
1

A.

Avem n vedere particularitati ale


cercului seminscris C(I1 , r1 ) de tipul:
1
I1 are o pozitie anumit
a pe bisectoare, cercul C(I1 , r1 ) trece printr-un
punct important al triunghiului ABC,
tangenta n T are o pozitie anumit
a
fata de laturile triunghiului, punctele
de contact B1 , C1 , T au pozitii particulare s.a. Vom examina doar cateva cazuri simple.

.I

.B

.C
O.

I
..1 .

1 Profesor,

S
coala general
a ,,Vasile Alecsandri, Mircesti (Iasi)

15

.
A1

2. C(I1 , r1 ) trece prin A1 . Evident, n acest caz A1 este punctul de tangenta a


cercului C(I1 , r1 ) cu cercul circumscris. Ca urmare, punctele A1 , I1 si O sunt coliniare.
b rezulta ca triunghiul ABC este
Din faptul ca O se aa pe bisectoarea unghiului A
isoscel.
3. C(I1 , r1 ) trece prin O. Conditia se scrie OI1 = r1 si se traduce prin
R r1 = r1 (caci O, I1 , T sunt coliniare). Ca urmare, R = 2r1 sau
()

R=

2r
.
cos2 A2

Propozitia 1. Dac
a O C(I1 , r1 ), atunci
A
(i) 8r = a ctg , 8r2 = a(p a);
2
a
a
A
A
(ii) tg2 =
, sin2 =
;
2
8(p a)
2
8(p a) + a
8(p a) a
(iii) cos A =
;
8(p a) + a
2bc A
(iv) R =
tg .
p
2
a
Demonstratie. (i) Cu 2R =
se elimina R din relatia () si se obtine
sin A
prima formul
a. A doua rezulta din prima, prin nmultirea sa cu relatia cunoscuta
A
r = (p a) tg .
2
A
(ii) Eliminam r din prima relatie (i) utilizand faptul ca r = (p a) tg
etc.
2
A
(iii) Se tine seama de formula 1 cos A = 2 sin2 .
2
2r
A
bc
2bc A
(iv) Avem: R =
=
2(p

a)
tg

=
tg .
2 p(p a)
p
2
cos2 A2
R
, poate privita ca
2
ind conditia n care cercul seminscris este egal cu cercul celor noua puncte (Euler)
sau, nc
a, conditia n care O este diametral opus n C(I1 , r1 ) punctului T .
Observatie. Conditia O C(I1 , r1 ), echivalenta cu r1 =

4. C(I1 , r1 ) are centrul pe BC (I1 BC). Adica I1 este piciorul bisectoarei,


b Deoarece
ceea ce revine la conditia AI1 = la (lungimea bisectoarei unghiului A.)
2bc
A
la =
cos , avem:
b+c
2
2bc
A
2r
2bc
A
r
=
cos
=
cos2
A
2
b+c
2
sin A
b+c
2
sin cos 2
A
r(b + c)
A
r(b + c)
A
b+c
cos2 =
cos2 =
cos2 =
.
2
bc sin A
2
2S
2
2p
A
2

16

Propozitia 2. Dac
a I1 BC, atunci au loc:
A
b
+
c
A
a
(i) cos2 =
, sin2 =
;
2
a+b+c
2
a+b+c
pa
r
(ii) cos A =
=
(ra -raza cercului A-exnscris).
p
ra
a
A
Cu formulele 2R =
si r = (p a) tg si tinand seama de propozitia precesin A
2
denta, obtinem:
Propozitia 3. Dac
a I1 este piciorul bisectoarei, au loc formulele:
1 a 2
a
(i) R2 =
p , r2 =
(p a)2 ;
4b+c
b+c
r
= 2 cos A, OI 2 = R2 (1 4 cos A).
(ii)
R
Observatii. 1) Daca ABC are proprietatea I1 BC, atunci el este ascutiunghic.
Intr-adev
ar, se constata usor, prin calcul, c
a A 90 implica AI1 > la .

a
, avem:
2) In aceeasi conditie si notand tg =
2
b+c
cos A =

deci A = = 2 arctg

1 tg2
pa
a + b + c
=
=
p
a+b+c
1 + tg2

= cos ,

a
a
. Observam ca raportul
are un rol important n
b+c
b+c

cazul considerat.
Bibliograe
1. L. Banko A Mixtilinear Adventure, Crux Math., 9 (1983), 2-7.
2. A. Ghirici C
ateva probleme asupra triunghiurilor si cercurilor (l. rusa), Kvant, 11
(1990), 46-48.
3. K.L. Nguyen, J.C. Salazar On Mixtilinear Incircles and Excircles, Forum Geom.,
6 (2006), 1-16.
4. P. Yiu Mixtilinear Circles, Ann. Math. Monthly, 106 (1999), 952-955.

Cu betisoare avem scris:

a) Mutati un betisor pentru a obtine egalitate; b) Mutati doua betisoare pentru


a obtine egalitate; c) Mutati trei betisoare pentru a obtine egalitate.
Nicolae Iv
aschescu, Craiova
(R
aspunsuri la pag. 35)
17

Multiplicative duality in triangles


Holger STEPHAN

Abstract. Using the general duality principle in the geometry of the triangle, the authors
suggests how it is possible to prove both well-known and new formulas.
Keywords: duality, area, altitude, incercle, escribed cercles.
MSC 2010: 51M04, 97D40.

Duality is one of the most fruitful principles in mathematics. Simply said, duality
means that every thing has two sides, or every problem can be considers from two
dierent sides. This has many advantages: Mostly the diculties of the two sides are
dierent. Thus, we can choose the easier way to solve the problem. If you have found
an interesting feature of an object its dual property is often interesting, too.
An important example of duality in mathematics is governed by the product of
two dierent values. Looking at a product as the product of two real numbers, due
to commutativity it is identical which of the two numbers is the rst, which is the
second. But often the values if they describe real things have dierent meanings.
In physics this is well know. Any value has a unit, e.g., power is voltage times current.
In Euclidean geometry such an example is the area, the product of two lengths.
Although, they have the same units, being perpendicular to each other, they point in
dierent directions. This is especially interesting for the triangle, because there is no
way to point out two xed directions.
1. The area of a triangle as a dual product sizes. We consider a given
triangle ABC with side lengths a, b, c and area SABC . This area can be calculate
as the half of the product of one side and the corresponding altitude. We denote the
altitudes by hA , hB and hC . To forget about the half in the product, we denote the
half of the side lengths by an index 2: a2 = a/2, b2 = b/2, c2 = c/2. Now, we have
(2)

SABC

= a2 hA = b2 hB = c2 hC .

If we have two variables x and y and the area of the triangle is S = xy, we call the
variables x and y dual to each other. If we choose the length unit in such a way,
that the area of our triangle is 1, then x = y1 . If we have a formula or some other
relationship with dierent variables of type x, then we get a similar relationship for
the dual variables y, by replacing the x-values by y1 .
2. Heron s area formula for the altitudes. Given the sides of a triangle, then
we can calculate the area by Heron s formula:
S=
1 Weierstrass

1
(a + b + c)(a + b + c)(a b + c)(a + b c).
4

Institute for Applied Analysis and Stochastics; e-mail: stephan@wias-berlin.de

18

We introduce new variables (in the sequel this turns out to be very useful):
a+b+c
,
2
a + b + c
pA = a2 + b2 + c2 =
,
2
ab+c
pB = a2 b2 + c2 =
,
2
a+bc
pC = a2 + b2 c2 =
;
2
then Heron s formula is equivalent to
p = a2 + b2 + c2 =

(3)

S2

= p pA pB pC =
= (a2 + b2 + c2 )(a2 + b2 + c2 )(a2 b2 + c2 )(a2 + b2 c2 ).

Here, p is the semiperimeter (one half of the triangles perimeter). The values pA , pB
and pC are parts of the side lengths, dened by the touch point of the incircle (see
the picture and the text below).
If we replace in (2), the half sides by their dual quantities, the altitudes, i.e.,
a2 h1A , b2 h1B , c2 h1C , we obtain a formula to calculate the area of the triangle
for given altitudes:
(4)

1
1
1
1
1
1
1
1
1
1
1
1
1

.
=
+
+
+
+

+
+

S2
hA
hB
hC
hA
hB
hC
hA
hB
hC
hA
hB
hC

AC
pB

pA

pC C p
C
r r
I

pA

r
Ic

rA
IA
pBIa rA
pC
pB

rA

B pC AB

3. The area, the incircle and the escribed circles. Besides formula (2), there
are other lengths in a triangle, giving the area as their product. For this purpose we
draw in a triangle the inscribed circle with radius r and escribed circles (touching the
sides from outer) with radii rA , rB and rC .
The values of the side partitions pA , pB and pC are easily calculated from the
system of equations
(5)

a = 2a2 = pB + pC , b = 2b2 = pC + pA , c = 2c2 = pA + pB .


19

In addition, the formulas


(6)

p = pA + pB + pC ,

and
pA = p 2a2 , pB = p 2b2 , pC = p 2c2

(7)

are easy to check, too.


Now, the area can be calculated by
SABC = SABI + SBCI + SCAI = a2 r + b2 r + c2 r = pr,
and
SABC

= SAAB IA + SAAC IA SBAB IA SBIa IA SCIa IA SCAC IA =


= prA pC rA pB rA = rA pA

and the analogous formulas for the other escribed circles.


Thus, we obtain (together with formula (2)) the following representations for the
area
(8)

S = a2 hA = b2 hB = c2 hC = pr = pA rA = pB rB = pC rC .

It provides another four pairs of dual variables: (r, p), (rA , pA ) (rB , pB ) (rc , pC ).
The sides of the triangle are connected with the side partitions by the formulas
(5), (6) and (7) in a very simple manner. By duality, we obtain easily relationships
B rC
of the dual variables that are not so obvious, such as the formula hA = r2r
,
B +rC
a relationship between an altitude and two radii of the escribed circles. Replacing
a2 h1A , pB r1B and pC r1C , from 2a2 = pB + pC (formula (4)) follows
2
1
1
=
+
,
hA
rB
rC
or hA =

2rB rC
rB +rC .

Analogously, can be derived the following relationships:


1
1
1
1
=
+
+
,
r
rA
rB
rC
1
1
1
1
p = a2 + b2 + c2 =
+
+
,
r
hA
hB
hC
1
1
2
1
1
1
pA = p 2a2
=
=
+
+
.
rA
r
hA
hA
hB
hC
p = pA + pB + pC

4. Two pairs of dual quadruples. Using the last relationships, from Heron s
formula follows S12 = rrA r1B rC and therefor
S 2 = ppA pB pC = rrA rB rC .
20

The product of the four values p, pA , pB , pC is equal to the product of their dual
values r, rA , rB , rC . Each of the quadruples (p, pA , pB , pC ) and (r, rA , rB , rc ) consist
of a value, which occurs only once in the triangle namely, p and r and three
cyclical variables. Examining formula (7) seven products, resulting the area from
an aesthetic point of view, it is immediately apparent that there is a product S = xy
missing. The unknowns x and y should be triangle sizes that exist only once and
form with the cyclic values a2 , b2 , c2 and hA , hB , hC quadruples (x, a2 , b2 , c2 ) and
(y, hA , hB , hc ) similar to (p, pA , pB , pC ) and (r, rA , rB , rc ).
2
The value x should satisfy x = a2Sb2 c2 . With the well known formula 4RS = abc,
where R is the circumradius, follows
x=

S2
8S 2
2S
=
=
a2 b2 c2
abc
R

Thus,
S=

R
x.
2

Obviously, y = R2 =: R2 , half of the circumradius, should be one of the unknown.


The other length x = RS2 can be found in the triangle, too. But this is not
so easy and a nice exercise. The answer is RS2 = uH is the semiperimeter of the
triangle formed by the foot points of the altitudes the so-called altitude triangle.
The value R2 is not only the half of the circumradius, but also the circumradius of
the altitude triangle. The circumcircle of the altitude triangle is a very famous circle
in the triangle may be the most important circle in a triangle at all. It is called
Feuerbach s circle (after the mathematician who rst described it) or nine-point circle,
because it passes through nine signicant points. These are, besides the three foot
points of the altitudes, the midpoints of the sides of the triangle and the midpoints
of the upper parts of the altitudes (the line segments from each vertex of the triangle
to the orthocenter).
Finally, we obtain the following relations
S
S2

= R2 uH = a2 hA = b2 hB = c2 hC = pr = pA rA = pB rB = pC rC ,
= ppA pB pC = rrA rB rC = R2 a2 b2 c2 = uH hA hB hC .

These amazing formulas raise the question, are there some other dual quadruples in
the triangle?

Numarul palindromic 918273645546372819 poate rezultatul unui calcul ce


comporta doar doua operatii aritmetice cu numere ce se scriu cu cifrele 0 si 1 (n baza
de numeratie 10)?
(R
aspuns la pag. 28)
21

Autour de l
equation X 2 = A dans Mn (R)
Adrien REISNER1
Abstract. Westudies in this article some
algebraic and topological properties of the set of
matrices Rac(A) = X Mn (R); X 2 = A , where A is an element of Mn (R).
Keywords: symmetrical matrix, characteristic polinomial, eigenvector, eigenvalue, closed set.
MSC 2010: 15B99, 81U20.

1.
Etant donne A Mn (R), on note Rac(A) l ensem Notations et2 denitions.

ble X Mn (R); X = A et on appelle racine carree de A tout element de Rac(A).
On se propose letude de quelques proprietes elementaires algebriques et topologiques
de cet ensemble. Sn+ (R) designera l ensemble des matrices reelles symetriques positives (i.e. A Sn+ (R) si et seulement si A = tA Mn (R) et X, t XAX 0).
2. Etude alg
ebrique: cas particuliers, exemples.
Th
eor`
eme 1. Si A Mn (R) admet n valeurs propres reelles distinctes 1 <
2 < ... < n , alors Rac(A) = si 1 < 0, CardRac(A) = 2n1 si 1 = 0,
CardRac(A) = 2n si 1 > 0.
D
emonstration. La matrice A ayant n valeurs propres distinctes, cette matrice est diagonalisable. Il existe donc P GLn (R) telle que A = P DP 1 , o`
u
D = diag(1 , 2 , . . . , n ). Soit alors R Mn (R) et S = P 1 RP . On a R2 = A si
et seulement si P 1 R2 P = D, i.e. S 2 = D. Par suite Rac(A) = P Rac(D) P 1 .
Soit S une racine carree de D. Il vient: SD = S 3 = DS. Le produit matriciel
SD = DS montre alors les i etant distincts que la matrice S est elle-meme
diagonalisable: S = diag(s1 , s2 , . . . , sn ) et que s2i = i , i = 1, 2, . . . , n. On en deduit
que si
on a: Rac(D)
1 < 0, alors Rac(A) = . Si tous les i sont positifs,

{diag(i i )i:1,...,n ; i = 1}. Reciproquement, si S = diag(i i )i:1,...,n o`


u i = 1,
alors S 2 = D, et l inclusion ci-dessus est une egalite. L application M P 1 M P
est une bijection de Rac(A)
d o`
u: si 1 < 0, Rac(A) = ; si 1
dans Rac(D),
1
0, Rac(A) = {P diag(i i )i:1,...,n P ; i = 1}. Finalement, le theor`eme est
demontre, en distinguant respectivement les cas o`
u 1 = 0 et 1 > 0.

11 5
5
5
3 3
Exemple. Soit `a determiner Rac(A) o`
uA=
. Le polynome
5 3
3
caracteristique de la matrice A etant A (X) = X(X 2 17X +16) = X(X 1)(X 16),
son spectre est Sp(A) = {0, 1, 16}. Determinons les sous-espaces propres de la matrice
A. Le noyau de A, sous-espace associe `a la valeur propre 1 = 0, est la droite Rv1
o`
u t v1 = (0, 1, 1). Le sous-espace propre associe `a la valeur propre 2 = 1 est le
sous espace Rv2 o`
u t v2 = (1, 1, 1). Enn Rv3 o`
u t v3 = (2, 1, 1) est le sous-espace
propre de la matrice A associe `a la valeur
propre
3
= 16. On a donc: P 1 AP =

0
1
2
1
1 1
diag(0, 1, 16), o`
u P est la matrice P =
. La matrice A admet quatre
1 1
1
1 TELECOM

ParisTech; e-mail: Adrien.Reisner@telecom-paristech.fr

22

racines carrees qui sont: P diag(0, 1, 4)P 1 , P diag(0, 1, 4)P 1 , P diag(0, 1, 4)P 1
1
et P diag(0,
, soit explicitement:
1, 4)P

3 1
1
7 5
5
1
1 1
5
1 1
R1 =
, R2 = 31
, R3 = R2 et R4 = R1 .
1 1
1
5 1
1
Remarque. La matrice A etant symetrique, A est diagonalisable, les sous-espaces
propres Rvi , i : 1, 2, 3, de cette matrice sont orthogonaux deux `a deux ce qu on verie immediatement (ainsi la matrice de passage P peut etre choisie orthogonale, i.e.
vi
veriant P 1 = tP , en remplacant les trois vecteurs vi par les vecteurs
, i : 1, 2, 3.
vi
Th
eor`
eme 2. On a:

O Ir
Rac(O) = {PM r P 1 ; P GLn (R), M r =
, avec r N [1, n/2]} {O}.
O O
D
emonstration. Soit R Mn (R) une racine carree de la matrice nulle et
designons par f l endomorphisme de Rn dont R est la matrice dans la base canonique
de Rn . Ayant f f = 0, il vient Imf Kerf et par suite `a partir de legalite
1
rgf + dimKerf = n : rgf = r n. Soit (e1 , . . . , er ) une base de Imf que l on
2
compl`ete avec (er+1 , . . . , enr ) pour former une base de Kerf . Si ui , i : 1, . . . , r est
un vecteur tel que f (ui ) = ei , soit B = (e1 , . . . , enr , u1, . . . , ur ). Montrons que les n
elements de B forment une famille libre, i.e. que B est une base de Rn . Supposant

nr

que

i ei +

i=1

nr
i=r+1

alors:

i ei +
r
P
i=1

r
P

r
P

i ui = 0, il vient, avec les notations precedentes:

i=1

r
P

i f (ui ) +

i=1

i ui = 0. Ayant f 2 = 0 et f (ei ) = 0 pour i : r +1, . . . , nr, on obtient

i=1

i f (ui ) =

r
P

i ei = 0. La famille {ei }i=1,...,r etant libre, i = 0, i : 1, . . . , r,

i=1

et nalement B est libre puisque les (e1 , . . . , enr ) sont libres


. Dans
cette base B, la
O
I
r
matrice de l endomorphisme f est: M at(f, B) = Mr =
. Si R Rac(O),
O O
1
alors soit R = O, soit R est semblable `a une matrice de type Mr avec r n.
2
1
1
2
Reciproquement, si r n, un calcul elementaire prouve que Mr =O (car r n),
2
2
donc que toute matrice semblable `a Mr est de carre nul.

Exemple. Les matrices carr


la matrice nulle
et les
ees d ordre 4 decarre nul sont
0 0 0 1
0 0 1 0
0 0 0 0
0 0 0 1
matrices semblables `a M1 =
ou `a M2 =
.
0 0 0 0
0 0 0 0
0 0 0 0
0 0 0 0
Th
eor`
eme 3. Rac(I n ) = {Pdiag(1 , 2 , . . . , n ); P GLn (R), i {1 , +1 }, i }.
D
emonstration. Soit R telle que R2 = I; alors detR = 1 et R est inversible

(d ailleurs d inverse elle-meme). Le polynome X 2 1 est un polynome annulateur


de la matrice R. C est un polynome scinde `a racines simples sur R et par suite R
est R-diagonalisable et les valeurs propres appartiennent `a l ensemble {1, 1}. Donc
RacI {P diag(1 , 2 , . . . , n )P 1 ; P GLn (R), i = 1, i : 1, . . . , n}.

23

Reciproquement, D = diag(1 , 2 , . . . , n ) verie D2 = I et il en est de meme pour


toute matrice semblable `a la matrice D. L inclusion precedente est donc une egalite.
Th
eor`
eme 4. Toute matrice A reelle symetrique positive admet au moins une
racine carree qui est elle meme symetrique et positive: Rac(A) Sn+ (R) = .
D
emonstration. Soit A une matrice reelle symetrique positive. Cette matrice
est donc ortho-diagonalisable: A = P DP 1 = P D t P, avec D = diag(1 , 2 , ..., n )
o`
u i 0, i: 1, . . . ,
n la matrice A etant positive et P une matrice orthogonale. Si
= diag( 1 , ..., n ), la matrice R = P t P est symetrique positive, car elle est
semblable `a la matrice positive (Sp(R) = Sp() R+ ). Enn R2 = P 2 t P = A.
Ainsi A admet au moins une racine carree R, egalement symetrique positive.
Remarques. 1)Toute matrice symetrique n admet pas forcement de racine carree
dans Mn (R), comme le prouve la matrice (1) M1 (R).
2) La matrice A de l exemple du Theor`eme 1 verie bien les hypoth`eses du
theor`eme precedent. On verie qu eectivement les quatre racines carrees de A sont
elles memes symetriques. R1 est de plus positive.
3. Etude topologique: Rac(A) ensemble ferm
e. On munit Mn (R) de la
norme: N (A) = max |aij | , si A = (aij ).
1i,jn

Remarque. Mn (R) etant de dimension nie, toutes les normes y sont equivalentes. Pour toute matrice A Mn (R), les proprietes topologiques de Rac(A) restent
inchangees lorsqu on remplace la norme N par toute autre norme de Mn (R).
Th
eor`
eme 5. a) Pour toute A Mn (R), Rac(A) est une partie fermee de Mn (R).
b) Rac(In ) n est pas une partie bornee de Mn (R).
D
emonstration. a) Soit A Mn (R). L application R R2 est continue sur
Mn (R) (chaque fonction coordonnee est continue comme fonction polynomiale des
coecients de R). Si (Rk ) est une suite convergentes de matrices de Mn (R) de limite
R, alors Rk2 R2 . Donc (Rk ) etant une suite convergente delements de Rac(A), la
limite appartient elle-meme `a Rac(A). Ainsi Rac(A)
est un
ensemble ferme de Mn (R).
1
0
b) Considerons pour q N la matrice Sq =
. On a: N (Sq ) = max(|q|, 1)
q 1
lorsque q et Sq2 = I2 . On en deduit que Rac(I2 ) n est pas bornee.

Sq
O
Denissons alors lorsque n 3 la matrice par blocs Mq =
. Cette matrice
O Inq
verie: Mq2 = In , N (Mq ) lorsque q et par suite Rac(In ) n est pas bornee
dans Mn (R) pour la norme N, donc aussi pour toute autre norme de Mn (R).
On appelle norme sur-multiplicative sur GLn (R) une norme N veriant, pour
tout A, B GLn (R) l inegalite: N (AB) N (A)N (B).
Corollaire 6. Pour n 2 il n existe pas de norme sur-multiplicative sur GLn (R).
D
emonstration. Compte tenu du theor`eme precedent il existe dans Rac(In )
une suite (Rk ) non bornee. Supposons alors par l absurde qu il existe sur GLn (R)
2
une norme N sur-multiplicative. On aurait alors: N (In ) = N (Rk2 ) [N (Rk )] . Le
membre de gauche est constant alors que celui de droite est de limite innie, les deux
normes N et N etant equivalentes voir remarque precedente , d o`
u contradiction.
24

Reguli de tip Sarrus


pentru calculul determinantilor de ordin 3
Constantin DRAGOMIR

Abstract. In this Note some type Sarrus procedures for calculation of the determinants of third
order are presented.
Keywords: determinant, Sarrus rule.
MSC 2010: 97H20.

Valoarea unui determinant de ordin 3 este data de formula


(1)


a11 a12 a13


a21 a22 a23 = a11 a22 a33 +a21 a32 a13 +a31 a12 a23 a13 a22 a31 a23 a32 a11 a33 a12 a21 ,


a31 a32 a33
elementele determinantului apartin
and unui corp K (n particular R sau C).
Pentru calculul practic al acestei valori este binecunoscuta regula lui Sarrus (Pierre
Fr
ed
eric Sarrus, 1798-1861), pe care este util sa o reamintim.
Regula lui Sarrus cu linii presupune trei etape: 1L) se scriu primele dou
a linii
sub determinant; 2) se pun n evident
a trei diagonale principale si trei secundare; 3)
produsele elementelor de pe fiecare dintre diagonalele principale sunt termenii cu +
din (1), iar cele relativ la diagonalele secundare sunt termenii cu .
Regula lui Sarrus cu coloane foloseste coloane n loc de linii, ceea ce revine la
nlocuirea etapei 1L) cu: 1C) se scriu primele dou
a coloane la dreapta determinantului
si pastrarea etapelor 2) si 3). Aceste reguli sunt ilustrate de schemele urmatoare:

(2)

a11
a21
a31
a11
a21

a12
a22
a32
a12
a22

a13
a23 ,
a33
a13
a23

a11
a21
a31

a12
a22
a32

a13
a23
a33

a11
a21
a31

a12
a22
a32

In aceasta Nota vom prezenta alte cateva procedee (reguli) de calcul al determinantului de ordin 3 nrudite cu regulile de mai sus si diferind de ele doar prin constructia
auxiliar
a din etapa 1L) sau 1C) - celelalte doua etape ramanand neschimbate.
Primul procedeu. a) cu linii. Se vor urma etapele: 1 L) se scrie prima linie
a determinantului sub determinant si linia a treia deasupra lui ; 2 L) diagonalele
principale si secundare sunt marcate n prima schema din 3); 3 L) aceeasi ca 3).
b) cu coloane. Se parcurg etapele: 1 C) se scrie prima coloana a determinantului
la dreapta lui si coloana a treia la stanga lui; 2 C) diagonalele sunt marcate n a doua
schema din (3); 3 C) aceeasi ca 3).
1 Profesor,

Liceul Teoretic ,,Ion Barbu, Pitesti

25

a31
a11
a21
a31
a11

(3)

a32
a12
a22
a32
a12

a33
a13
a23
a33
a13

a13
a23
a33

a11
a21
a31

a12
a22
a32

a13
a23
a33

a11
a21
a31

Al doilea procedeu. Este un procedeu mixt, care utilizeaza atat linii cat si
coloane. In prima etapa efectuam urmatoarele operatii: prima linie a determinantului
se pune sub el, a treia linie se pune deasupra lui, prima coloana se pune la dreapta
lui, iar ultima coloana la stanga lui.
In a doua etapa indicam doua moduri de a pune n evidenta diagonalele, ce se
desprind usor din schemele urmatoare:

(4)

a13
a23
a33

a31
a11
a21
a31
a11

a32
a12
a22
a32
a12

a33
a13
a23
a33
a13

a11
a21
a31

a13
a23
a33

a31
a11
a21
a31
a11

a32
a12
a22
a32
a12

a33
a13
a23
a33
a13

a11
a21
a31

In ne, n a treia etapa se vor scrie termenii sumei din (1) la fel ca si n toate
procedeele precedente.
Observatie. Practic, nu se vor scrie n ntregime liniile si coloanele care bordeaza
determinantul, ci numai elementele care sunt necesare la trasarea diagonalelor. Astfel,
la prima schem
a din (4) se vor scrie n afara determinantului numai a13 , a31 , a11 si
a33 , iar la a doua schem
a numai a11 , a33 , a31 si a13 .
Al treilea procedeu. Este simetric fata de linii si coloane si nu comporta decat
o singura variant
a. Nu vom mai borda determinantul cu linii si coloane, ci vom scrie
n afara lui numai elementele necesare. Etapele procedeului sunt: 1) n dreptul liniei
de mijloc a determinantului se scriu: a21 la dreapta si a23 la stanga, iar n dreptul
coloanei de mijloc se scriu: a12 sub determinant si a32 deasupra lui; 2) diagonalele
sunt indicate n schema ce urmeaza:

(5)

a23

a11
a21
a31

a32
a12
a22
a32
a12

a13
a23
a33

a21

3) se scrie suma din (1) n mod obisnuit.


Observatii. 1) Comparativ, utlimul procedeu are avantajele: este simetric, diagrama (5) se formeaza usor, toate elementele scrie sunt folosite.
2) Schemele de mai sus folosesc ca elemente grace numai linii drepte; regula
triunghiurilor foloseste atat linii drepte cat si triunghiuri.
26

Utilizarea injectivit
atii functiilor
n rezolvarea ecuatiilor
Mih
aly BENCZE1
Abstract. In this Note a procedure involving the injectivity of the functions in order to solve
an inequation of the form (1) is presented.
Keywords: equation, strictly increasing function, injective function.
MSC 2010: 97H20.

Vom prezenta si ilustra cu exemple un procedeu de rezolvare a ecuatiilor algebrice, trigonometrice, exponentiale etc. sau a unor ecuatii ce combina aceste tipuri.
Procedeul consta n a pune o ecuatie
F (x) = 0, x I,

(1)

(I - interval al spatiului R), sub forma


f (g(x)) = f (h(x)), x I,

(2)

unde f : J R este o functie injectiv


a pe J (se subntelege ca functiile g si h au
valorile n J). Datorita injectivitatii functiei f , ecuatia (2) se reduce la
g(x) = h(x), x I,

(3)

care, n general, este mai simpla.


Alegerea functiei f moment cheie al procedeului, atunci cand acest fapt este
posibil, nu este un lucru usor de facut, ceea ce face ca aplicarea procedeului sa e
limitata.
Exemplul 1. S
a se rezolve n R ecuatia

x6 + 4x2 4x + 2 = (x 1) x 1.


Ecuatia este denita pentru x 1. Sa consideram functia f (t) = t+ t+ 3 t, t 0,
si sa observam ca ea este strict crescatoare, deci este injectiva. Se constata prin calcul
direct ca
f (x6 ) = f ((x 1)3 ).
Datorita injectivitatii functiei f , obtinem ecuatia
x6 = (x 1)3 , x 1,
care este echivalent
a cu x2 = x 1, x 1, ecuatie ce nu are solutii n R. Asadar,
ecuatia data nu admite solutii.
Exemplul 2. S
a se rezolve n R ecuatia

x
2
2(2 1) log2 ( x + 1) = 4x + 2x+1 + x.
1 Profesor,

Colegiul National ,,Aprily Lajos, Brasov

27

Domeniul de denitie a ecutiei este x 0. Functia f (t) = 2t + t este strict


crescatoare si injectiva pe R. Ecuatia este echivalenta cu

f ((2x 1)2 ) = f (log2 ( x + 1)), x 0.


Deducem ca

(2x 1)2 = log2 ( x + 1), x 0.

Cu notatia y = (2x 1)2 = log2 ( x + 1) , urmeaza ca 2y 1 = x, 2x 1 = y si,

deci, 2x + x = 2y + y, de unde deducem ca x = y (functiag(t) = 2t + t, t 0,


x
2
x
ind strict crescatoare si injectiva). A
sadar, x = (2 1) sau x = 2 1, ecuatxie ce
are doua solutii: 0 si 1 (caci (x) = x, x 0, este strict concava, iar (x) = 2 1
este strict convex
a). In concluzie, ecuatia data admite solutiile x = 0 si x = 1.
Exemplul 3. S
a se rezolve ecuatia
log2 (xlog2

3
5

+ xlog2 5 ) + x2 log2

3
5

+ x2 log2

4
5

+ 2xlog2

12
25

= 1, x > 0.

Ecuatia data este echivalenta cu


f (xlog2

3
5

+ xlog2 5 ) = f (1), x > 0,

unde functia f este denita prin f (t) = log2 t + t2 , t > 0, este strict crescatoare si
injectiva. Ca urmare,
3
4
xlog2 5 + xlog2 5 = 1.
3

Aceasta ecuatie are o singura solutie: x = 4, ntrucat functia g(t) = tlog2 5 +tlog2 5 , t >
0, este strict descrescatoare (exponentii ind negativi). In nal, ecuatia data are
solutia x = 4.
Exemplul 4. S
a se rezolve ecuatia

512 sin9 x + 54 sin 3x + 57 3 = 216 sin x + 54 3 cos 2x, x R.


Utiliz
and formulele cos 2x = 1 2 sin2 x si sin 3x = 3 sin x 4 sin3 x, vom scrie
ecuatia precedent
a n forma

(8 sin3 x)3 = (6 sin x 3)3

3
3
3
(i.e., f (t) = t , t R); deci 8 sin x = 6 sin x 3 sau, nca, sin 3x =
. Prin
2

k
urmare, x (1)k +
;k Z .
9
3

R
aspuns la ,,recreatia de la pag. 21:
918273645546372819 = (1010
| {z. . . 10} 1) : 11.
20 cifre

Vericarea se face prin calcul direct.


28

Problema L222 din nr. 1/2012 revizitat


a
Nota Redactiei. Problema L222 a atras atentia mai multor autori, care au trimis
observatiile lor redactiei revistei. Este vorba de inegalitatea

(1)

1
1
+ 2
b2
c

+b

1
1
+ 2
c2
a

+c

1
1
+ 2
a2
b

18
,
a+b+c

a, b, c R+ ,

Florin St
anescu Recreatii Matematice - 1/2012
care se poate scrie si n forma

b+c c+a a+b


18
,
+ 2 + 2
2
a
b
c
a+b+c

(2)

a, b, c R+ .

In num
arul urm
ator, Recreatii Matematice - 2/2012, 120-123, sunt date mai multe solutii
pornind de la forma (2) a inegalit
atii si utiliz
and inegalitatea mediilor, inegalitatea Cebsev,
inegalitatea Bergstr
om sau a altor procedee de c
atre Fl. St
anescu, autorul problemei, T.
Zvonaru si D. V
acaru.

Intre timp, Ioan Viorel Codreanu ne trimite trei solutii pornind de la forma
(1) a inegalitatii, iar Dumitru M. B
atinetu-Giurgiu si Neculai Stanciu propun
o generalizare a acesteia. Publicam mai jos aceste materiale.
I. Solutii la Problema L222 date de I.V. Codreanu.
Solutia 1. Folosind inegalitatea mediilor obtinem

X 1
a

b2

1
c2

Xa
bc

Este sucient sa arat


am ca
2

Xa

18
P
bc
a

X X 2

9abc.

P
P 2
2
Folosind inegalitatea mediilor avem:
a 3 3 abc si
a 3 3 (abc) si, dupa
nmultirea acestor inegalitati, obtinem rezultatul dorit.
Solutia 2. Folosind inegalitatea Bergstrom, vom avea

X 1

1
a 2+ 2
b
c

si cum (

a)

P
1
a

1
b2
1
a

1
c2
1
a

P 1 2
2 P

a
1
a

=2

X1

9, obtinem imediat inegalitatea din enunt.


29

1
1
1
1
1
1
Solutia 3. Putem presupune ca a b c; ca urmare 2 + 2 2 + 2 2 + 2 .
b
c
c
a
a
b
Folosind inegalitatea lui Cebsev, obtinem:
3

X 1

1
a 2+ 2
b
c

X X 1

X X 1
1
a
+ 2
=2
a

b2
c
a2

X1
2
2 X X 1 2
54
a
9
P ,
3
a
3
a
a

dupa care rezulta inegalitatea de stabilit.

II. O generalizare a Problemei L222


dat
a de D.M. B
atinetu-Giurgiu si N. Stanciu.
Generalizarea pe care o propun autorii este urmatoarea:
Dac
a a, b, c R+ si m [1, ), atunci

(3)

1
1
+ m
bm
c

+b

1
1
+ m
cm
a

+c

1
1
+ m
am
b

2 3m

m1 .

(a + b + c)

Demonstratia 1. Avem:

X 1
a

bm

1
cm

X am+1 + bm+1
a
b
b
c
c
a
=
+ m+ m+ m+ m+ m =
m
m
b
a
c
b
a
c
(ab)
X am+1 X bm+1
=
m +
m.
(ab)
(ab)
=

Aplicand inegalitatea lui J. Radon [1]:


m+1

xm+1
xm+1
(x1 + x2 + x3 )
xm+1
2
3
1
+
+

m ,
m
m
m
y1
y2
y3
(y1 + y2 + y3 )
vom obtine

X 1
a

bm

1
cm

xk , yk R+ , k = 1, 2, 3,

m+1

2 (a + b + c)

m+1 .

(ab + bc + ca)

Deoarece (a + b + c) 3 (ab + bc + ca) , obtinem

X 1
a

bm

1
cm

2 (a + b + c)

m+1

2m

(a + b + c)

ceea ce era de demonstrat.


30

3m =

2 3m

m1 ,

(a + b + c)

Demonstratia 2. Corespunde Solutiei 3 din [2]. Se observa ca

X 1
a

bm

1
cm

Xb+c

am

X (b + c)m+1

m,

(a (b + c))

de unde, aplicand inegalitatea lui Radon, deducem ca

X 1
a

bm

1
cm

m+1

2m+1 (a + b + c)
m
2m (ab + bc + ca)

si, cum (a + b + c) 3 (ab + bc + ca) , deducem usor inegalitatea (3).


Demonstratia 3. Corespunde Solutiei 1 din [2]. Aplicand inegalitatea mediilor
obtinem:

X 1
a

bm

Cum a + b + c 3

1
+ m
c

Xb+c

(b + c) (c + a) (a + b)

m
(abc)
s

6
bc 2 ca 2 ab
3 2
=
3
.
m
m1
3
(abc)
(abc)
=

am

abc, rezulta ca pentru ultima fractie avem


6

(abc)

m1

6 3m1

m1 ,

(a + b + c)

m [1, ),

si, drept consecint


a, obtinem inegalitatea (3).
Demonstratia 4. F
ar
a a restrange generalitatea, presupunem ca a b c si
1
1
1
atunci m m m si b + c c + a a + b. Aplicam inegalitatea lui Cebsev si
a
b
c
obtinem

X 1
a

bm

1
cm

Xb+c
am

2
(a + b + c)
3

1
1
1
+ m+ m
m
a
b
c

si, prin aplicarea inegalitatii lui Radon ultimei paranteze, rezulta ca

X 1
a

bm

1
+ m
c

m+1

2
(1 + 1 + 1)
(a + b + c)
m
3
(a + b + c)

2 3m
m1

(a + b + c)

si astfel demonstratia este ncheiat


a.
Bibliograe
1. D.M. B
atinetu-Giurgiu Aplicatii la inegalitatea lui Radon, Gazeta Matematica,
nr. 7-8-9/2010, 359 362.
2. T. Zvonaru C
ateva solutii la problema L. 222 din Recreatii Matematice, Nr.
1/2012, Recreatii Matematice, nr. 2/2012, 120-123.
31

O identitate misterioas
a
Marian TETIVA

Abstract. The author wants to show the reader (the schoolchild, especially) the ways to find an
algebraic identity which hides behind a trigonometric identity, a fact giving the clue for estabilishing
the latter one.
Keywords: identity, trigonometric functions.
MSC 2010: 97D50.

Dac
a observi ca 23 + 33 + (5)3 = 3 2 3 (5) (si nca vreo cateva exemple
aseman
atoare) s-ar putea sa ai o banuiala ca pentru orice numere u, v, w cu suma
u + v + w = 0 are loc egalitatea u3 + v 3 + w3 = 3uvw si probabil ca o sa reusesti sa si
demonstrezi ca asa stau lucrurile (de exemplu nlocuind pe w cu (u+v)). Totusi, zic
eu, asta nu nseamn
a ca ai si nteles de ce lucrurile stau asa. Asta se va ntampla doar
n momentul n care ai ca u3 +v 3 +w3 3uvw = (u+v+w)(u2 +v 2 +w2 uvuwvw)
este o identitate valabil
a oricare ar numerele u, v, w (deci, daca u+v +w = 0, atunci
u3 + v 3 + w3 3uvw = 0). O situatie similara vrem sa discutam si noi n cele ce
urmeaza; anume, este vorba despre
Exercitiul 1. S
a se arate c
a are loc egalitatea (misterioas
a)
cos2 50 + cos 50 cos 70 + cos2 70 =

3
.
4

Aceasta este problema 8 de la pagina 9 din [1] (noi ne vom stradui sa o transformam din problema n exercitiu) care primeste, n lucrarea mentionata, la aceeasi
pagina, doua solutii. Este o problema frumoasa (exprima o identitate n care sunt
implicate doua cosinusuri care nu pot calculate efectiv, oricum nu ntr-un fel care sa
e de folos n acest context) si generoasa cu profesorii, ntrucat diversele ei rezolvari
apeleaza la diferite formule trigonometrice - asadar, reprezinta o buna modalitate de
a exersa aceste formule. Desigur, e nevoie si de cte un mic truc de ecare data, dar
nu unul care sa depaseasc
a nivelul de ntelegere si receptare al elevului mediu.
De exemplu, o prima solutie foloseste formulele cos(a b) = cos a cos b sin a sin b,
pentru a = 60 si b = 10 (trucul nu e deloc articial, ba chiar e foarte natural).
Folosind aceste formule avem
cos2 50 + cos 50 cos 70 + cos2 70 =
= cos2 (a b) + cos(a b) cos(a + b) + cos2 (a + b) =
= 2(cos2 a cos2 b + sin2 a sin2 b) + cos2 a cos2 b sin2 a sin2 b =
3
3
= 3 cos2 a cos2 b + sin2 a sin2 b = (cos2 b + sin2 b) = ,
4
4
1 Profesor,

Colegiul National ,,Gheorghe Rosca Codreanu, B


arlad

32


deoarece cos a = 1/2 si sin a = 3/2 (si, binenteles, folosind formula fundamentala
cos2 t + sin2 t = 1, t R).
A doua solutie din [1] se bazeaza pe transformarea produselor n sume, adica
esentialmente pe formula cos a cos b = (cos(a b) + cos(a + b))/2 (cu varianta ei
cos2 a = (1 + cos 2a)/2). Incercati! (Veti avea nevoie si de transformarea sumei de
cosinusuri n produs, dupa care veti obtine rezultatul ca prin farmec: iata de ce e
frumoasa matematica, e plina de surprize - e si daca ne referim doar la matematica
de liceu).
Nemultumirea, dupa ce ai vazut aceste doua solutii, poate veni nsa exact din
acelasi loc de unde vine si bucuria (asa se ntmpla si n viata, nu?). Anume, avem
doua solutii destul de clare ale aceleiasi probleme, putin articiale, poate, dar, n cele
din urma, frumoase - sau macar placute. Insa nici una dintre ele nu explic
a ntradev
ar aceast
a identitate: de unde provine ea? Cum s-a gndit vreodata cineva ca
suma cos2 50 + cos 50 cos 70 + cos2 70 este egala cu 3/4? Nu e limpede deloc!
Marturisesc ca eu nsumi am trecut multa vreme cu usurinta peste aceasta ntrebare
altminteri perfect justicata. Am rezolvat problema cu elevii poate de cincizeci
de ori, folosind e prima solutie, e pe-a doua, e vreo varianta de-a lor - dar
abia dupa ce a trecut ceva timp mi-am pus aceasta ntrebare simpla: de ce, totusi,
cos2 50 + cos 50 cos 70 + cos2 70 = 3/4? (La chestiunile cele mai simple, cele mai
elementare, ca sa zic asa ne gndim cel mai greu.) Culmea e ca, odata problema pusa,
rezolvarea ei a venit aproape de la sine. E sucient sa te gndesti ca o expresie de
forma u2 + uv + v 2 aproape ca striga dupa formula u3 v 3 = (u v)(u2 + uv + v 2 )
si gasesti imediat urmatoarea rezolvare (pe care eu nu am mai vazut-o pe nicaieri ceea ce nu nseamn
a catusi de putin ca nu exista):
cos2 50 + cos 50 cos 70 + cos2 70 =

3
(cos 50 cos 70 )
4
4 cos3 50 3 cos 50 = 4 cos3 70 3 cos 70 cos 150 = cos 210 .
cos3 50 cos3 70 =

Am mai avut nevoie de formula pentru cosinusul unghiului triplu cos 3a = 4 cos3 a
3 cos a, care poate nu e chiar uzuala, iar la nal de cos(360 t) = cos t. De asemenea, trebuie sa mention
am ca nmultirea cu cos 50 cos 70 conduce la o egalitate
echivalent
a deoarece aceasta diferent
a de cosinusuri este nenula.
Eu cred ca a meritat, pentru ca, daca rescriem putin demonstratia de mai sus,
vedem ca identitatea noastra e doar un caz particular al unei formule mai generale

1
cos 3x cos 3y
cos x + cos x cos y + cos y =
3+
4
cos x cos y
2

valabile ori de cate ori cos x = cos y (iar aceasta nu spune nimic altceva decat ca
u2 + uv + v 2 = (u3 v 3 )/(u v) pentru u = cos x si v = cos y).
Iata cum identitatea ce parea att de misterioasa la nceput a fost demisticata!
Era doar o impostoare! Ori, daca ne mai gndim putin, putem pune problema si asa:
poate ca noi nu ne-am uitat la ea cum trebuie. La urma urmei, ne-a oferit deja destul
33

de mult, si daca ne straduim, putem sa mai obtinem cate ceva de la ea. De exemplu,
acum sunt sigur ca veti putea rezolva urmatoarele probleme (de fapt, exercitii):
Exercitiul 2. S
a se arate c
a
cos2

7
3
+ cos cos
+ cos2
= .
9
9
9
9
4

Exercitiul 3. S
a se arate c
a
sin2

2
2
3
+ sin sin
+ sin2
= .
9
9
9
9
4

S
a se deduc
a identitatea general
a al c
arei caz particular este aceast
a egalitate cu sinusuri. (Mai pare ea misterioasa? Cred ca nu. Nu-i asa ca ,,noua identitate astfel
obtinut
a este, practic, tot cea de mai sus, cu cosinusuri? De ce?).
Pentru a ncheia, sa ne ntoarcem putin la prima solutie: putem oare sa o aplicam
identit
atii generale, adica sa obtinem

1
cos 3x cos 3y
cos x + cos x cos y + cos y =
3+
4
cos x cos y
2

procedand ca acolo? Pentru asta ar trebui sa scriem x = a b, y = a + b (deci a


trebuie sa e (x + y)/2 si b = (y x)/2), si sa calculam, ca la nceput:
cos2 x + cos x cos y + cos2 y = cos2 (a b) + cos(a b) cos(a + b) + cos2 (a + b) =
x+y
yx
x+y
yx
= 3 cos2 a cos2 b + sin2 a sin2 b = 3 cos2
cos2
+ sin2
sin2
=
2
2
2
2
x+y
xy
x+y
xy
= 3 cos2
cos2
+ sin2
sin2
,
2
2
2
2
apoi sa arat
am ca
3 cos2

cos 3x cos 3y
x+y
xy
x+y
xy
1
3+
.
cos2
+ sin2
sin2
=
2
2
2
2
4
cos x cos y

Evident, nu ne aam pe drumul cel (mai) bun, dar am obtinut o alta identitate! Deloc
atrag
atoare si deloc sugestiva, e adevarat, dar daca urmam calea din cea de-a doua
solutie obtinem ceva mai interesant. Lasam, din nou, cititorului placerea de a face
acest lucru:
Exercitiul 4. S
a se arate c
a are loc identitatea
(2 cos(x + y) + 1)(2 cos(x y) + 1) =

cos 3x cos 3y
cos x cos y

pentru orice numere reale x si y care ndeplinesc conditia cos x = cos y.


Exercitiul 5. Ar
atati c
a
cos4

3
3

+ cos3
cos
+ cos2
cos2
+ cos
cos3
+ cos4
=
10
10
10
10
10
10
10
10
34

3
cos2
+ cos
cos
+ cos2
4
10
10
10
10

5
.
16

Evident, si aici exista o identitate generala. Gasiti-o! Nu-i asa ca, folosind-o,
putem face si alte lucruri, de exemplu putem calcula cos(/5)?
In ncheiere propunem cititorului interesat (si dornic sa vada daca a nteles cu
adevarat ce urmareste aceasta nota) sa stabileasca singur ce identitate se ascunde n
spatele misterioasei (deocamdata)

tan 20 cot 50 + tan 80 = 3 3


(tot din [1], problema 17, pag. 12; poate de ajutor sa scrieti tan 140 n loc de
cot 50 ). Desigur, va puteti ncerca puterile cu oricare asemenea identitate, sunt
destule, atat n cartea mentionat
a, cat si n multe alte lucrari mai mult sau mai putin
didactice.
Bibliograe
1. Traian Cohal Probleme de trigonometrie, Editura Moldova, 1994.

R
aspunsuri la ,,recreatiile de la pag. 17:
a)

b)

c)

Sagetile indica modicarile prin care s-au obtinut aceste egalitati.


35

Ireductibilitate n inele de polinoame1


2

Marius TARN
AUCEANU

Abstract. The main aim of this paper consists in the presentation of some irreducibility criteria
for polynomials.
Keywords: irreducible polynomials, criteria for irreducibility.
MSC 2000: 11C08, 11R09.

1. Introducere
Denitia 1. Fie R un domeniu de integritate si a, b R. Spunem ca:
a) a divide b, si notam a|b, daca exista c R astfel ncat b = ac;
b) a si b sunt asociate n divizibilitate, si notam a b, daca a|b si b|a.
Denitia 2. Fie R un domeniu de integritate si U (R) = {a R | a = inversabil}.
Un element p R \ U (R) se numeste:
a) ireductibil daca d R, d|p = d 1 sau d p (n caz contrar spunem ca p
este reductibil );
b) prim daca a, b R, p|ab = p|a sau p|b.
Observatie. Orice element prim din R
este element ireductibil. Reciproca acestei
implicatii este falsa (3 este ireductibil n Z[i 5], dar nu este prim). Cea mai cunoscuta
conditie sucient
a pentru a avea ,,prim ireductibil este ca oricare doua elemente
din R sa admita un c.m.m.d.c. In particular, cele doua concepte coincid n inele
euclidiene, principale sau factoriale.
Caracteriz
ari. Fie R un domeniu de integritate si p R \ U (R). Atunci:
a) p = ireductibil (p) = element maximal n multimea idealelor principale
proprii ale lui R. In particular, daca R este un inel principal, atunci p =
ireductibil (p) = ideal maximal al lui R.
b) p = prim (p) = ideal prim al lui R.
1 Lucrare prezentat
a la simpozionul Matematica n scoal
a: educatie stiintific
a studiu, competitie,
27.10.2012, Zilele Universit
atii ,,Alexandru Ioan Cuza.
2 Lector dr., Universitatea ,,Al.I. Cuza din Ia
si, tarnauc@uaic.ro

36

Observatii. Dac
a K este un corp comutativ, atunci inelul de polinoame K[X]
este euclidian, deci: 1. Orice polinom nenul din K[X] poate scris ca un produs nit
de polinoame ireductibile peste K. 2. Polinoamele ireductibile peste K determina
ideale maximale n K[X]. Acest fapt este extrem de util n constructia corpurilor
nite. Mai precis, daca p, n N cu p = prim, atunci un corp nit cu pn elemente
(unicul pan
a la un izomorsm!) este Fpn = Zp [X]/(f ), unde f Zp [X] este un
polinom ireductibil de grad n.
Avem astfel o puternica motivatie pentru studiul polinoamelor ireductibile si, n
particular, pentru identicarea unor criterii de ireductibilitate.
2. Criterii de ireductibilitate pentru polinoame
Criteriul 1. a) Fie f C[X]. Atunci f este ireductibil dac
a si numai dac
a
deg(f )=1. b) Fie f R[X]. Atunci f este ireductibil dac
a si numai dac
a deg(f )=1
sau deg(f )=2 si f nu are r
ad
acini reale.
Criteriul 2. Fie K un corp comutativ si f K[X]. a) Dac
a deg(f )=1, atunci
f este ireductibil peste K. b) Dac
a deg(f ) {2, 3}, atunci f este ireductibil peste K
f nu are r
ad
acini n K.
Exemplu. f = X 3 + 2X 2 +
1 este ireductibil peste Z3 .
Observatii. 1. Implicatia ,,= de mai sus are loc si pentru deg(f ) 4 (Bezout).
2. Implicatia ,,= de mai sus nu are loc n absenta ipotezei deg(f ) {2, 3}. Spre
exemplu, polinomul f = (X 2 + 1)(X 2 + 2) nu are radacini n R, dar este reductibil
peste R.
Criteriul 3. Fie R un domeniu de integritate si f = f0 +f1 X +...+fn X n R[X]
cu f0 = 0. Numim reciprocul lui f polinomul f = fn + fn1 X + ... + f0 X n R[X].
Atunci f este ireductibil n R[X] f este ireductibil n R[X].
Exemple. 1. f = 6X 9 + 27X 5 + 17 este ireductibil n Z[X].
2. f = pX n 1 este ireductibil n Z[X], unde p = prim si n N .
Observatie. Fie R un domeniu de integritate. Un polinom f = f0 + f1 X +
... + fn X n R[X] se numeste primitiv daca c.m.m.d.c. (f0 , f1 , ..., fn ) 1. Daca R
este inel factorial si n 1, atunci legatura dintre ireductibilitatea lui f peste R si
ireductibilitatea lui f peste corpul de fractii K al lui R este urmatoarea:
f este ireductibil n R[X] f este primitiv si ireductibil n K[X].
Criteriul 4 (Eisenstein). Fie R un inel factorial, K corpul de fractii al lui R,
f = f0 + f1 X + ... + fn X n R[X] cu n 1 si p R un element prim cu propriet
atile:
a) p - fn ; b) p | fi , i = 0, 1, ..., n 1; c) p2 - f0 . Atunci f este ireductibil n K[X].
Dac
a, n plus, f este primitiv, atunci el este ireductibil n R[X].
Exemple. Urm
atoarele polinoame sunt ireductibile n inelele indicate:
1. f = X 5 + 15X 4 + 20X 3 40X + 35 n Z[X];
37

2. f = X n p n Z[X], unde p = prim si n N ;


3. f = X p1 + X p2 + ... + X + 1 n Z[X], unde p = prim;
n
4. f = X p + p 1 n Z[X], unde p = prim;
3
5. f = X Y + X 2 Y 2 + X 3 + Y n Z[X, Y ];
6. f = X 2 + XY + Y 2 X 1 n K[X, Y ], unde K este un corp comutativ de
caracteristica = 2;
7. f = X 6 +X 5 Z +Y Z 3 +Z XY Z n K[X, Y, Z], unde K este un corp comutativ;
8. f = X1p + X2p + ... + Xnp n K[X1 , X2 , ..., Xn ], unde p 3 este prim, n
N \ {0, 1, 2} si K este un corp comutativ de caracteristica diferita de p;
9. f = X n 28 n (Z[i])[X],
unde n N ;
2012
10. f = X
2010 n (Z[i 2])[X].
Criteriul 5 (Perron). Fie polinomul f = f0 +f1 X +...+fn1 X n1 +X n Z[X],
unde n 2 si f0 = 0. Dac
a |fn1 | > 1 + |f0 | + |f1 |... + |fn2 |, atunci f este ireductibil
n Z[X].
Exemple. Urmatoarele polinoame sunt ireductibile n Z[X]: 1. f = X 10 +
2011X 9 + 2008X 4 + 1; 2. f = X n + 5X n1 + 3, unde n 2; 3. f = X n + (a +
3)X n1 + X + a, unde n 2 si a N .
Criteriul 6. Fie R1 , R2 dou
a domenii de integritate, K1 , K2 corpurile lor de
fractii si : R1 R2 un morfism de inele. Presupunem c
a R1 este inel factorial
si not
am cu : R1 [X] R2 [X] morfismul de inele definit prin (X) = X si
|R1 = . Dac
a f R1 [X] satisface: a) deg((f )) = deg(f ); b) (f ) este ireductibil
n K2 [X], atunci f este ireductibil n K1 [X]. Dac
a, n plus, f este primitiv, atunci
el este ireductibil n R1 [X].
Caz particular. Fie p = prim, f = f0 + f1 X + ... + fn X n Z[X] si f =

f0 + f1 X + ... + fn X n Zp [X] redusul lui f modulo p. Daca: a) deg(f) = deg(f );


b) f este ireductibil n Zp [X], atunci f este ireductibil n Q[X]. Daca, n plus, f este
primitiv, atunci el este ireductibil n Z[X].
Exemple. Urmatoarele polinoame sunt ireductibile n Z[X]:
1. f = 17X 7 + 4X 5 X 3 8X + 1;
2. f = X p X + a, unde p = prim, a Z si p - a.
Criteriul 7 (Sch
onemann). Fie polinomul f Z[X] ce are coeficientul dominant 1 si poate fi scris sub forma f = g n + ph, unde n N si p este prim, iar
Atunci f
polinoamele g, h Z[X] satisfac: a) g este ireductibil n Zp [X]; b) g - h.
este ireductibil n Z[X].
Exemple. Urmatoarele polinoame sunt ireductibile n Z[X]:
1. f = (X 2 + 2)n + 5(X 2n1 + 10X n + 5), unde n N ;
2. f = (X 2 + 1)n + p, unde n N si p = prim, p 3 (mod 4).
Criteriul 8 (Cohn). Dac
a un num
ar prim p se exprim
a n baza 10 sub forma
p = a0 + a1 101 + ... + an 10n , 0 ai 9, i = 0, 1, ..., n, atunci polinomul f =
a0 + a1 X 1 + ... + an X n este ireductibil n Z[X].
38

Exemplu. f = 3 + 7X + 9X 2 + 9X 3 este ireductibil n Z[X] (9973 = prim).


Generalizare. Fie b 2 un num
ar natural si polinomul f = a0 +a1 X 1 +...+an X n
astfel nc
at 0 ai b 1, i = 0, 1, ..., n. Dac
a f (b) este un num
ar prim, atunci
polinomul f este ireductibil n Z[X].
Observatie. O reciproca a criteriului lui Cohn este urmatoarea: Dac
a f Z[X]
este un polinom primitiv ireductibil, atunci exist
a o baz
a b astfel nc
at coeficientii lui
f formeaz
a reprezentarea unui num
ar prim n baza b. Aceasta constituie o celebra
problema deschis
a, cunoscuta sub titulatura de Conjectura lui Buniakovsky.
P
olya si Szeg
o au dedus un criteriu de ireductibilitate pentru polinoame cu
coecienti ntregi din studiul marginilor radacinilor lor.
Criteriul 9. Dac
a f este un polinom cu coeficienti ntregi si exist
a m N, m 2,
1
astfel nc
at: a) f (m 1) = 0; b) f (m) este num
ar prim; c) Re(z) < m pentru
2
orice r
ad
acin
a z a lui f , atunci polinomul f este ireductibil n Z[X].
Exemplu. f = X 4 X 2 + 1 este ireductibil n Z[X] (m = 2 satisface conditiile).
Tot lui P
olya i datoram si urmatorul criteriu:
Criteriul 10. Dac
a f Z[X] este un polinom de grad n astfel nc
at exist
a
x1 , x2 , ..., xn Z distincte cu 0 < |f (xi )| < 2N N !, i = 1, 2, ..., n,unde N = [n/2],
atunci f este ireductibil peste Q.
Inspirate de rezultatele de mai sus, au fost stabilite mai multe criterii de ireductibilitate pentru polinoame cu coecienti ntregi ce au un coecient sucient de
mare sau iau o valuare prima. Amintim aici doua din ele, probate de matematicienii
romani A.I. Bonciocat si N.C. Bonciocat n 2009:
Criteriul 11. Dac
a un num
ar prim se exprim
a ca o sum
a de numere ntregi
a0 , a1 , ..., an cu a0 an = 0 si |a0 | >
an X n este ireductibil peste Q.

n
X

|ai |2i , atunci polinomul f = a0 + a1 X 1 + ... +

i=1

Criteriul 12. Dac


a toti coeficientii unui polinom f sunt 1 si exist
a m N,
m 3, astfel nc
at f (m) este num
ar prim, atunci polinomul f este ireductibil peste Q.
In nal, mention
am ca pentru toate criteriile de mai sus sunt cunoscute numeroase
generaliz
ari.
3. Clase remarcabile de polinoame ireductibile
3.1 Polinoame ciclotomice. Fie K un corp, K nchiderea algebrica a lui K, n
N si Pn (K) multimea rad
acinilor primitive de grad n ale unitatii din K. Polinomul
n =

(X ) K[X]

Pn (K)

39

se numeste al n-lea polinom ciclotomic peste K.


Propriet
ati.
1. n K[X]. Mai precis, toti coecientii lui n apartin corpului prim al lui K.
In particular, daca char(K) = 0, atunci n Z[X], iar daca char(K) = p > 0, atunci
n Zp [X].
2. deg(n ) =Y
(n).
n
3. X 1 =
d .
4. n =

Yd|n n

(X d 1)(d) , unde : N Z este functia lui Mobius. Cazuri

d|n

particulare: a) 1 = X 1; b) 4 = X 2 + 1; c) 6 = X 2 X + 1; d) p =
X p1 + X p2 + ... + X + 1 pentru p = prim.
5. Daca K = C, atunci n este ireductibil peste Z (deci este polinomul minimal
peste Q al oricarei radacini primitive de grad n a unitatii complexe).
6. Daca K = Zp , atunci n este ireductibil peste Zp daca si numai daca (n)
coincide cu gaussianul lui p modulo n, adica cu ordinul elementului p n grupul multiplicativ (U (Zn ), ). In plus, n acest caz d este ireductibil peste Zp pentru toti
divizorii d ai lui n.
O problema formulat
a de catre I. Schur ([12]) n 1908 vizeaza ireductibilitatea
polinoamelor de tipul P = (X a1 )(X a2 ) (X an ) 1, unde a1 , a2 , ..., an Q
sunt distincte. Aceasta a fost solutionata un an mai tarziu de catre W. Fl
ugel ([6])
si J. Westlund ([16]), care au aratat urmatorul rezultat:
Teorem
a. Polinimul P este ireductibil peste Z, iar polinomul P+ este ireductibil
peste Z dac
a si numai dac
a nu exist
a a Z astfel nc
at
P+ (X a) = (X 1)2 sau P+ (X a) = (X 2 3X + 1)2 .
In acest paragraf prezentam un rezultat ce furnizeaza o modalitate simpla de
constructie a unei clase innite de polinoame ireductibile peste Z, plecand de la un
polinom arbitrar.
Teorem
a. Fie polinomul nenul f Z[X] si k N fixat. Atunci exist
a N1 N
astfel nc
at polinomul f kp este ireductibil, oricare ar fi p prim, p N1 .
In plus,
dac
a f are o r
ad
acin
a simpl
a n Q, atunci exist
a N2 N astfel nc
at polinomul
f kpm este ireductibil, oricare ar fi p prim si m N cu pm N2 .
O conjectura interesant
a privitoare la o clasa de polinoame ireductibile peste Q a
fost enuntat
a de catre M. Filaseta n 1986:
Conjectura A. Fie n 2 un num
ar natural si polinomul f = 1+X +X 2 +...+X n .

Atunci f este ireductibil peste Q.


Ulterior, n 1991, aceasta a fost generalizata de catre T.Y. Lam:
40

Conjectura B. Fie n, k N satisf


ac
and n 2 si 1 k n 1, si polinomul
f = 1 + X + X 2 + ... + X n . Atunci f (k) este ireductibil peste Q.
Utilizand calculatorul, ambele au fost probate pentru valori sucient de mari ale
lui n.
Bibliograe
1. I. B
aetu Algebr
a finit
a pentru Centrele de Excelent
a, Editura Taida, Iasi, 2010.
2. M. Becheanu, A. Dinc
a, I.D. Ion, C. Nit
a, I. Purdea, N. Radu, M. S
tef
anescu, C. Vraciu Algebr
a pentru perfectionarea profesorilor, Editura Didactica si
Pedagogic
a, Bucuresti, 1981.
3. A.I. Bonciocat, N.C. Bonciocat The irreducibility of polynomials that have one
large coeficient and take a prime value, Canad. Math. Bull. 52 (2009), 511-520.
4. A. Borisov, M. Filaseta, T.Y. Lam, O. Trifonov Classes of polynomials
having only one non-cyclotomic irreducible factor, Acta Arith. XC (1999), 121-153.
5. J. Brillhart, M. Filaseta, A. Odlyzko On an irreducibility theorem of A. Cohn,
Canad. J. Math. 33 (1981), 1055-1059.
6. W. Fl
ugel Solution to problem 226, Archiv. Math. Physik 15 (1909), 271.
7. I.D. Ion, N. Radu Algebr
a, Editura Didactica si Pedagogica, Bucuresti, 1991.
8. I.D. Ion, N. Radu, C. Nit
a, D. Popescu Probleme de algebr
a, Editura
Didactica si Pedagogic
a, Bucuresti, 1981.
9. C. N
ast
asescu, C. Nit
a Teoria calitativ
a a ecuatiilor algebrice, Editura Tehnica,
Bucuresti, 1979.
10. G. P
olya Verschiedene Bemerkungen zur Zahlentheorie, Jber. Deutsch. Math.Verein. 28 (1919), 31-40.
11. G. P
olya, G. Szeg
o Aufgaben und Lehrs
atze aus der Analysis, Springer Verlag,
Berlin, 1964.
12. I. Schur Problem 226, Archiv. Math. Physik 13 (1908), 367.
13. M. Sepanski Algebra, American Mathematical Society, Providence, Rhode Island,
2010.
14. M. T
arn
auceanu Probleme de algebr
a, vol. II., Editura Universitatii Al. I.
Cuza, Iasi, 2004.
15. I. Tofan, A.C. Volf Algebr
a: Inele. Module. Teorie Galois, Editura Matrix
Rom, Bucuresti, 2001.
16. J. Westlund On the irreducibility of certain polynomials, Amer. Math. Monthly
16 (1909), 6667.
17. Gazeta Matematic
a, 1990-2012.

41

Colegiul National ,,Mihail Sadoveanu din Pascani


Am stat cu mine nsumi, trudindu-m
a s
a dezleg ntreb
ari f
ar
a r
aspuns.
E nevoie pentru asta de t
acere si singur
atate.
Desf
ac
andu-m
a de tot ce at
arn
a si ntunec
a, m-am dus s
a v
ad numai cu duhul. . .
Mihail Sadoveanu

1.
Inceputurile nv
at
am
antului p
asc
anean. La 1 ianuarie 1851, domnitorul Grigore Alexandru Ghica a sanctionat Asez
am
antul pentru reorganizarea
nv
at
aturilor publice n Principatul Moldovei, document revolutionar prin ideile promovate, de referint
a pentru proiecte ulterioare privind organizarea nvatamantului
n Moldova. Documentul prevedea crearea unui nvatamant public gratuit n limba
national
a pentru toti locuitorii tarii, dezvoltarea nvatamantului de toate gradele,
nintarea unor scoli pentru fete, nintarea de scoli primare la sate s.a.
Pentru pregatirea nv
atatorilor s-a nintat la 15 decembrie 1855, n ncaperile
man
astirii Trei-Ierarhi Institutul Preparandal, care mai tarziu si va schimba numele n
S
coala Normal
a de la Trei-Ierarhi si apoi n S
coala Normal
a ,,Vasile Lupu. Imediat
apar si primele scoli satesti n Moldova, nintate de stat. Unirea Principatelor din
1859 a impulsionat dezvoltarea nvatamantului public la sate. In cadrul reformelor
din timpul domniei lui Al. I. Cuza a fost elaborata la 25.11.1864 Legea organic
a
pentru instructiunea public
a n Principatele Unite, prima lege pentru nvatamantul
public conceputa pe baza de principii democratice. Legea prevedea lucruri care
sunt actuale si astazi: unitatea scolii primare de sat si oras prin: uniformizarea
programelor de studii, egalitatea sexelor la nv
ata
tur
a, preg
atirea nv
at
atorilor, ocuparea posturilor vacante prin concurs, inspectarea periodic
a a scolilor de c
atre revizorii
42

judeteni, ntocmirea de recens


aminte anuale a copiilor de v
arst
a scolar
a. In anul 1864,
n Moldova au fost nintate 253 scoli n care frecventau cursurile 61 977 de elevi.
Prin eforturile revizorului scolar N. Vicol, la 2 decembrie 1864 s-a nintat S
coala
de b
aieti Pascani, care a devenit scoala urbana ,,model. In primavara anului 1868,
la Pascani functiona S
coala de fete, care a fost desintata (temporar) n 1874 si
renintat
a n 1879. Constructia cailor ferate Roman-Burdujeni (1869), PascaniIasi (1870) precum si a Atelierului CFR din Pascani (1869) a determinat Societatea
concesionar
a Oenhein sa aduca muncitori, maistri si conducatori tehnici, care ,,au
consimtit sa ram
an
a n localitate cu conditia de a li se face biserica si scoli.
2. Colegiul din Pascani etape ale devenirii sale. Nucleul actualului colegiu
se regaseste n Gimnaziul Mixt nintat n 1918, o forma de scolarizare superioara
n localitate. Criza economica declansata n 1929 a dus la desintarea acestuia. S-a
constituit o delgatie pasc
anean
a care a intervenit la guvernul G. Mironescu din acea
perioada pentru renintarea unui gimnaziu n localitate. Acest lucru se realizeaza n
1932, cand este creat Gimnaziul de b
aieti CFR, care a functionat ntre 1932-1940
ntr-o casa particulara, cumparat
a de CFR. Cladirea avea la parter patru sali de clasa,
iar la etaj existau doua nc
aperi care erau folosite ca biblioteca si cancelarie. In curtea
scolii era amenajat un teren de sport si o sala de gimnastica.
In primavara anului 1944, n conditiile apropierii frontului, gimnaziul, la fel ca
si celelalte institutii din Pascani, a fost nevoit sa se refugieze n sud-estul tarii.
Rentoarcerea provizorie a ceferistilor pascaneni n localitatile Marasesti, Panciu si
Adjud a facut ca si Gimnaziul Teoretic de Baieti CFR din Pascani sa revina n
Moldova. Aici a functionat pan
a n 1945, cand pascanenii s-au rentors n orasul
natal. Motivat de faptul ca tinerii pascaneni erau nevoiti sa frecventeze liceele din
Iasi, Suceava, Roman, Falticeni n conditiile materiale precare de atunci, la propunerea directorului Gimnaziului de Baieti CFR, I. Mitroiu, si cu sprijinul parintilor si
conducerii Atelierelor CFR, gimnaziile unice de baieti si fete din Pascani s-au transformat n anul 1946 n liceu, Liceul Teoretic de B
aieti CFR. De fapt, liceul era
mixt, nc
at din 1947 se va numi pentru scurt timp Liceul Mixt CFR.
In urma reformei nv
at
am
antului din 3 august 1948, devine S
coala Medie Mixt
a,
ind sub tutela CFR-ului si sub administratia Ministerului Invatamantului. In perioada urmatoare, liceul a avut succesiv denumirile: S
coala Medie ,,C. Burc
a
(1956), S
coala Medie de Cultur
a General
a (1964), Liceul de Cultur
a General
a (1965), Liceul Real-Umanist (1975), Liceul Industrial ,,C. Burc
a (1977).
Au suferit schimb
ari, de asemenea, durata scolaritatii si prolul liceului, iar din anul
1964 liceul are un nou local, unde functioneaza si n prezent.
In 1990, aceasta unitate de nv
at
amant si-a schimbat denumirea n Liceul Teoretic ,,Mihail Sadoveanu, pan
a la data de 1 septembrie 2008, cand, din ordinul
ministerului, a devenit Colegiul National ,,Mihail Sadoveanu.
3. Realiz
ari si contributii importante. Colegiul din Pascani, datorita rezultatelor obtinute de-a lungul timpului, a devenit un simbol al orasului si o mandrie a
locuitorilor sai. El poarta numele marelui povestitor roman Mihail Sadoveanu, pe
care, ca un semn de recunostint
a, l-a adoptat ca patron spiritual. Mihail Sadoveanu
s-a nascut la 5 noiembrie 1880 n Pascani si urmeaza cursurile scolii primare (nvatator
M. Busuioc) n acest oras. Schita de debut, Domnisoara M. din F
alticeni (1897), apare
43

sub semnatura Mihai din Pascani, consntind astfel legatura cu orasul natal.
Dintre personalitatile proeminente ale culturii romanesti, care au fost dascali la un
moment dat ai acestei institutii scolare, amintim: acad. Constantin Ciopraga - profesor de franceza si director, sculptorul Ion Irimescu - profesor de desen, prof.univ.dr.
Nicolae Irimiciuc - profesor de zica s.a.
P
an
a n 2012 pe bancile colegiului s-au format un numar de aproximativ 9000 de
absolventi, care si-au continuat studiile si au devenit profesori universitari, ingineri,
economisti, medici, profesori n nvatamantul preuniversitar, preoti, oteri etc.
Numerosi dintre ei s-au remarcat n diferite domenii de activitate: profesori universitari : Corneliu Zolyneac - stiinte biologice, fost decan al Fac. de Biologie din Iasi,
Aurel Ciopraga - jurist, fost decan al Fac. de Stiinte Juridice din Iasi, Constantin
P
apusoi - zica, Mihail Voicu - m.c. al Acad. Romane, fost prorector al Univ. Tehnice
,,Gh. Asachi din Iasi, Paraschiv Monica (Voicu) - chimie, Constantin F
alticeanu la Univ. ,,Dunarea de Jos din Galati, Leonard Gavriliu - psihologie, Costic
a Cazacu
- matematic
a etc.; artisti lirici: Mihaela Agapi - prim solista la Opera Romana (Bucuresti), Constantin Florescu, Ioan Chiril
a, Alexandru Hasmas ; scriitori : Corneliu
Sturzu - fost director al Teatrului National din Iasi si alte domenii.
Multi altii ocupa pozitii importante n societatea romaneasca sau n strainatate:
Narciza-Eugenia Irimescu - medic, cercetator stiintic, Kalinka Rudolf Franz - ing.
electronist, latelist recunoscut pe plan international, Lacint T
ornghebel, Gheorghe
Voinea, Eugen Nechifor Moraru, Ion Sabin-Istrati etc.
Vom face cateva referiri doar n privinta studiului matematicii, o parte a bogatei
activitati a Colegiului. De-a lungul timpului, Colegiul a avut un corp de profesori de
matematica de elita, iar elevii sai au obtinut rezultate remarcabile atat la examenele
de admitere n universit
ati, cat si la concursurile de matematica interjudetene sau
nationale. Mention
am performanta profesorului Dumitru Gherman care a participat cu elevi de 30 de ori la faza nationala a olimpiadei de matematica obtinand
numeroase premii si mentiuni. Actualii titulari ai catedrei de matematica: Macovei Liviu, Cr
aciun Dorinel-Mihai, Popescu Claudia, Plescan Vasile, Pricop Vasile,
ndrum
a elevii sa devina colaboratori si rezolvitori la reviste de matematica ca: Gazeta
Matematic
a, Revista Matematic
a din Timisoara, Recreatii Matematice si Arhimede.
Astfel, elevul Vntur Cristian, la faza nationala a olimpiadei, a obtinut n 2009
(Slatina) medalie de bronz, iar n 2012 (Constanta) medalie de argint la clasa a VIII-a.
Profesorii participa n mod frecvent la conferinte, simpozioane, tabere de matematica si sunt autori ai unor culegeri de probleme, manuale scolare, articole, reviste.
Din 2011 la Pascani functioneaza un centru de excelenta (liala a Iasului). Dintre proiectele proprii amintim: Concursul Mathematiques sans fronti`eres (coord. L.
Macovei) si Concursul Interjudetean ,,Sperante Olimpice (coord. M. Craciun).
O scoal
a este o carte deschisa, carte ce naste mereu le cu ecare an, cu ecare
generatie, mereu spre viitor. O scoala este o parte din viata tuturor, mici si mari,
copii si tineri, parinti si bunici.

Carmen DIMITRIU -director al Colegiului

Mihai CRACIUN
-profesor de matematic
a
44

Concursul ,,Recreatii Matematice


Editia a X-a, Muncel, 24 august 2012

Clasa a IV-a
1. Aati suma numerelor naturale a si b stiind ca a reprezinta descazutul dintr-o
operatie de scadere n care suma dintre descazut, scazator si diferenta este 1502, iar
b reprezint
a cel mai mare rest al mpartirii unui numar natural la 511.
2. Pe tabla sunt scrise numerele: 1, 2, 3, 6, 7, 9, 10, 23, 48. Dan si Ana au sters
ecare cate 4 numere si au observat ca suma numerelor sterse de Dan este de patru
ori mai mare decat suma numerelor sterse de Ana.
a) Ce num
ar a ramas pe tabla?
b) Ce numere a sters ecare copil?
3. Sa se calculeze 29 30 + 31 32 + 33 34 + . . . + 149 150 + 151.
4. Un fermier are la dizpozitie o sfoara lunga de 540 m pentru a delimita o
suprafata de teren. El are de ales ntre a delimita un dreptunghi cu lungimea de doua
ori mai mare decat latimea sau un patrat. Cum va delimita fermierul suprafata de
teren, pentru ca aceasta sa e cat mai mare?
(Propun
ator, prof. Doina Nechifor)

Clasa a V-a
1. Fie A, B, C, D patru puncte coliniare (situate pe aceeasi dreapta) n aceasta
ordine, cu AB + AD = 2AC si BD = 231 (cm). Sa se ae lungimea segmentului BC.
2. Se considera num
arul a = 7 + 72 + 73 + . . . + 72009 .
a) Demonstrati ca num
arul a nu este patrat perfect.
b) Aati restul mp
artirii lui a la 400.
(Recreatii Matematice, 1/2009)
3. Ionel, Gigel si Vasilic
a au un cos cu mere. Ionel a mpartit merele din cos n
trei parti egale si a luat una din parti. Dupa aceasta, Gigel a mpartit merele ramase
n patru parti egale si a luat una din parti. In nal, Vasilica a mpartit merele ramase
n cinci parti egale, a luat una din parti si a constatat ca n cos au mai ramas 36 de
mere.
a) Aati cate mere au fost la nceput n cos.
b) Cate mere a luat ecare copil?

Clasa a VI-a

1. In triunghiul ABC avem AC < BC si m(]ACB) = 60 . Pe latura (BC) se


considera punctul D astfel nc
at (BD) (AC) si e punctul E simetricul punctului
A fata de punctul C. Sa se demonstreze ca [DE] [AB].
45

2. Se considera triunghiul isoscel ABC cu AB = AC si m(]BAC) = 30 . Punctul


D se aa n interiorul unghiului ]ACB, astfel ncat m(]DCB) = 15 si m(]DBA) =
75 . Demonstrati ca are loc egalitatea AC = AD.
(Recreatii Matematice, 2/2012)
3. Aati numerele naturale de patru cifre n baza 10 care au exact 64 de divizori.

Clasele a VII-a si a VIII-a


1. Intr-un plan consideram cercurile C(O1 ; r1 ), C(O2 ; r2 ) si C(O3 ; r3 ), cu razele
r1 = r2 = r3 = r1 . Tangentele comune exterioare cercurilor C(O1 ; r1 ) si C(O2 ; r2 ) se
intersecteaz
a n M , cele ale cercurilor C(O1 ; r1 ) si C(O3 ; r3 ) se intersecteaza n N iar
cele ale cercurilor C(O2 ; r2 ) si C(O3 ; r3 ) se intersecteaza n P . Aratati ca punctele
M, N si P sunt coliniare.
2. Ar
atati ca oricare ar n N , exista m N astfel ncat n4 m + 1 sa e
num
ar natural compus.
(Recreatii Matematice, 1/2009)
3. Fie triunghiul ABC si punctele M, N, P astfel ncat M (AB), N (AC),
MQ
BP AC AM
P (BC) si AP M N = {Q}. Sa se arate ca
=

.
NQ
CP AB AN

Clasele a IX-a si a X-a


1. Pe un cerc x C(O; R) se iau punctele mobile A, B, C. Stiind ca suma razelor
cercurilor exnscrise triunghiului ABC este constanta, sa se determine locul geometric
al centrului cercului nscris n triunghiul ABC.

2. Fie functia f : [1, +) R, f (x) =

|x 2|, x [1, ]
, 0 ind
x 1, x (, +)

un parametru real.
a) Rezolvati ecuatia f 13 (x) = f3 (x).
b) Aratati ca, pentru orice 0, compunerea f f este posibila.
c) Rezolvati ecuatia f2 (f0 (x)) = f1 (x).
3. Fie P un punct pe mediana din A a triunghiului ABC. Paralela prin P la
AC taie AB n M , iar simetricul lui P fata de mijlocul lui AC este N . Aratati ca
M N BC daca si numai daca P este centrul de greutate al triunghiului ABC.

Vizitati pagina web a revistei Recreatii Matematice:

http://www.recreatiimatematice.ro
46

Concursul interjudetean ,,Sperante Olimpice


Editia a XII-a, Pascani, 3 noiembrie 2012

Clasa a III-a
1. a) Aati suma a + b + c + d stiind ca a, b, c, d sunt numere pare consecutive, iar
a + c = 96.
b) Ordonati crescator numerele a 2, 2a 3, 3a 2, 2a 1, stiind ca a este mai
mare sau egal cu 2.
2. Se considera sirul de numere naturale 42, 72, 102, 132, . . . .
a) Scrieti urmatorii trei termeni ai sirului.
b)Aati al 100-lea termen al sirului.
3. a) In cate moduri putem forma un sir indian compus din 4 baieti si 3 fete astfel
ncat doua fete sa nu stea una lang
a alta, iar sirul sa nu nceapa cu o fata?
b) Maria a cules nuci. Vaz
and ca poate forma un numar exact de grupe de 12
nuci, ea da cate 4 nuci din ecare grupa prietenei sale si constata ca diferenta dintre
numarul nucilor care i-au ramas si al celor date este 36. Cate nuci a cules Maria?

Clasa a IV-a
1. a) Calculeaza 175 {[(40 10 100) : 100 + 18] 3 12} : 3.
b) Sa se calculeze a + b + c, stiind ca: a b = 60, a c = 90 si b + c = 10.
2. a) Aati valoarea lui a pentru care este adevarata egalitatea: (aaa + aa) :
a a = 119.
b) Fie trei numere naturale a caror suma este 170. Daca marim dublul celui de-al
doilea cu 4, obtinem cincimea primului numar. Sa se determine numerele, stiind ca
suma ultimelor doua este mai mare decat primul cu 10.
3. a) Suma a sapte numere naturale este 2012. Se poate termina produsul lor n
2013? Justicati.
b) Demonstrati ca din 61 de numere naturale distincte a caror suma nu depaseste
2012 putem alege doua a caror suma este 61.

Clasa a V-a
1. Se considera num
arul A = 3a + a3.
a) Determinati cifra a pentru care A este patrat perfect.
b) Aratati ca nu exista valori ale lui a astfel ncat A sa e cub perfect.
c) Determinati a pentru care restul mpartirii lui A la 9 sa e egal cu 3.
2. a) Aratati ca exista numere naturale a, b, c astfel ncat 62009 = a3 + b3 + c3 .
b) Sa se determine ultimele trei cifre ale numarului a = 34n+4 + 2 34n+2 + 34n .
3. Scriem, pe rand, 2013 numere naturale, nenule, distincte, cu proprietatea ca
suma oricaror doua numere vecine este numar par.
a) Sa se demonstreze ca cea mai mica suma posibila a celor 2013 numere este
patrat perfect.
47

b) Sa se arate ca, oricum am alege sapte dintre aceste numere, exista cel putin
doua a caror diferent
a se mparte exact la 12.

Clasa a VI-a
1. a) Aratati ca n(n + 3) + 2 = (n + 1)(n + 2), oricare ar n N.
(2 5 + 2)(4 7 + 2)(6 9 + 2) ... (2012 2015 + 2)
b) Demonstrati ca num
arul A =
(3 6 + 2)(5 8 + 2)(7 10 + 2) ... (2011 2014 + 2)
este natural.
2. a) Aati numerele naturale x si y stiind ca suma lor este 250, iar catul mpartirii
(cu rest) a lui x la y este 7.
b) Sa se ae restul mp
artirii numarului S = 11 +22 +33 +44 +55 +66 +. . .+20092009
la 4.
3. a) Determinati numerele naturale x si y stiind ca x2 + x + y = 58 si y este
num
ar prim.
b) Scrieti num
arul 2010 ca o suma a unor numere naturale si ca produs al acelorasi
numere naturale.

Clasa a VII-a
1. Pe un cerc sunt 11 numere naturale astfel ncat suma oricaror trei numere
alaturate este cel mult 19, iar suma oricaror patru numere alaturate este cel putin 25.
Sa se determine suma celor 11 numere.
2. Fie a, b, c R ; aratati ca:
 2
2
a
a4
a)
b = 2 2a2 + b2 ;
b
b
a4
b4
c4
b) 2 + 2 + 2 a2 + b2 + c2 ab + bc + ca.
b
c
a
3. Se da patratul ABCD cu latura 7 si M, N, P, Q pe laturile [AB], [BC], [CD],
respectiv [DA], astfel nc
at AM = BN = CP = DQ = 3.
Demonstrati ca M N P Q este patrat si determinati latura acestuia.

Clasa a VIII-a

1
n
1
1
=

, n N .
1 + n2 + n4
2 (n 1)n + 1 n(n + 1) + 1
b) Calculati suma
1. a) Aratati ca

1
2
3
2012
+
+
+ ... +
.
1 + 12 + 14
1 + 22 + 2 4
1 + 32 + 3 4
1 + 20122 + 20124

2. a) Aati numerele reale x, y, z stiind ca 3(x + y + 1) ( x 1 + y 2 + 2)2 .


2
2012
2012
b) Aratati ca numerele a = (2 + 1)(22 + 1)(22 + 1) ... (22
+ 1) si b = 42
sunt consecutive.

= 20 si [AB] [AC]. Fie M (AC) astfel


3. Triunghiul ABC are m BAC
S=

) = 10 . Demonstrati ca [AM ] [BC].


nc
at m(ABM
(Gazeta Matematic
a, nr. 9/2012 )
48

Solutiile problemelor propuse n nr. 2/2012


Clasele primare
P.240. Alina are 14 baloane rosii si verzi. Baloane verzi are cel mult 5. C
ate
baloane rosii poate avea Alina?
(Clasa I)
Inst. Maria Racu, Iasi
Solutie. Dac
a num
arul baloanelor verzi este cel mult 5, atunci numarul baloanelor
rosii este cel putin 9. Numarul baloanelor rosii poate lua valori de la 9 la 13.
P.241. Completati casetele cu semnele + sau astfel nc
at scrierea
1234 = 122134 s
a fie corect
a. Dati cel putin dou
a solutii.
(Clasa I )
Paula Balan, elev
a, Iasi
Solutie. Putem avea, de exemplu: 1 + 2 + 3 4 = 12 2 1 3 4 sau
1 + 2 + 3 + 4 = 12 2 + 1 + 3 4.
P.242. C
ate puncte se afl
a n interiorul p
atratului si cercului, dar nu si n interiorul dreptunghiului?
(Clasa I )
Ionut Airinei, elev, Iasi
Solutie. In exteriorul dreptunghiului sunt 5 puncte. Dintre acestea, doua sunt n exteriorul cercului si unul n exteriorul patratului.
In concluzie, numai doua puncte ndeplinesc conditia problemei.
P.243. Un elev realizeaz
a urm
atoarea structur
a:  . . ., n total
44 forme geometrice. Exist
a o portiune a structurii n care s
a se g
aseasc
a exact trei
triunghiuri, iar num
arul dreptunghiurilor s
a fie 20?
(Clasa a II-a)
Mariana Nastasia, elev
a, Iasi
Solutie. R
aspunsul este armativ: consideram cele 23 de guri de pe pozitiile
16, 17, . . . , 38. Primele cinci sunt dreptunghiuri; urmeaza un triunghi. Apoi, urmatoarele sase sunt dreptunghiuri si urmeaza un nou triunghi. Avem nca sapte dreptunghiuri, un triunghi si, n sfarsit, doua dreptunghiuri. In total, secventa contine trei
triunghiuri si 5 + 6 + 7 + 2 = 20 dreptunghiuri.
P.244. Avem dou
a vase, unul de 5 litri si cel
alalt de 8 litri. Cum m
asur
am 4 litri
de ap
a?
(Clasa a II-a)
Codruta Filip, elev
a, Iasi
Solutie. Umplem vasul de 5l, apoi l turnam n cel de 8l. Umplem din nou vasul
de 5l si completam vasul de 8l, obtinand 2l n vasul de 5l. Golim vasul de 8l, apoi
turnam n el cei 2l. Umplem din nou vasul de 5l si l turnam peste cei 2l, obtinand 7l.
Umplem din nou vasul de 5l si completam vasul de 8l cu 1l. La aceasta operatiune,
n vasul de 5l ram
an 4l.
P.245. Dac
a a este cel mai mare num
ar natural par mai mic dec
at 902, iar b este
cel mai mare num
ar de trei cifre diferite cu suma 10, s
a se arate c
a
(b a) + (b a) + . . . + (b a) = a (suma are 90 termeni).
(Clasa a II-a)
Lidia Balica, elev
a, Iasi

... .. . ..
.

49

Solutie. Avem a = 900, b = 910, iar b a = 10. Deoarece 10 zeci formeaza


num
arul 100, atunci 90 de zeci formeaza numarul 900.
P.246. Cele dou
a verisoare, Oana si Camelia, au primit mere de la bunica lor.
Oana spune:
D
a-mi 2 mere, pentru a avea c
at tine!
Camelia r
aspunde:
D
a-mi tu 2 mere, pentru ca merele r
amase tie s
a reprezinte jum
atate din c
at voi
avea eu!
C
ate mere a primit fiecare?
(Clasa a III-a)
Inst. Maria Racu, Iasi
Solutie. Camelia are cu 4 mere mai mult decat Oana. Dupa ce Oana da cele
doua mere Cameliei, ea ramane cu jumatate din numarul merelor Cameliei, adica
2 + 4 + 2 = 8. Rezulta ca Oana are 8 + 2 = 10 mere, iar Camelia 10 + 4 = 14 mere.

P.247. Intr-o
cutie sunt bile. Tripl
am num
arul bilelor si scoatem din cutie 17
bile, apoi tripl
am num
arul bilelor r
amase si iar scoatem 17 bile s.a.m.d. Putem goli
cutia prin repetarea acestei operatii?
(Clasa a III-a)
Iulia Sticea, elev
a, Iasi
Solutie. In cutie ram
an de ecare data 3 a 17 bile. Acest numar nu poate
0, deoarece 17 nu se mparte exact la 3.
P.248. O elev
a a cules mere. V
az
and c
a poate forma un num
ar exact de grupe de
10 mere, eleva d
a c
ate trei mere din fiecare grup
a unui c
amin de b
atr
ani si constat
a
c
a diferenta dintre num
arul merelor care i-au r
amas si cel al merelor date este 36.
C
ate mere a cules eleva?
(Clasa a III-a)
Ioana Gr
aunte, elev
a, Iasi
Solutie. Not
am cu a numarul grupelor de cate 10 mere. Eleva doneaza 3a mere.
Trebuie sa avem 7a 3a = 36, de unde a = 9. Eleva a cules 10 9 = 90 mere.
P.249. Ar
atati c
a num
arul 343 343 343 . . . 3437 (2012 grupe de 343) se mparte
exact la 7.
(Clasa a III-a)
Tatiana Ignat, elev
a, Iasi
Solutie. Numarul 343 se mparte exact la 7, catul ind 49. Inseamna ca si
num
arul 343 . . . 3437 (2012 grupe de 343) se mparte exact la 7, catul ind
490490 . . . 490491 (2011 grupe de 490).
P.250. Patru frati au mpreun
a 45 de ani. V
arstele lor ar deveni egale dac
a
primul ar avea cu doi ani mai mult, al doilea cu doi ani mai putin, al treilea de dou
a
ori mai mult dec
at are, iar al patrulea jum
atate din v
arsta pe care o are. C
ati ani are
fiecare?

(Clasa a IV-a)
Inv. Valeria Avas
alcei, Iasi
Solutie. Dac
a varstele fratilor sunt a, b, c si d, atunci a + b + c + d = 45 si
a + 2 = b 2 = 2c = d : 2, de unde a = 2c 2, b = 2c + 2, d = 4c. Vom avea relatia
2c 2 + 2c + 2 + c + 4c = 45, de unde c = 5 ani, iar a = 8 ani, b = 12 ani, d = 20 ani.
P.251. Se consider
a urm
atorul sir de perechi de numere: (1, 100), (2, 99), (3, 98),
. . ., (100, 1). C
ate perechi (x, y) din sir au proprietatea 3x < 5y?
(Clasa a IV-a)
Nicoleta Cump
at
a, elev
a, Iasi
50

Solutie. 3x < 5y 3x + 3y < 8y 3(x + y) < 8y 303 < 8y. Deoarece


303 = 8 37 + 7, rezulta y 38 si y poate lua 100 38 + 1 = 63 valori. Satisfac
conditia problemei primele 63 perechi.
P.252. Se consider
a sirul 3; 4; 6; 9; 13; 18; 24; . . ..
a) Scrieti al 100-lea termen al sirului.
b) Ar
atati c
a termenul de pe locul 1113334 se mparte exact la 3.
(Clasa a IV-a)
Andreea Bzdg
a, elev
a, Iasi
Solutie. a) Observam ca 3 = 3 + 0, 4 = 3 + 1, 6 = 3 + 1 + 2, 9 = 3 + 1 + 2 + 3,
13 = 3 + 1 + 2 + 3 + 4, deci T100 = 3 + 1 + 2 + 3 + . . . + 99 = 4953.
b) T1113334 = 3 + 1113334 1113333 : 2 se mparte exact la 3, deoarece numarul
1113333 se mparte exact la 3.
P.253. V
arstele a doi frati sunt dou
a numere impare consecutive, iar v
arsta tat
alui
lor este de patru ori mai mare dec
at v
arsta fiului cel mare.
a) Ar
atati c
a suma v
arstelor tat
alui si fiilor se mparte exact la 4, dar nu si la 3.
b) Care din numerele 32, 44, 52 poate s
a reprezinte suma v
arstelor lor?
(Clasa a IV-a)
Amalia Munteanu, elev
a, Iasi
Solutie. a) Daca varsta ului mic este a, atunci suma varstelor tatalui si ilor
este 6a + 10, unde a este num
ar impar, deci de forma 2k + 1. Avem 6a + 10 =
12k + 16 = 4(3k + 4), care se mparte exact la 4, dar nu si la 3.
b) 12k + 16 = 32 12k = 16, fals; 12k + 16 = 44 12k = 28, fals; 12k + 16 =
52 12k = 36 k = 3 si varstele sunt 7 ani, 9 ani, 36 ani.
P.254. Cinci elevi joac
a urm
atorul joc. Primul scrie pe tabl
a dou
a numere distincte. Al doilea le nlocuieste cu suma si diferenta lor; la fel si ceilalti trei elevi.
In
final, pe tabl
a sunt scrise dou
a numere a c
aror sum
a este 128 si a c
aror diferent
a este
8. Ce numere a scris primul elev pe tabl
a?
(Clasa a IV-a)
Petru Asaftei, Iasi
Solutie. Primul elev a scris pe tabla numerele a si b, a > b, al doilea a scris
numerele a + b si a b, al treilea a scris numerele 2a si 2b. Al cincilea a scris pe tabla
numerele 4a si 4b, care satisfac simultan 4a + 4b = 128, 4a 4b = 8. Gasim a = 17 si
b = 15.

Clasa a V-a
V.151. Determinati cifra x (n baza 10) pentru care x + x2 + x3 + x4 + x5 = x6x.
Nicolae Iv
aschescu, Craiova
Solutie. Dac
a x 4, atunci x + x2 + x3 + x4 + x5 4 + 42 + 43 + 44 + 45 >
999 > x6x. Efectuand veric
ari pentru x {1, 2, 3}, convine doar cazul x = 3, cand
3 + 32 + 33 + 34 + 35 = 363.
V.152. Determinati restul mp
artirii num
arului A = 1 + 2 + 22 + . . . + 22012 prin
60.
Anca Chiritescu, T
ig
anasi (Iasi)
.
Solutie. Deoarece 2n + 2n+1 + 2n+2 + 2n+3 = 2n 15..60, n 2, iar A =
1+2+22 +23 +24 +(25 +26 +27 +28 )+. . .+(22009 +22010 +22011 +22012 ) = 31+M60 ,
rezulta ca restul cautat este 31.
51

V.153. Demonstrati c
a num
arul 3225 are cel putin 101 cifre.
Constantin Dragomir, Pitesti
Solutie (Titu Zvonaru, Comanesti). Deoarece 34 = 81 > 23 10, avem ca
3225 = 3(34 )56 > 3(23 10)56 = 328 2160 1056 = 768(210 )16 1056 > 768(103 )16 1056 =
768 10104 > 10106 , prin urmare numarul 3225 are cel putin 107 cifre (rezultat mai
bun decat cel cerut de enunt).
V.154. Demonstrati c
a exist
a o infinitate de p
atrate perfecte cu ultimele dou
a
cifre ale scrierii zecimale egale cu 69.
Cristian Laz
ar, Iasi
Solutie. Dac
a A = a1 a2 . . . an 13, atunci A2 = . . . 69.
V.155. Se consider
a num
arul natural A = 1 |00 {z
. . . 0} 44. Demonstrati c
a A nu
2012 de 0

este nici p
atrat perfect nici cub perfect.
Ioana Maria Popa, elev
a, Iasi
2
2
<
A
<
1
00
.
.
.
0
1
,
deci
A
nu
este
p
a
trat
perfect.
Solutie. Observam ca 100
.
.
.
0
| {z }
| {z }
1007 de 0

1006 de 0

Cum A se divide cu 2, dar nu se divide cu 23 , rezulta ca A nu este nici cub perfect.


V.156. Fie succesiunea de cifre 1 si 0 urm
atoare: 10110111011110111110 . . .
a) A 2012-a cifr
a este 0 sau 1?
b) C
ati de 0 sunt nainte de a 2012-a cifr
a? Dar de 1?
Temistocle Brsan, Iasi
Solutie. a) Observam ca numarul de cifre scrise pana la n-lea 0 (inclusiv acest 0)
este
n(n + 1)
n(n + 3)
(1 + 2 + 3 + . . . + n) + n =
+n=
.
2
2
Sa presupunem ca a 2012-a cifra din succesiune ar 0 si ca acest 0 ar al n-lea 0.
n(n + 3)
Conform observatiei precedente, am avea 2012 =
sau n(n + 3) = 4024. Cum
2
61 64 = 3904 < 4024 < 4030 = 32 65, rezulta ca a 2012-a cifra nu este 0, ci 1.
61 64
62 65
b) Am vazut la punctul a) ca
< 2012 <
. Din nou, utilizand
2
2
observatia facut
a, deducem ca sunt 61 de 0 pana la a 2012-a cifra din succesiune.
61 64
Apoi, pan
a la al 61-lea 0 din succesiune avem scrise
61 = 1891 cifre de 1,
2
iar dupa al 61-lea 0 urmeaza un grup de 62 cifre de 1, dintre care numai 60 cifre sunt
scrise pan
a la cifra a 2012-a (inclusiv). In total avem 1891 + 60 = 1951 cifre de 1 pana
la a 2012-a cifra din succesiune.
V.157. Pe un cerc se afl
a 160 de bile, numerotate n ordine cresc
atoare de la 1

la 160. Incep
and cu bila 1, se elimin
a bilele, din dou
a n dou
a, p
an
a r
am
ane una
singur
a. Ce num
ar este scris pe bila r
amas
a?
Petre B
atr
anetu, Galati
Solutie. La primul tur se iau de pe masa bilele impare; la al doilea, cele numerotate cu numere de forma 2k, k impar; la al treilea, cele cu numere de forma 22 l, l
impar etc. Dupa turul cinci, raman pe masa bilele cu numerele 25 = 32, 25 2 = 64,
25 3 = 96, 25 4 = 128 si 25 5 = 160. La turul sase se elimina bilele 32, 96 si 160.
52

La turul sapte se sare bila 64 si se ia bila 128, deci utlima bila ramasa este cea cu
numarul 64.

Clasa a VI-a
VI.151. Fie numerele p1 , p2 , . . . , p6 prime si cel putin egale cu 5. Ar
atati c
a suma
p21 + p22 + . . . + p26 se divide cu 6, dar nu se divide cu 12.
Camelia Dan
a, Craiova
Solutie. Dac
a p 5 este num
ar prim, atunci p = M6 1, deci p2 = M12 + 1.
Rezulta ca p21 + p22 + . . . + p26 = M12 + 6, de unde cerinta problemei.
a b
VI.152. Demonstrati c
a nu exist
a a si b cifre nenule pentru care + = a, b+b, a.
b a
Bianca Petrescu, elev
a, Iasi
a
a
Solutie. Avem a = 0 si b = 0. Evident, a si a < a, b, deci < a, b. Analog,
b
b
b
a
b
< b, a. Ca urmare, + < a, b + b, a.
a
b
a
VI.153. Determinati num
arul solutiilor ntregi ale ecuatiei x2 + y 2 + z 2 = |x|
|y| + z.
Elena Iurea, Iasi
Solutie. Daca (x, y, z) Z3 este o solutie, atunci x2 |x| + y 2 + |y| + z 2 z = 0.
Insa x2 |x| 0 (cu egalitate pentru x {1, 0, 1}), y 2 + |y| 0 (cu egalitate cand
y = 0) si z 2 z 0 (cu egalitate pentru z {0, 1}). Cele trei egalitati trebuie sa se
atinga simultan, prin urmare exista 3 1 2 = 6 solutii ale ecuatiei din enunt.
VI.154. Determinati n N pentru care exist
a numerele naturale a1 , a2 , . . . , an
cu proprietatea c
a a1 + a2 + . . . + an + n = a1 a2 . . . an .
Gheorghe Iurea, Iasi
Solutie. Valoarea n = 1 nu convine; e n 2. Cautam a1 , a2 , . . . , an cu cat mai
multe dintre numere egale cu 1. Consideram deci a3 = a4 = . . . = an = 1 si a1 , a2
satisfac egalitatea a1 + a2 + 2n 2 = a1 a2 , i.e. (a1 1)(a2 1) = 2n 1. Luam
a1 = 2n, a2 = 2 si rezulta ca pentru orice n 2, exista a1 , a2 , . . . , an cu proprietatea
din enunt.
VI.155. La mp
artirea a dou
a numere naturale nenule, demp
artitul si mp
artitorul sunt direct proportionale cu restul, respectiv c
atul. Restul si c
atul sunt prime
ntre ele. Ar
atati c
a demp
artitul este p
atrat perfect si aflati cele mai mici trei valori
posibile ale acestuia.
Gheorghe Bumb
acea, Busteni
Solutie. Not
am cu a, b, c si r dempartitul, mpartitorul, catul, respectiv restul;
b
a
avem ca a = bc + r, cu r < b, apoi = (= k) si (r, c) = 1. Din ac = br obtinem ca
r
c
c|br si, cum (r, c) = 1, deducem ca c|b, prin urmare k N . Atunci: a = bc + r
rk = c2 k + r r(k 1) = c2 k r|c2 k; deoarece (r, c2 ) = 1, rezulta ca r|k. Pe de alta
parte, r(k 1) = c2 k k|r(k 1) si, cum (k, k 1) = 1, obtinem k|r, prin urmare
k = r. In acest fel, a = r2 este patrat perfect. Avem ca r2 = c2 r + r si, cum r = 0,
deducem ca r = c2 + 1. Daca c = 1, ar urma ca r = b = 2, imposibil. Cele mai mici
trei valori ale lui a, se gasesc pentru c {2, 3, 4} si sunt 25, 100, 289.
53

b = 30 .
VI.156. Se consider
a triunghiul isoscel ABC, cu AB = AC si m(A)
astfel ncat m(DCB)
= 15 si
Punctul D se afl
a n interiorul unghiului ACB,

= 75 . Demonstrati ca AC = AD.
m(DBA)
Petrisor Rocsoreanu, Craiova

A
Solutie. Cum m(BCD) = 15 si m(CBD) = 75 +75 =

= 15 , prin urmare BCD este


150 , rezulta ca m(BDC)
isoscel cu BC = BD. Atunci ABC ABD (L.U.L.),
deci AC = AD.
VI.157. Se consider
a triunghiul ABC cu AC = 1,
= 10 si m(C)
= 20 . Demonstrati ca perimetrul tri- D
m(B)
unghiului este mai mic dec
at 6.
Petre B
atr
anetu, Galati
C
B

Solutie. Fie D (BC) astfel ncat AD = AC = 1; atunci m(ADC) = m(C) =


este unghi exterior triunghiului ABD, prin urmare
20 . Pe de alta parte, ADC

Obtinem de aici ca m(DAB)


= 10 , asadar DAB
m(ADC) = m(DAB) + m(B).
este isoscel cu DA = DB = 1. Inegalitatea triunghiului aplicata n triunghiurile
ACD si DAB arata ca DC < AD + AC < 2, iar AB < AD + BD = 2. Astfel,
PABC = AB + (BD + DC) + AC < 2 + (1 + 2) + 1 = 6.

Clasa a VII-a

VII.151. Comparati numerele A = 4 8 + 3 + 15 + 10 2 5 si B =


( 5 1)( 6 2) + ( 5 + 1)( 3 + 1) 5 5.
Ionel Tudor, C
alug
areni
(Giurgiu)

2
2
Solut

ie.
Efectu
a
nd
calculele,
se
constat
a
c
a
A
=
B
=
128
+
16
3 + 16 15 +

16 10 2 5. Cum A > 0 si B > 0, rezulta ca A = B.


VII.152. Determinati numerele prime p, q si r, stiind c
a 2p = 2012 + q 2 r2 .
Adriana Dragomir si Lucian Dragomir, Otelu-Rosu
Solutie. Deoarece 2p > 2012, deducem ca p 11. Pentru p = 11 obtinem ca
qr = 6, deci (q, r) {(2, 3); (3, 2)}. Vom arata ca acestea sunt singurele solutii ale
problemei.
Pentru p 13, cum 2p si 2012 sunt numere pare, deducem ca r sau q este numar
par, asadar r = 2 sau q = 2. Fie, de exemplu, q = 2 si notam p 2 = s, cu s
impar s 11. Impartind prin 4 egalitatea data, ajungem la 2s = 503 + r2 . Insa
2s = 22k+1 = (22 )k 2 = (3 + 1)k 2 = (M3 + 1) 2 = M3 + 2, 503 = M3 + 2, deci
r2 = M3 . Singurul num
ar prim care se divide cu 3 este r = 3, caz n care s = 9,
contradictie!
VII.153. Dac
a n este num
ar natural par, ar
atati c
a putem alege p {3, 5, 6}
astfel nc
at num
arul 2n + 5n + p s
a fie multiplu al lui 7.
Radu Gologan, Bucuresti
Solutie. Avem 2n + 5n = 2n + (7 2)n = 2n + M7 + (2)n = M7 + 2 2n =
M7 + 2n+1 (am tinut seama de paritatea lui n). Daca n + 1 = 3k, k N, atunci
2n+1 = (23 )k = (7 + 1)k = M7 + 1 si alegem p = 6. Daca n + 1 = 3k + 1, atunci
54

2n+1 = (23 )k 2 = M7 + 2 si luam p = 5. In sfarsit, daca n + 1 = 3k + 2, avem ca


2n+1 = (23 )k 22 = M7 + 4, deci putem considera p = 3.
VII.154. Demonstrati c
a nu exist
a numere ntregi x cu proprietatea c
a
x5 + (x + 1)5 + . . . + (x + 10)5 = 13311332.
S
tefan Dominte, elev, Iasi
Solutie. Resturile la mp
artirea prin 11 a termenilor sumei din stanga sunt
0, 1, 10, 1, 1, 1, 10, 10, 10, 1, 10 (sau o permutare circulara a acestora), prin urmare
suma din stanga se divide cu 11. Cum numarul 13311332 da restul 1 la mpartirea
prin 11, egalitatea din enunt nu poate avea loc.
VII.155. Ar
atati c
a exist
a o infinitate de numere naturale nenule a, b, c, d astfel
nc
at a3 = b2 + c4 + d8 .
Lucian Tutescu, Craiova
Solutie. Observam ca (33 )3 = (34 )2 +(32 )4 +38 . Cum c.m.m.m.c. al exponentilor
3, 2, 4 si 8 este 24, numerele de forma a = k 8 33 , b = k 12 34 , c = k 6 32 , d = k 3 3,
k N , formeaza o familie innita de solutii pentru ecuatia data.
AB
VII.156. Fie ABCD un trapez cu bazele AB si CD, astfel nc
at raportul k =
CD
este num
ar rational.
AC BD.
r Notam {O} = r
AABCD
AABCD
a) Ar
atati c
a
Q si
Q.
AAOB
rACOD
r

AABCD
AABCD
b) Dac
a, n plus, k Q, atunci
Q si
Q.
AAOD
ABOC
Claudiu-S
tefan Popa, Iasi
AB
D
C
Solutie. a) Din AOB COD si
= k, rezulta
CD
AAOB
AAOB
OB
OA
AAOD
O
ca
= k 2 . Apoi,
=
=
=
,
ACOD
AAOD
OD
OC
ACOD

deci AAOD = AAOB ACOD . Atunci AABCD = AAOB +


B
A
ABOC + ACOD + AAOD = k 2 ACOD + ACOD + 2AAOD = r

A
ABCD
(k 2 + 1)ACOD + 2 k 2 A2COD = (k + 1)2 ACOD , prin urmare
= k + 1 Q.
A
COD
r
k+1
AABCD
Analog se arata ca
=
Q.
AAOB
k
AABCD
b) Cum AAOD = ABOC , este sucient sa dovedim prima armatie. Avem
=
AAOD

AAOB
ACOD
BO
CO
1
k+1

2+
+
= 2+
+
= 2+k+ =
si, cum k Q,
AAOD r AAOD
DO
AO
k
k
AABCD
Q.
rezulta ca
AAOD

b < m(B)

VII.157. Se consider
a triunghiul isoscel ABC cu AB = AC si m(A)

b
si fie M un punct pe latura AC. Demonstrati c
a M BC A dac
a si numai dac
a
M B 2 = AC M C.
Mirela Marin, Iasi
55


b triunghiurile BM C si ABC sunt
Solutie. Daca M
BC A,

MC
BM
=
. De aici, rezulta ca
BC
AB
2
M B = AC M C. Reciproc, e AB = AC = a si CM = x, astfel
nc
at M B = ax. Din relatia lui Stewart BC 2 AM BM 2 AC +
2
2
AB 2 M C = M C AM AC,
M
obtinem ca BC (a x) ax a + a x =
x(a x)a, adica BC = a x. Rezulta ca BC = M B, de unde

b = 180 2m(C).

m(M
BC) = m(A)
C
B
Pentru o alta demonstratie a reciprocei, se poate considera punctul D pe semidreapta (BC astfel nc
at BM = M D. Folosind cazul I de asemanare se arata ca
A.
b Insa D
M

ABM DM C, de unde obtinem ca D


BC si de aici rezulta
concluzia problemei.
asemenea, deci BM = BC si

Clasa a VIII-a
VIII.144. Dac
a a, b, c R, demonstrati identitatea
2(a2 + b2 + c2 ab ac bc)2 = (a b)4 + (b c)4 + (c a)4 .
Marian Tetiva, B
arlad
Solutie. Notam x = a b, y = b c, z = c a, cu x + y + z = 0. Trebuie
sa dovedim ca x4 + y 4 + z 4 = 2(xy + yz + zx)2 . Cum z = x y, aceasta ultima
identitate revine la x4 + y 4 + (x + y)4 = 2(x2 + xy + y 2 )2 , care se verica prin calcul
direct.
VIII.152. Determinati numerele reale x, y si z pentru care x2 y 2 = x y + 2z,
y z 2 = y + z + 2x si z 2 x2 = x z + 2y.
Vasile Chiriac, Bac
au
Solutie. Adun
and ultimele doua ecuatii, gasim ca (x + y)(y x 3) = 0, de
unde y = x sau y = x + 3. In primul caz, relatiile din problema se reduc la
x + z = 0, deci x = a, y = z = a, cu a R. In al doilea caz obtinem, dupa
nlocuire, ca 6x 9 = 3 + 2z; x2 + 3x + 6 = z 2 + z z = 3x 3; 2x2 + 3x = 0,
3 3 3
prin urmare (x, y, z) {(0, 3, 3); ( , , )}. In concluzie, S = {(a, a, a)|a
2 2 2
R} {(0, 3, 3)}.

VIII.153. Fie numerele a, x, y, z reale, pozitive si astfel nc


at x yz + y zx +

z xy a. Ar
atati c
a x + y + z 3a.
Lucian Tutescu, Craiova si Marian Voinea, Bucuresti

y+z
z+x
x+y

Solutie. Avem ca x
+y
+z
x yz + y zx + z xy a, deci
2
2
2
x(y + z) + y(z + x) + z(x + y) 2a. Obtinem de aici c
a xy + yz + zx a. Insa

2
(x + y + z) 3(xy + yz + zx), prin urmare x + y + z 3a.
2

VIII.154. Dac
a n Z, consider
am An = n6 + (n + 3)6 + (3n + 4)6 + (3n + 5)6 .
Demonstrati c
a An se divide cu 2n2 + 6n + 5.
Cosmin Manea si Dragos Petric
a, Pitesti
Solutie. Intruc
at a6 + b6 = (a2 + b2 )(a4 a2 b2 + b4 ), a, b Z, rezulta ca
a2 + b2 |a6 + b6 , a, b Z. Deducem de aici ca n2 + (3n + 5)2 |n6 + (3n + 5)6
56

10n2 + 30n + 25|n6 + (3n + 5)6 si ca (n + 3)2 + (3n + 4)2 |(n + 3)6 + (3n + 4)6
10n2 + 30n + 25|(n + 3)6 + (3n + 4)6 . Cum 2n2 + 6n + 5|10n2 + 30n + 25, rezulta ca
2n2 + 6n + 5|An .
VIII.155. Dac
a x, y, z R si x2 + y 2 + z 2 = x3 + y 3 + z 3 = 1, calculati xyz.
Lenuta Andrei, Craiova
Solutie. Din x2 + y 2 + z 2 = 1 rezulta ca |x| 1, |y| 1 si |z| 1. Daca
x, y, z [1, 1]\{0}, din x2 (x1)+y 2 (y1)+z 2 (z1) = 0 urmeaza ca x = y = z = 1,
fals. Ram
ane ca unul dintre numerele x, y sau z este zero, asadar xyz = 0.
VIII.156.
In cubul ABCDA B C D cu muchia de lungime a, not
am cu N mijlocul muchiei A D . Ar
atati c
a dreptele BN si DC sunt perpendiculare si calculati
distanta dintre ele.
Mirela Marin, Iasi
Solutie. Observam ca DC (A BC) (deoarece DC CD si DC BC)
si, cum BN (A BC), rezulta ca DC BN.
C
B
Fie {O} = DC CD si P = P rBN O; evident ca
A
N
OP (A BC) si, deoarece DC (A BC), obtinem ca D
DC OP. Deducem ca OP este perpendiculara comuna
P
O
a dreptelor BN si DC , prin urmare dist (BN, DC ) = OP .
Pentru a aa lungimea segmentului OP , calculam n doua
B
moduri aria triunghiuluiBON : ABON = ABCD A ABCO
C
2
3a 2
BN OP
3a
A
ABA N AOD N =
si ABON =
=
OP , D
8
2
4

2
3a 2 3a
a 2
asadar OP =
:
=
.
8
4
2
VIII.157. Ar
atati c
a suma distantelor de la centrul de greutate al unui tetraedru
la fetele sale este cel putin egal
a cu suma distantelor de la centrul sferei nscrise n
tetraedru la fetele acestuia.
Aurel B
arsan, Brasov
Solutie. Not
am cu di distanta de la G la fata de arie Si , i {1, 2, 3, 4} si cu
r reza sferei nscris
a n tetraedru. Cum G mparte tetraedrul n patru tetraedre cu
3V
acelasi volum, rezulta ca d1 S1 = d2 S2 = d3 S3 = d4 S4 =
, unde V este volu 4

3V
1
1
1
1
mul tetraedrului. Deducem ca d1 + d2 + d3 + d4 =
+
+
+

4 S1
S2
S3
S4
3V
16
, conform inegalitatii dintre media armonica si cea aritmetica.
4 S1 + S2 + S3 + S4
3V
Insa
= r si de aici urmeaza concluzia problemei. Egalitatea se
S1 + S2 + S3 + S4
atinge n cazul tetraedrului echifacial.

Clasa a IX-a
IX.131.

Rezolvati ecuatiile:
5
b) x[x] + x{x} + {x}[x] = .
4

a) x[x] + x{x} + {x}[x] = x2 + [x]2 + {x}2 ;


Mariana M
arculescu, Craiova
57

Solutie. a) Se stie ca a2 + b2 + c2 = ab + bc + ca a = b = c, prin urmare relatia


data are loc doar daca x = [x] = {x}. Atunci x Z [0, 1) = {0}, deci singura solutie
a ecuatiei este x = 0.
5
b) Ecuatia data se scrie sub forma x([x] + {x}) + {x}(x {x}) +
= 0
4
5
x2 + {x}x + {x}2 = 0, cu discriminantul = {x}2 + 4{x}2 5 < 0 (deoarece
4
{x}2 < 1). Rezulta ca ecuatia nu are solutii.
3
2

2
1 + 1 + 4(n + 1)2
1
+
1
+
4n
5=
, n N .
IX.132. Demonstrati c
a4
2
2

Ovidiu Pop, Satu Mare

1 + 1 + 4(n + 1)2
1 + 4n2
, yn =
, n N . Cum
2
2
xn , yn (n, n + 1), n N , rezulta ca [xn ] = [yn ] = n.
Solutie. Notam xn =

1+

IX.133. Determinati sirul de numere reale (an )n1 din intervalul

0,

1
, stiind
2

c
a sirurile (sin an )n1 si (cos an )n1 sunt progresii geometrice.
Dumitru Cr
aciun, F
alticeni

1

Solutie. Cum 0,
0,
, sirurile (sin an )n1 si (cos an )n1 au termenii
2
2
strict pozitivi. Avem sin2 an = sin an1 sin an+1 , iar cos2 an = cos an1 cos an+1 .
Adunand, apoi scaz
and membru cu membru aceste egalitati, obtinemca 1 =
cos(an+1
1 1
an1 ), respectiv cos 2an = cos(an+1 + an1 ). Cum an+1 an1 ,
, rezulta
2 2
ca an+1 an1 = 0, deci
 an+1
 = an1 , n 2. Deducem cos 2an = cos 2an+1 si,
ntruc
at an (0, 1) 0,
, urmeaza ca an = an+1 , n 1. Evident ca toate
2
sirurile constante veric
a ipotezele problemei.
IX.134. Fie (n = An An+1 An+2 )n1 un sir de triunghiuri dreptunghice coplanare,
fiecare av
and c
ate un unghi ascutit de 30 , astfel nc
at ipotenuza An An+1 a lui n+1
este cateta opus
a unghiului de 30 n triunghiul n , iar celelalte dou
a laturi ale lui
n+1 sunt n interiorul lui n , oricare ar fi n N . Demonstrati c
a exist
a un unic
punct M situat n interiorul tuturor triunghiurilor n , iar distantele an = M An
1
formeaz
a o progresie geometric
a de ratie q = .
2
Silviu Boga, Iasi
Solutie. Notam n = An An+1 An+2 ; sirul de puncte (An )n1 se obtine recurent
astfel: An+3 este proiectia lui An+1 pe mediana corespunzatoare ipotenuzei An An+1 a
bn+1 ) = 60 . Se observa ca triunghiurile
triunghiului n , care este dreptunghic cu m(A

n si n+3 , n N , sunt omotetice, toate cu acelasi centru de omotetie M si n acelasi


raport de omotetie. Pozitia punctului M este determinat
a prin {M } = A1 A4 A2 A5
A3 A
.
Consider
a
nd
reprezentarea
analitic
a
A
(0,
6
1



3);
2 (1, 0); A3 (0, 0), obtinem:
A

1
3
1
2
3
3
3
3
7
2
A4
,
; A5
,
; A6
,
si M
,
. Atunci a1 = 2
,
4 4
8 8
4 8
7 7
7
58

7
7
0
1 7
a2 = 2
, a3 = 2
, a4 = 2
, rezultand o progresie geometrica de ratie
7
7
7
1
q= .
2
IX.135. Ar
atati c
a dou
a triunghiuri care au egale perimetrele, razele cercurilor
nscrise si c
ate o n
altime sunt congruente.
Temistocle Brsan, Iasi
Solutie. Fie ABC si A B C astfel ncat p = p , r = r si ha = ha . Avem
2S
2S
ca S = pr = p r = S , deci a =
= = a . Apoi, A = A ; ntr-adevar,
ha
ha
A
A
A
A

r = (p a) tg si r = (p a ) tg
conduc la tg = tg , de unde rezulta relatia
2
2
2
2
a
a

indicata. Atunci R =
=
= R , apoi abc = 4RS = 4R S = a b c ,

2
sin
A
2
sin
A
prin urmare bc = b c . Avem ns
a si ca b + c = p a = p a = b + c , asadar
perechile (b, c) si (b , c ) au aceeasi suma si acelasi produs. Obtinem ca b = b , c = c
sau b = c , c = b si, n ambele situatii, ntre triunghiurile ABC si A B C exista
cate o congruent
a.
1

Clasa a X-a

X.131. Intr-o
urn
a se afl
a 2 bile albe si 3 bile negre, iar n alt
a urn
a se afl
a 4 bile
albe si 1 bil
a neagr
a. Se alege aleator o urn
a, din care se extrage o bil
a. Consider
am
evenimentul B: bila extras
a este alb
a, iar A este un eveniment compatibil cu B astfel
4
nc
at P (A B) = si PA (B) = P (B). Calculati P (A).
5
Laurentiu Modan, Bucuresti
Solutie. Consideram evenimentele Ai : se alege
urna Ui , i = 1, 2; atunci P (B) =
3
1 2 4
= . Apoi, cum PA (B) = P (B),
PA1 (B) P (A1 ) + PA2 (B) P (A2 ) =
+
2 5 5
5
rezulta ca P (A B) = P (A) P (B). Evenimentele A si B ind compatibile, avem
4
3
3
ca P (A B) = P (A) + P (B) P (A) P (B), de unde = P (A) + P (A) , prin
5
5
5
1
urmare P (A) = .
2
1
1
1
X.132. Demonstrati c
a
+
+
3.
log 12 sin 70
log 12 sin 50
log 12 sin 10
Bogdan Victor Grigoriu, F
alticeni
Solutie. Remarc
am nt
ai ca numitorul ecarei fractii este strict pozitiv. Folosind
inegalitatea lui Bergstrom, obtinem ca suma din membrul stang este cel putin egala
cu
9
(1 + 1 + 1)2
=
log 12 sin 70 + log 21 sin 50 + log 12 sin 10
log 12

1
8

= 3.

1
Am folosit identitatea sin 70 sin 50 sin 10 = , care se demonstreaza usor trans8
formand produsele n sume.
59

X.133. Determinati tripletele de numere complexe nenule (x, y, z) pentru care


1
1
1
3
+ 6
+ 6
=
.
2|xyz|2
x6 + x6
y + y6
z + z6
Florin St
anescu, G
aesti
Solutie. Se arata imediat ca z 2 + z 2 2|z|2 , z C, cu egalitate daca si numai
daca z R. Atunci, tin
and cont si de inegalitatea mediilor, avem:
1
3
1
1
= 3 2
+ 3 2
+ 3 2

2|xyz|2
(x ) + (x3 )2
(y ) + (y 3 )2
(z ) + (z 3 )2
1
1
1
3

+
+

3
2
3
2
3
2
2|x |
2|y |
2|z |
2

1
3
=
.
6
|xyz|
2|xyz|2

Astfel, inegalitatile devin egalitati; deducem ca x3 , y 3 , z 3 R (de unde x6 , y 6 , z 6


R+ ) si |x|6 = |y|6 = |z|6 , prin urmare x6 = y 6 = z 6 , deci x3 = y 3 = z 3 .
Notand, de
exemplu, z = C , unde 6 R+ , iar x, y {, , 2 }, unde
1 i 3
= +
este rad
acina primitiva de ordin trei a unitatii, iar tripletele (x, y, z)
2
2
av
and aceste forme veric
a ipoteza problemei.
X.134. Fie A si B dou
a puncte fixate pe un cerc de centru O si raz
a R, astfel
= 2 < 90 . Determinati doua puncte C si D ale cercului astfel ncat
nc
at m(AOB)
aria patrulaterului ABCD s
a fie maxim
a.
Adrian Corduneanu si Paul Georgescu, Iasi
CD,
DA
sa aiba masurile
Solutie. Fie C, D puncte pe cerc astfel ncat arcele BC,
si
1 , 2 respectiv 3 ; evident, vom considera punctele C si D pe arcul mare AB
1 2
atunci 2 + 1 + 2 + 3 = 2, 0 < i < . Avem ca AABCD = R (sin 2 + sin 1 +
2
sin 2 +sin 3 ), deci avem de maximizat suma sin 1 +sin 2 +sin 3 . Functia sinus sin 1 + sin 2 + sin 3
ind concav
a pe (0, ), din inegalitatea lui Jensen obtinem ca

1 + 2 + 3
2 2
1
2( )
.
sin
= sin
, prin urmare AABCD R2 sin 2 + 3 sin
3
3
2
3
Se constata ca aceasta valoare maxima a ariei se atinge n cazul n care 1 = 2 =
2( )
3 =
, deci doua puncte C, D cu proprietatile din enunt se pot gasi con3
siderand trapezul isoscel ABCD cu BC = CD = DA.
X.135. Demonstrati c
a n orice triunghi ABC are loc inegalitatea
cos2 A
cos2 B
cos2 C
+
+
1.
sin B sin C
sin A sin C
sin A sin B
I.V. Maftei, Bucuresti si Mihai Haivas, Iasi
Solutie. Inegalitatea se rescrie succesiv astfel:

X 1 sin2 A

X 4R2 a2

1
sin B sin C
bc
4R2 (a + b + c) (a3 + b3 + c3 ) abc
4R2 2p 2p(p2 3r2 6Rr) 4Rrp p2 4R2 + 3r2 + 4Rr.
60

Aceasta inegalitate este o consecint


a a cunoscutei a2 + b2 + c2 8R2 + 4r2 .

Clasa a XI-a
XI.131. Se consider
a un triunghi

0 a

a 0
1
|V (a, b, c)|, unde V (a, b, c) =
4
b c
c b

ABC,
cu notatii uzuale. Aratati ca S =
b c
c b
.
0 a
a 0
Constantin Dragomir, Pitesti
Solutia 1 (data de elevele Nicoleta Srghi, Iasi, Monica Golea si Andreea
Enuca, Craiova). Dezvolt
and determinantul dupa prima linie, obtinem ca V (a, b, c) =
a4 + b4 + c4 2a2 b2 2a2 c2 2b2 c2 = 16S 2 , dupa o formula binecunoscuta pentru
arie, si, de aici, concluzia problemei.



0 X b c


X 0 c b
.
Solutia 2 (a autorului). Consideram polinomul de grad IV: P (X) =

b c 0 X
c b X 0

Se observa ca P (b+c) = P (bc) = P (bc) = P (cb) = 0, si, cum P are coecientul


dominant 1, rezulta ca P (X) = (X + b + c)(X b c)(X b + c)(X + b c). Deducem
ca V (a, b, c) = P (A) = 16p(p a)(p b)(p c) si, de aici, concluzia urmeaza tinand
cont de formula lui Heron.
XI.132. Determinati functiile continue f : R R care au proprietatea c
a
f (x + y) = f (x) f (y) 20122xy , x, y R, iar f (1) = 20122 .
Carmen Liana Georgescu, Craiova
2
Solutie. Denim functia auxiliara g : R R prin f (x) = g(x) 2012x , x R.
Functia g este continu
a, iar g(x + y) = g(x)g(y), x, y R, x, y R; rezulta ca
f (1)
x
g(x) = a , x R, unde a (0, ). Cum g(1) =
= 2012, obtinem ca a = 2012
2012
2
2
si atunci f (x) = 2012x 2012x = 2012x+x , x R.

XI.133. Dac
a un =

(n + 2)n+1

, n N , calculati lim ( n+1 u1 u2 . . . un+1


n
(n + 1)n

u1 u2 . . . un ).

D.M. B
atinetu-Giurgiu, Bucuresti si Neculai Stanciu,
Buz
au

n+1 u u . . . u
1 2
n+1
Solutie. Notand an sirul a carui limita trebuie calculata si vn =
,
n u u ...u
1 2
n
n 2, avem ca

n u u ...u

1 2
n vn 1
an = n u1 u2 . . . un (vn 1) =

ln vnn , n 2.
n
ln vn

n u u ...u
u1 u2 . . . un+1
nn
u 1 u 2 . . . un
1 2
n
Insa lim
= lim n
=
lim

=
n
n (n + 1)n+1
n
n
nn
u 1 u 2 . . . un

n+2
un+1
1
1
(n + 3)n+2
1
n+2
1
n = lim
lim

=
lim 1 +
=
n+1
n n + 1
1
e n (n + 2)
(n + 1)
e n
n+2
n+1
1+
n
61

vn 1
1
e 1; obtinem ca lim vn = 1, deci lim
= 1; n sfarsit, lim vnn =
n
n ln vn
n
e
un+1
n+1

lim
n+1
= e 1 = e. In aceste conditii, lim an = 1 1 ln e = 1.

n n + 1
n
u1 u2 . . . un+1
XI.134. Fie A R o multime nevid
a. Dac
a A2 = {x2 : x A}, determinati
intervalele m
arginite de numere reale I R care satisfac relatia I 2 = I.
Dan Popescu, Suceava
Solutie. Fie I = [a, b] un interval pentru care I 2 = I; evident ca b > a 0. Din
Din continuitatea functiei f : R R, f (x) = x2 , rezulta ca I 2 = [a2 , b2 ], prin urmare
a2 = a, b2 = b. Deducem ca a = 0, b = 1. Analog se trateaza situatiile I = (a, b),
I = (a, b] si I = [a, b); intervalele nedegenerate care satisfac enuntul sunt cele cu
extremitatile 0 si 1.
XI.135. Fie f : [a, b] R o functie de clas
a C 2 cu proprietatea c
a (b a)f (a) =

f (b) f (a). Demonstrati c


a exist
a c (a, b) pentru care f (c) f (c) = f (a).
Dan Nedeianu, Drobeta Tr. Severin
Solutie. Consider
am functia auxiliara g : [a, b] R, g(x) = ex [f (x) f (a)
(x a)f (a)]. Din enunt, avem ca g este derivabila pe [a, b] si g(a) = g(b) = 0;
folosind teorema lui Rolle, rezulta ca exista (a, b) astfel ncat g () = 0, altfel
spus f () f (a) ( a) f (a) = f () f (a).
Aplic
am acum teorema lui Rolle functiei h : [a, ] R, h(x) = f (x) f (a)
[f (x) f (a) (x a)f (a)] si gasim c (a, ) pentru care h (c) = 0. Insa h (x) =
f (x) f (x) + f (a) si de aici urmeaza cerinta problemei.

Clasa a XII-a

XII.131. Fie n 2 un num


ar natural pentru care exist
a un element b
a al inelului
Zn cu proprietatea c
a b
a=b
a1 .

a) Dac
a a < n, demonstrati c
a n = a2 + 1.
b) Ar
atati c
a exist
a n 2 si a astfel nc
at n = a2 + 1.
Ovidiu Pop, Satu Mare

Solutie. a) Din b
a=b
a1 rezulta ca b
a2 + 1 = b
0, deci n|a2 + 1. Insa a < n, prin
urmare a2 + 1 n si atunci a2 + 1 = n.
b) De exemplu, putem considera n = 13, a = 5.
XII.132. Fie p 5 un num
ar prim. Demonstrati c
a num
arul 3p1 2p1 se
p
p
divide cu p, ns
a 3 2 nu se divide cu p.
R
azvan Ceuc
a, elev, Iasi
Solutie. Numarul p ind relativ prim cu 2 si cu 3, din teorema lui Fermat rezulta
ca 3p1 1 (mod p), 2p1 1 (mod p), prin urmare 3p1 2p1 0 (mod p). Pe
de alta parte, 3p 3 (mod p), 2p 2 (mod p), asadar 3p 2p 1 (mod p), adica
3p 2p nu se divide cu p.
XII.133. Functia derivabil
a g : R Im g are proprietatea c
a g (x) = 0, x R,

iar f : Img R este o functie derivabil


a astfel nc
at f (g(x)) = x, x R. Ar
atati
c
a g este inversabil
a, iar f este o primitiv
a a lui g 1 .
Dan Popescu, Suceava
62

Solutie. Functia g este strict monotona, deci g : R Img este inversabila,


cu inversa continu
a pe intervalul Img. Daca H este o primitiva a lui g 1 , atunci

f (g(x)) g (x) = x g (x) = g 1 (g(x))g (x) = H (g(x)) g (x), prin urmare exista
k R astfel nc
at f (g(x)) = H(g(x)) + k, x R. Deducem ca f (x) = H(x) + k
x Img.
XII.134. Fie f : [a, b] [0, ) o functie integrabil
a. Ar
atati c
a
n Z
Y

2k

f (x)dx (b a)

f2

k=0

n+1

(x)dx.

Romanta Ghit
a si Ioan Ghit
a, Blaj
Solutie. Folosind inegalitatea C-B-S, forma integrala, avem:

f2

n+1

(x)dx

Z b

Z

=
a

ba

2

(x)dx

f
a

1
=
(b a)2

f 2n+1 1 (x)

f (x)dx (

2n

1
=
ba

Z

(x)dx
a

2n

1
...
(b a)n

Rb

(x)dx

2n

(x)dx

a
2n1

f (x)dx)2 =

Z

(x)dx

2n1

2n

f (x)dx

b
2

1 dx

Z

(x)dx

! Z

2k

2

n Z
Y
k=1

2n1

f (x)dx

b
2

1 dx

2

f (x)dx
a

(x)dx

Rb

Daca a f (x)dx = 0, atunci a f (x)dx > 0; simpliciam prin aceasta cantitate si se


Rb
obtine inegalitatea dorita. Daca a f (x)dx = 0, inegalitatea din enunt este evidenta.
XII.135. Fie f : R R o functie impar
a si continu
a; ar
atati c
a

xf (cos x + sin x)dx =

f (cos x + sin x)dx.


0

Z
Solutie. Not
am Ik =

Adrian Corduneanu, Iasi


k
2
(k1)
2

xf (cos x + sin x)dx, k = 1, 4; integrala din membrul

stang va egala cu I1 +I2 +I3 +I4 = I. In I2 , I3 si I4 facem schimbarile de variabila y =


Z 2
3

x , y = x , respectiv y = x ; obtinem ca I2 =
(x + )f (cos x sin x)dx,
2
2
2
0
Z 2
Z 3
3
I3 =
)f ( cos x + sin x)dx.
(x + )f ( cos x sin x)dx, respectiv I4 =
(x +
2
0
0
63

Folosind imparitatea lui f , rezulta ca

Z

I=

xf (cos x + sin x)dx

(x + )f (cos x + sin x)dx +


0

Z

(x +

+
0

)f (cos x sin x)dx


2

f (cos x + sin x)dx

Cu schimbarea de variabil
a x = y, obtinem ca
2

cos x)dx =

f (cos x sin x)dx, deci

blemei se impune.

(x +
0

3
)f (cos x sin x)dx
2


=

f (cos x sin x)dx.

f (cos xsin x)dx =


0

f (sin x
0

f (cos x sin x)dx = 0 si concluzia pro-

Solutiile problemelor pentru preg


atirea
concursurilor propuse n nr. 2/2012
A. Nivel gimnazial
G226. C
ate dintre numerele de trei cifre n baza 10 se pot scrie sub forma abc +
ab + a?
Andrei Eckstein, Timisoara
Solutie. Ar
at
am mai ntai ca abc + ab + a = def + dc + d abc = def . Implicatia
reciproca este evident
a, iar 111a + 11b + c = 111d + 11c + f 111|11e + f 11b c,
108 11e+f 11bc 108 11e+f = 11b+c 11|f c, 9 f c 9 f =
c e = b a = d abc = def . Avem asadar 9 10 10 = 900 de numere distincte
de forma abc + ab + a, toate cel putin egale cu 100. Numere mai mari de 999 obtinem
numai daca a = 9 si b, c nu sunt ambele nule. In concluzie, exista 8 10 10 + 1 = 801
numere de aceasta forma.

1 1 1
1
G227. Se consider
a multimea M = 1, , , , . . . ,
. Demonstrati c
a pentru
2 3 4
100
fiecare n {3, 4, . . . , 15}, exist
a o submultime A = {a1 , a2 , . . . , an } a lui M si o alegere
convenabil
a a semnelor astfel nc
at a1 a2 . . . an = 0.
Gabriel Popa, Iasi
Solutie. Observ
am ca
1 1 1
1 1
1
1 1
1
= 0 (1),

= 0 (2),

=0
2 3 6
4 5 20
7 8 56
1
1
1
1
1
1
1
1

= 0 (4), iar

= 0 (5).
9 10 90
12 18 24 36 72

(3),

Luand (2), (1), (5), (2)+(3), (1)+(2), (2) + (5), (1)+(5), (1)+(2) +(3), (2)+(3)+(5),
(1)+(2)+(5), (1)+(2)+(3)+(4), (2)+(3)+(4)+(5), (1)+(3)+(4)+(5), obtinem cerinta
problemei.
64

G228. Spunem c
a num
arul natural m are proprietatea (P ) dac
a exist
a a, b N
7m + 4
a
astfel nc
at a+b 3m si
= . Not
am cu E(n) num
arul elementelor multimii
11m + 2
b
{1, 2, . . . , n} care au proprietatea (P ). Determinati n N pentru care E(n) = 2012.
Vlad Emanuel, Bucuresti
7m + 4
Solutie. Dac
a m are proprietatea (P), fractia
este reductibila, sim11m + 2
(7m + 4) + (11m + 2)
2
plicandu-se cel putin prin
= 6 + , deci exista d 7 astfel
3m
m
ncat d|7m + 4 si d|11m + 2. Dar d|11(7m + 4) 7(11m + 2), adica d|30 si atunci
d {10, 15, 30}. Daca m = 10k + r, r {0, 1, . . . , 9}, avem ca 10|(7m + 4, 11m + 2)
10|(7r + 4, 11r + 2) r = 8. Analog, scriind m = 15l + p, cu p {0, 1, . . . , 14},
.
obtinem ca 15|(7m + 4, 11m + 2) p = 8. In plus, 30|(7m + 4, 11m + 2) m 8..10
.
.
si m 8..15 m 8..30.
Denim E10 (n) ca ind cardinalul multimii {10k + 8|k N, 10k + 8 n} si
la fel denim E15 (n) si E30 (n). Din principiul includerii si excluderii, deducem ca
E(n) = E10 (n) + E15 (n) E30 (n). Considerand n = 30k + r, r {0, 1, . . . , 29}, din
cele de mai sus rezulta ca E(n) = 4k+, unde = 0 daca r {0, 1, . . . , 7}; = 1 daca
r {8, 9, . . . , 17}; = 2 daca r {18, 19, . . . , 22}, = 3 daca r {23, 24, . . . , 27};
.
= 4 daca r {28, 29}. Deoarece 2012..4, suntem e n situatia 4k = 2012, cu
r {0, 1, . . . , 7} si obtinem solutiile n {15090, 15091, . . . , 15097}, e n situatia
4k + 4 = 2012, cu r {28, 29} si atunci n {15088, 15089}. In nal, avem zece
solutii: n {15088, 15089, . . . , 15097}.

G229. Se consider
a numerele ntregi distincte a, b, c si d, cu proprietatea c
a
ab + ac + ad + bc + bd + cd 500. Demonstrati c
a a2 + b2 + c2 + d2 340.
Dan Nedeianu, Drobeta Tr. Severin
Solutie. Far
a a restrange generalitatea, putem presupune ca a < b < c < d;
atunci b a 1, c a 2, d a 3, c b 1, d b 2 si d c 1, prin
urmare (b a)2 + (c a)2 + (d a)2 + (c b)2 + (d b)2 + (d c)2 20. Rezulta
de aici ca 3(a2 + b2 + c2 + d2 ) 20 2(ab + ac + ad + bc + bd + cd) 1000, de unde
1020
a2 + b2 + c2 + d2
= 340.
3
x
y
z
1
G230. Ar
atati c
a 3
+ 3
+ 3
, x, y, z
y + z 3 10
z + x3 10
x + y 3 10
4
[1, 0).
Bogdan Chiriac, student, Iasi
Solutie. Folosind faptul ca y 3 + z 3 10 x3 + y 3 + z 3 9, x, y, z [1, 0) si
P
x+y+z
x
3
, x, y, z [1, 0). Ramane
analoagele, rezulta ca
y 3 + z 3 10
x + y3 + z3 9
x+y+z
1
sa aratam ca 3
, x, y, z [1, 0); aceasta inegalitate revine la
3
3
x +y +z 9
4
P
P
4x+4y+4z x3 +y 3 +z 3 9, adica (x3 +4x+3) 0, altfel spus (x+1)(x2 +x+
3) 0, ceea ce este adevarat (x+1 0 si x2 +x+3 = (1+x)(1x)+(1+x)+1 1,
x [1, 0)).
65

G231. Ar
atati c
a oricum am aseza 2012 puncte n interiorul unui triunghi echilateral de latur
a 2012, exist
a cel putin dou
a puncte astfel nc
at distanta dintre ele s
a
fie mai mic
a dec
at 46.
Nicolae Iv
aschescu, Craiova

Solutie. Impartim ecare latura n cate 44 de segmente egale si, prin punctele
obtinute, ducem paralelele la laturi. Obtinem astfel 1+3+5+. . .+(4421) = 442 =
2012
1936 triunghiuri echilaterale cu latura
= 45, 73 . . . < 46. Conform principiului
44
cutiei, vor exista doua puncte n interiorul unui acelasi astfel de triunghi, de unde
cerinta problemei.

= 90 si AB =
G232. Fie ABCD un trapez dreptunghic cu ABCD, m(B)
BC = a. Not
am cu E mijlocul segmentului BC. Demonstrati c
a AE este bisectoarea
daca si numai daca CD = a .
unghiului BAD
4
Adrian Zanoschi, Iasi
Solutie. Fie CD = x si {F } = AE CD; atunci CEF BEA (C.U.), deci
EAB
DAE

CF = AB = a. Avem:
DAE
D C
F

DF
E DA = DF
(a x)2 + a2 = a + x
a
2a2 2ax + x2 = a2 + 2ax + x2 4ax = a2 x = .
4
E
Not
a. Dac
a AB = a si BC = b atunci, n ipoteza
daca si
problemei, AE este bisectoarea unghiului BAD
A
B
b2
; demonstratia este analoaga celei de mai sus.
numai daca CD =
4a
G233. Determinati punctele M din planul triunghiului echilateral ABC cu proprietatea c
a M B = 2M A si M C = 3M A.
Temistocle Brsan, Iasi
Solutia 1. Avem (*) M C = M A + M B, deci triunghiul construit cu segmentele
[M A], [M B] si [M C] este degenerat. Conform teoremei lui Pompeiu, M se aa
A
pe cercul circumscris triunghiului echilateral. Tot din relatia (*)
M

mai rezulta ca M se aa pe arcul AB ce nu contine punctul C


(teorema Van Schooten). Pe de alta parte, conform teoremei lui
Pitagora generalizata aplicata n M AB, avem:
AB 2 = M A2 + (2M A)2 + M A(2M A),

1
. Ultima relatie ne conduce la un punct unic
7
(arc ce nu contine varful C) care satisface conditiile problemei.
pe AB
2

adica AB = 7M A sau M A = AB

Solutia 2. Consideram n planul triunghiului reperul xoy (BC ind axa Ox,
iar perpendiculara din A pe BC axa Oy).
Daca latura triunghiului ABC este
inem sisAB =2, atunci C(1, 0), B(1, 0) si A(0, 3). Considerand M (x,
y), obt


(x + 1)2 + y 2 = 4(x2 + (y 3)2 )
5 6 3

, cu unica solutie M ,
.
temul
7 7
(x 1)2 + y 2 = 9(x2 + (y 3)2 )
66

A
G234. Fie ABC un triunghi cu AB < AC, D un
punct pe latura AC astfel nc
at AD = AB si M mijlocul
laturii BC. Paralela la AC prin M intersecteaz
a pe BD
n punctul E, iar dreapta AE intersecteaz
a latura BC n
punctul F . Dac
a paralela prin F la AC intersecteaz
a pe

BD n punctul T , demonstrati c
a BAT M AC.
Titu Zvonaru, Com
anesti
Solutie. Not
am {S} = AT BC. Cu teorema lui B

D
T E

C
SF M
AD F C EB

= 1;
Menelaus n BDC si transversala A E F obtinem ca
AC F B ED
EB
MB
FC
AC
dar
=
= 1, deci
=
. Aplicand din nou teorema lui Menelaus n
ED
MC
FB
AB
AD SC T B
BDC, dar cu transversala AT S, obtinem ca

= 1. Cum T F DC,
AC SB T D
FB
AB
SB
AB 2
TB
=
=
si relatia precedenta conduce la
=
avem
, adica AS este
TD
FC
AC
SC
AC 2
simedian
a n ABC si deci ]BAT = ]M AC.
G235. Fie ABC un triunghi ascutitunghic, M punctul n care bisectoarea din A
retaie cercul circumscris, iar I centrul cercului nscris. Fie D piciorul n
altimii din
A, iar N si P sunt proiectiile punctului I pe BC, respectiv AD. Ar
atati c
a punctele
M, N si P sunt coliniare.
Neculai Roman, Mircesti (Iasi)
Solutie. Not
am cu E si F piciorul medianei, respectiv bisectoarei din A n
avem ca
A
ABC; cum M este mijlocul arcului BC,
EM BC. Far
a a restrange generalitatea, presupunem
IA
ca AB < AC. Din teorema bisectoarei obtinem ca
=
IF
c
b+c
BA
I
P
= ac =
. Folosind faptul ca AP I
BF
a
F E
b+c
B
C
D N
PA
DN
b+c
ADF , deducem ca
=
=
. De
AD
DF
a+b+c
a+b+c
aici, DF =
N F ; avem F E = BE BF =
M
a
a
ac
a(b c)
a
bc

=
; N E = BE BN =
(p b) =
; NF = NE
2
b+c
2(b + c)
2
2
(b c)(b + c a)
(b c)[(b + c)2 a2 ]
EF =
, prin urmare DF =
. Din asemanarea
2(b + c)
2a(b + c)
M EF ADF gasim ca
EF
a(b c)
2a(b + c)
a2
MF
=
=

=
,
FA
DF
2(b + c) (b c)[(b + c)2 a2 ]
(b + c)2 a2
asadar

a2
MF
=
. Deducem ca
MA
(b + c)2
PA ND MF
b+c b+c
a2

=1
PD NF MA
a
a
(b + c)2
67

si de aici concluzia urmeaza tinand seama de reciproca teoremei lui Menelaus.

B. Nivel liceal
L226. Fie C cercul circumscris triunghiului oarecare ABC, iar A1 centrul cer mod analog concului tangent interior cercului C si laturilor [AB] si [AC]. In
struim punctele B1 si C1 . Fie A2 centrul cercului tangent exterior cercului C si
semidreptelor [AB, [AC si n mod similar construim punctele B2 si C2 . Ar
atati c
a
A1 B1 C1 A2 B2 C2 .
Neculai Roman, Mircesti (Iasi)
Solutie. Fie I centrul cercului nscris, IM AC, M AC, A1 M1 AC,
M1 AC, A1 M1 AB, M1
AB, A2 M2 AC, M2 AC si
M2
A2 M2 AB, M2 AB. Cum
IA1
B
IM A1 M1 A2 M2 , obtinem ca
=
IA2
M1
A2
M M1
. Ins
a M M1 = AM1 AM =
A1
M M2
I
bc
4bc 4p(p a)
(p a) =
=
p
4p
A
4bc (a + b + c)(a + b + c)
C
M1 M
M2
=
4p
4bc + a2 (b + c)2
a2 (b c)2
=
=
4p
4p
(p b)(p c)
bc
bc (p a)2
, iar M M2 = AM2 AM =
(p a) =
,
4p
pa
pa
M M1
(p a)(p b)(p c)
IA1
IB1
prin urmare
=
=
. Analog se obtine ca
=
2
M M2
4p[bc (p a) ]
IA2
IB2
(p a)(p b)(p c)
(p a)(p b)(p c) IC1
,
=
. Observam ca bc (p a)2 = ac
2
4p[ac (p b) ]
IC2
4p[ab (p c)2 ]
2ab + 2ac + 2bc a2 b2 c2
IA1
IB1
IC1
(p b)2 = ab (p c)2 =
, asadar
=
=
.
4
IA2
IB2
IC2
B1 C1
C1 A1
A1 B1
=
=
si atunci A1 B1 C1 A2 B2 C2 .
De aici rezulta ca
A2 B2
B2 C2
C2 A2
L227. Fie ABC un triunghi ascutitunghic, D un punct pe latura BC si M simeBM 2 CM 2
tricul lui A fat
a de D. Dac
a
+
= AB +AC, ar
atati c
a AD este bisectoare
AB
AC
sau n
altime n ABC.
Titu Zvonaru, Com
anesti si Neculai Stanciu, Buz
au
Solutie (Gheorghe Iurea, Iasi). Fie E, F proiectiile punctelor B, C pe AM .
CM 2
BM 2
+
= AB + AC se
Daca E, F coincid cu D, deci AD BC, conditia
AB
AC
veric
a (deoarece (AB) (BM ) si (AC) (CM )).
= 90 ; putem presupune ca m(ADB)
< 90 , deci E (AD),
Fie deci m(ADB)
AC 2 CM 2
BM 2 AB 2
=
. Cum
F (DM ). Scriem conditia data sub forma
AB
AC
68

BM 2 AB 2 = M E 2 AE 2 = (M E AE) (M E + AE) = AM (M D + DE AD +
DE) = 2AM DE si, analog, AC 2 CM 2 = 2AM DF , conditia precedenta devine
DE
DF
DE
BD
BD
AB
=
. Dar
=
(din BDC CDF ) si rezulta ca
=
, adica
AB
AC
DF
DC
DC
AC
AD este bisectoarea unghiului BAC. Prin urmare, AD este bisectoare sau naltime
n ABC.
Not
a. Autorii problemei, precum si domnii Daniel V
acaru si Ioan Viorel
BD
n functie de elementele triunghiului, gasind valoCodreanu, exprima raportul
DC
c
c cos B
rile posibile (ce corespunde bisectoarei) si
(ce corespunde naltimii).
b
b cos C
L228.
In triunghiul ABC consider
am simedianele AD si BE, av
and mijloacele
ABP
.
P , respectiv Q. Demonstrati c
a BAQ
Titu Zvonaru, Com
anesti
Solutia 1 (a autorului). Fie M si N mijloacele laturilor BC, respectiv AC.
BD
c2
ac2
M

Cum AD este simediana, avem: BAD


AC;
= 2 BD = 2
;
DC
b
b + c2
A
2
2
2
2bc
2(b + c ) a
sa = AD = 2
ma , cu m2a =
.
x
b + c2
4
a

Notam = m(BAQ), = m(ABP ), x = m(M AC),


E

y = m(N
BC). Cu teorema sinusurilor n AM C
N
.P
a
ma
a sin C
obtinem ca
=
sin x =
, iar
.
2 sin x
sin C
2ma
b Q
din teorema sinusurilor aplicata n ABP rezulta
AP
sin x AD
a sin C
BP
y
=
sin =

ca
C
sin x
sin
BP
2
2ma
B
N
D
1
bc
abc sin C
abc sin C

ma =
. Analog se deduce ca sin =
.
BP b2 + c2
2(b2 + c2 )BP
2(a2 + c2 )AQ
Folosind teorema medianei n ABD si ABE, avem succesiv:

ABP
sin = sin (b2 + c2 )BP = (a2 + c2 )AQ
BAQ
(b2 + c2 )2 BP 2 = (a2 + c2 )2 AQ2 (b2 + c2 )2 (2AB 2 + 2BD2 AD2 ) =

= (a2 + c2 )2 (2AB 2 + 2AE 2 BE 2 ) (b2 + c2 )2 2c2 +

4b2 c2
m2a
(b2 + c2 )

= (a2 + c2 )2 2c2 +

2a2 c4

+ c2 )2

(b2

2b2 c4
4a2 c2
2
m2b ,
2
2
2
(a + c )
(a + c2 )2

iar aceasta relatie se dovedeste a adevarata dupa desfacerea parantezelor.

BD
c2
Solutia 2 (Gheorghe Iurea, Iasi). Deoarece
= 2 , deducem BD =
DC
b


1
c2
c2
BC

s
i,
cum
P
este
mijlocul
lui
(AD),
rezult
a
c
a
BP
=
BA
+
BC
.
b2 + c2
2
b2 + c2




1
c2
) = BA BP si
Deoarece cos(ABP
BA BP = BA BA + 2
BC
=
BA BP
2
b + c2
69

c2
a2 + c2 b2
c2 (3c2 + a2 + b2 )
c + 2

=
, iar
b + c2
4(b2 + c2 )
2




c2
c2
BP = BP BP =
BA + 2
BC
BA
+
BC
b + c2
b2 + c2

c2
a2 + c2 b2
c4
c 4
= c2 + 2 2

+ 2
a2 = 2
2c + 2b2 c2 + 2a2 c2 + a2 b2 ,
2
2
2
b +c
2
(b + c )
b + c2
3c2 + a2 + b2
) =
deducem ca cos(ABP
. Schimband pe a cu b, obtinem
4 2c4 + 2b2 c2 + 2a2 c2 + a2 b2
3c2 + b2 + a2
=
= cos(BAQ)
si,
ca cos BAQ
. Prin urmare, cos ABP
4 2c4 + 2a2 c2 + 2b2 c2 + b2 a2
, BAQ
(0, ), rezulta ca ABP
BAQ.

cum ABP
1
2

L229. Consider
am cercurile de ecuatii x2 +2Rx+y 2 = 0, respectiv x2 2Rx+y 2 =
0 si fie C1 si C2 centrele lor. Cercul cu centrul n C(0, y) cu y > 0 este tangent celor

dou
a cercuri date. Fie S = S(), 0 < <
aria suprafetei din semiplanul superior
2
m
arginit
a de cele 3 cercuri, unde este m
asura (n radiani) a unghiurilor de
 la baza

C1 C2 C. Demonstrati c
a functia S = S() este strict cresc
atoare pe 0,
si
2
calculati lim S().
2

Adrian Corduneanu, Iasi


Solutie.
Observam ca C1 (R, 0), C2 (R, 0), iar y = R tg , deci aria
triunghiului CC1 C2 este R2 tg .
C(0,y)
Daca 1 < , S(1 ) este aria triunghiului curbiliniu OM1 N1 , inclus
n triunghiul curbiliniu OM N a carui
r
arie este S(). Rezulta ca S(1 ) <
S() pentru 1 < , asadar functia
C3

M
N
S() este strict crescatoare pe (0, ).
2
Fie r = CM ; cum masura
R M1
N1
este 2, aria secunghiului C
a
1
a
torului circular CM N este r2 (
2
C1
O R C2
R
R
2). Din cos =
obtinem ca
R+r
R(1 cos )
r =
. Descompunand
cos
triunghiul CC1 C2 n patru parti (trei sectoare circulare si portiunea hasurata), avem:
1

1
1 R2 (1 cos )2
1 2
R + R2 + S() +
( 2)
2
2
2
cos2
1 R2 (1 cos )2
S() = R2 tg R2
( 2) = R2 [f () ].
2
cos2

R2 tg =

Atunci, aplicand regula lui l Hospital de doua ori, obtinem:


70

lim f () =

sin 2 (1 cos )2 ( 2)

= 2. Ca urmare lim S() = R2 (2 ).


2
2
2 cos2
2
Not
a. Solutie corecta a dat d-l Daniel V
acaru, Pitesti.
L230. Pentru x R, demonstrati inegalit
atile:
p

2
a) 1 + 2 cos2 x sin2 x + 1 + 2 sin2 x cos2 x + | sin x| + | cos x|;
3 3
p

2
b) 3 2 sin4 x sin4 x + 3 2 cos4 x cos4 x 1 .
3 3
Mih

aly Bencze, Brasov


Solutie. Vom demonstra ca t2 9 2t2 + 1 3t, t [0, 3]. Intr-adevar, daca

9
t [0, 1], atunci 9 2t2 7 > 4t2 9 2t2 9t2 (t2 9 2t2 + 1)2
4

2
4t
9 2t2 9t2 t2 9 2t2 + 1 3t, iar daca t (1, 3], atunci inegalitatea
t2 9 2t2 3t 1 revine, dupa calcule, la 2t6 9t4 + 9t2 6t + 1 0; nsa
2t6 9t4 + 9t2 6t + 1 < 2t6 9t4 + 9t2
5 = (2t2 1)(t2 1)(t2 3) (t2 + 2) < 0.
Luand n inegalitatea demonstrata t = 3u, cu u [0, 1], rezulta ca u2 3 2u2 +
1
u. Atunci a) se obtine nlocuind, pe rand, u = | sin x| si u = | cos x| si adunand
3 3
relatiile gasite, iar b) se obtine nlocuind, pe rand, u = sin2 x si u = cos2 x si adunand
relatiile gasite.
L231. Un paralelipiped dreptunghic are dimensiunile x, y, z si diagonala d. Ar
atati
c
a
d4
d4
d4
3a + 2b
+ 4
+ 4

,
4
4
4
ad + bx
ad + by
ad + bz 4
a(a + b)
lim

oricare ar a > b 0.
Marius Olteanu, Rm. V
alcea
Solutie. Cum d2 = x2 + y 2 + z 2 , avem ca
d4
1
=
ad4 + bx4
a

1+(

1
2

b
x

)2
a x2 + y 2 + z 2

b
x2
2
. Mai notam v =
a x + y2 + z2

1
1

,
a 1 + u2

b
y2
b
unde u =
2
, w =

a x + y2 + z2
a
2
P 1
z
3a + 2b
; inegalitatea din enunt revine la

. Insa, conform
x2 + y 2 + z 2
1 + u2
a+b
problemei 7, pag. 271, din V. Crtoaje - Algebraic Inequalities, GIL, Zalau, 2006,
daca x1 + x2 + . . . + xn = s, cu xi 0, i = 1, n, n N , atunci
1
1
1
ks2
.
+
+ ... +

max
2
2
1kn k 2 + s2
1 + x1
1 + x2
1 + x2n

b
Consider
am n = 3, x1 = u, x2 = v, x3 = w si s = u + v + w =
si deducem
a

P 1
b
2b
3b
3 M, unde M = max
,
,
ca
. Se observa usor ca,
1 + u2
a + b 4a + b 9a + b
71

b
si, astfel, cerinta problemei urmeaza imediat.
a+b
Nota 1. Dac
a a, b sunt numere reale nenegative, nu ambele nule, se constata ca
b
2b
3b
M =
dac
a b 2a, M =
daca 2a b 6a si M =
daca b 6a.
a+b
4a + b
9a + b
Corespunzator ecarei situatii, obtinem cate o inegalitate de tipul celei din enunt.
Nota 2. Problema ofera o mbunatatire a rezultatului din E:13949 din

G.M.
6 a b

1/2010, unde se arata ca suma din membrul stang este cel putin egala cu
2a a
(num
ar mai mic decat cele obtinute aici, n cele trei situatii posibile).
L232. Dac
a a b c > 0, ar
atati c
a are loc inegalitatea
daca a > b, atunci M =

a6 + b6 + c6 3a2 b2 c2 (a3 c3 ) (a3 b3 )(b3 c3 ).


Marian Tetiva, B
arlad
Solutia 1 (a autorului). Este adevarata inegalitatea
a6 + b6 + c6 3a2 b2 c2 (a3 b3 )(a3 c3 ).
Intr-adev
ar, aceasta inegalitate este echivalenta cu (b3 c3 )2 + a3 b3+ a3 c3 + b3 c3
3
2 2 2
3a b c 0, iar aceasta este adevarata deoarece a3 b3 +a3 c3 +b3 c3 3 a3 b3 a3 b3 b3 c3
2 2 2
= 3a b c . Analog se arata ca
a6 + b6 + c6 3a2 b2 c2 (a3 c3 )(b3 c3 )
si, prin nmultirea membru cu membru a acestor doua inegalitati si extragerea radicalului, obtinem cerinta problemei.
Solutia 2 (Daniel V
acaru, Pitesti si Ioan Viorel Codreanu, Satulung, Mara(a3 b3 ) + (b3 c3 ) 3
mures). Din inegalitatea mediilor, avem
(a b3 )(b3 c3 ).
2
a6 + b6 + c6 3a2 b2 c2
a3 c3
Daca mai arat
am ca

, solutia ar ncheiata. Inea3 c3


2
6
6
galitatea precedent
a revine, dupa calcule, la a + 2b + c6 + 2a3 c3 6a2 b2 c2 , care se
obtine aplicand inegalitatea mediilor numerelor a6 , b6 , b6 , c6 , a3 c3 si a3 c3 .
Solutia 3 (Titu Zvonaru, Comanesti). Se verica identitatea
a6 + b6 + c6 3a2 b2 c2 =

1 3
[(a c3 )2 + (a3 b3 )(b3 c3 )]+
2

1
+ [(a3 b3 )2 + (b3 c3 )2 + (c3 a3 )2 ] + (b2 + 2ac)(b2 ac)2 .
4
3
3 2
3
3
3
3
3
Aplic
and inegalitatea mediilor, obtinem ca (a c ) + (a b )(b c ) 2(a
3
3
3
3
3
c ) (a b )(b c ) si rezulta astfel o ntarire a inegalitatii din problema:

a6 + b6 + c6 3a2 b2 c2 (a3 c3 ) (a3 b3 )(b3 c3 )+


1
+ [(a3 b3 )2 + (b3 c3 )2 + (c3 a3 )2 ] + (b2 + 2ac)(b2 ac)2 .
4
72

L233. Fie n 2 un num


ar natural si a1 , a2 , . . . , an numere reale pozitive cu
proprietatea c
a a1 + a2 + . . . + an 1. Demonstrati inegalitatea
a31

a1
a2
an
n3
.
+ 3
+ ... + 3
3
2
2
2
+ a1 + 1 a2 + a2 + 1
an + an + 1
n +n+1

Titu Zvonaru, Com


anesti
x
Solutie. Fie f (x) = 3
; procedand ca n articolul Inegalit
ati omogene
x + x2 + 1
si putin
a analiz
a.... din RecMat-1/2008, vom demonstra inegalitatea
x
n2
(n3 n 2)n3
1

+
(x ).
3
2
3
3
2
x +x +1
n +n+1
(n + n + 1)
n

(1)

Inegalitatea (1) este echivalent


a succesiv cu:
n2 (n3 n 2)(nx 1)(x3 + x2 + 1) + n2 (n3 + n + 1)(x3 + x2 + 1) x(n3 + n + 1)2 0
(n5 n3 2n2 )(nx4 x3 + nx3 x2 + nx 1) + (n5 + n3 + n2 )(x3 + x2 + 1)
x(n6 + n2 + 1 + 2n4 + 2n3 + 2n) 0
(n6 n4 2n3 )x4 + (n6 n4 + 3n2 )x3 + (2n3 + 3n2 )x2
(3n4 + 4n3 + n2 + 2n + 1)x + 2n3 + 3n2 0
(nx 1)2 [(n4 n2 2n)x2 + (n4 + 2n3 n2 2n 1)x + 2n3 + 3n2 ] 0.
Ultima inegalitate este adevarat
a pentru x 0 deoarece, cand n 2, avem
ca n4 n2 2n 3n2 n2 2n = 2n2 2n 0 si n4 + 2n3 n2 2n 1 >
2n3 + 2n3 n2 2n 1 = n3 n2 + n3 1 + 2n3 2n > 0. Folosind (1) si ipoteza
a1 + a2 + . . . + an 1 obtinem ca
a31

a2
an
n3
a1
+ 3
+ ... + 3
3
+
2
2
2
+ a1 + 1 a2 + a2 + 1
an + an + 1
n +n+1
n3 (n3 n 2)
n3
+
(a1 + a2 + . . . + an 1) 3
.
3
(n + n + 1)
n +n+1

1
.
n
L234. Determinati multimile A de numere reale cu proprietatea ,,x, y R, x2 +
2
y A x3 + y 3 A.
Vlad Emanuel, Bucuresti
Solutie. Notam a = x2 + y 2 0, s = x + y, p = xy; atunci x3 + y 3 = s3 3sp =
s2 a
1
3
s3 3s
= s3 + as. Din inegalitatea (x+y)2 2(x2 +y 2 ) deducem ca s2 2a
2
2
2

si se arata ca oricare ar s [a 2, a 2], sistemul x2 + y 2 = a, x + y = s are solutie.

3
1
Functia f : [a 2, a 2] R, f (t) = t3 + at are punctele de extrem t1,2 = a
2
2

si valorile extreme a a, deci valorile expresiei x3 +y 3 acopera n ntregime intervalul


Avem egalitate daca si numai daca a1 = a2 = . . . = an =

73

[a a, a a]. In concluzie, problema se poate reformula astfel:


i multimile
Determinat

A de numere reale cu proprietatea: ,,a A [0, ) [a a, a a] A.

Dac
a exista x1 A (1, ), denim sirul xn+1 = xn xn . Prin inductie se arata
ca [xn , xn ] A si, cum lim xn = +, rezulta ca A = R.
n

Presupunem n continuare ca A (, 1] si e a = sup A. Daca a 0, atunci A


poate orice multime
ii:
inclus
a n (, a]. Daca a > 0, deosebim doua situat

I. a A. Atunci [a a, a a] A si
A va o multime de forma M [a a, a a]N ,
unde M (, a a) si N (a a, a) sunt arbitrare; se verica imediat faptul
ca mult

/ A. In acest caz,
imile
de aceasta forma au proprietatea dorita. II.
a
(a a, a a) A si A va o multime de forma M
(a a, a a) N, unde M
(, a a] este o multime arbitrara, iar N [a a, a) este o multime care contine
un sir convergent la a (din conditia de supremum). Cu aceasta, solutia este completa.
L235. Fie f : I R o functie de dou
a ori derivabil
a cu derivata a doua m
arginit
a
pe intervalul I. Demonstrati c
a exist
a un num
ar k 0 (depinz
and de functia f ) astfel
nc
at inegalitatea
f (x) + f (y) + 6f

x + y 
2

+ k(x y)2 4 f

x + 3y
4

+f

3x + y
4

sa e adevarat
a pentru orice x, y I.
Marian Tetiva, B
arlad
Solutie. Fie m si M marginile derivatei a doua a lui f pe intervalul I, deci
m f (t) M , pentru orice t I si sa consideram a, b I; conform formulei lui
Taylor, avem
f (c)
f (a) = f (b) + (a b)f (b) +
(a b)2 ,
2
pentru un anumit c I, deci obtinem inegalitatea
m
f (a) f (b) + (a b)f (b) + (a b)2 ,
2

valabil
a pentru orice a, b I. In conformitate cu aceasta avem:

3(x y) x + 3y
9m
x + 3y
+
+
f
(x y)2 ,
f (x) f
4
4
4
32

3x + y
3(y x) 3x + y
9m
f (y) f
+
f
+
(x y)2 ,
4
4
4
32

x + y 
3(x y) x + 3y
3m
x + 3y
3f
3f
+
f
+
(x y)2 ,
2
4
4
4
32

x + y 
3(y x) 3x + y
3m
3x + y
3f
+
+
3f
f
(x y)2 .
2
4
4
4
32
Adunam membru cu membru toate aceste inegalitati si obtinem:

x + y

x + 3y
4

3x + y

2
4

3x + y
x + 3y
3m
3(x y)
(x y)2 .
f
f
+

2
4
4
4

f (x) + f (y) + 6f

74

+f

Cu teorema lui Lagrange scriem


f

3x + y
4

x + 3y
4

x y
f (z),
2

pentru un anume z din intervalul I (cuprins, de fapt, ntre


urmare, inegalitatea obtinut
a devine
f (x) + f (y) + 6f

x + y 
2

4 f

x + 3y
4

x + 3y
3x + y
si
); prin
4
4

+f

3x + y
4

3(x y)2
3m
f (z) +
(x y)2 .
4
4

In sfarsit, deoarece avem 3(x y) f (z) 3M (x y)2 , ajungem la inegalitatea


4
4
2

f (x) + f (y) + 6f

x + y
2

3(M m)
(x y)2 4 f
4

x + 3y
4

+f

valabila pentru orice x, y I, adic


a numarul cerut de enunt poate k =
inf f (t)).

3x + y
4

3
(sup f (t)
4 tI

tI

Observatie. Pornind de la inegalitatea similara f (a) f (b) + (a b)f (b) +


M
(a b)2 , a, b I, putem demonstra, la fel ca
2
f (x) + f (y) + 6f

x + y
2

3(M m)
(x y)2 4 f
4

x + 3y
4

+f

3x + y
4

deci avem, pan


a la urma, inegalitatea





f (x) + f (y) + 6f x + y 4 f x + 3y + f 3x + y


2
4
4

3
(sup f (t) inf f (t))(x y)2 ,
tI
4 tI

pentru orice x, y I.
Not
a. Solutie corecta a trimis d-l Daniel V
acaru, Pitesti.

Vizitati pagina web a revistei Recreatii Matematice:

http://www.recreatiimatematice.ro
75

Probleme propuse1
Clasele primare
P.255. Dorina are un numar de mere egal cu cel mai mare numar scris cu o cifra.
Eu as avea cat Dorina, daca as mai avea 2 mere. Cate mere am eu?
(Clasa I )
Inst. Maria Racu, Iasi
P.256. Doamna nv
atatoare a scris pe tabla sase litere m, sase litere i si sase
litere n. Cate bastonase a folosit doamna nvatatoare pentru scrierea acestor litere?
(Clasa I )
Dumitrita Grigoriu, elev
a, Iasi
P.257. Completati casutele goale din sirul 1, 1, 2, 1, 2, 3, 1, 2, , , , , , , .
(Clasa I )
Mihaela Buleandr
a, elev
a, Iasi
P.258. Aati numerele a, b, c si d, stiind ca a 18 = b 15 = c 8 = b c 1.
(Clasa a II-a)
Tatiana Ignat, elev
a, Iasi
P.259. Fie sirul de numere 1, 2, 3, . . . , 12. Se formeaza grupe de cate trei numere
luate din acest sir. Cate dintre grupe pot avea suma numerelor egala cu 30?
(Clasa a II-a)
Ionut Airinei, elev, Iasi
P.260. Bomboanele dintr-o punga se mpart la cinci copii. Fiecare copil primeste
cel mult patru bomboane si cel mult trei copii au acelasi numar. Care este cel mai
mic num
ar posibil de bomboane din punga?
(Clasa a II-a)
Alexandra Tololoi, elev
a, Iasi
P.261. Scrieti num
arul 66 ca suma de numere naturale al caror produs este tot
66. Cate solutii are problema? (Nu se va tine seama de ordinea termenilor.)
(Clasa a III-a)
Monica R
aileanu, elev
a, Iasi
P.262. Ar
atati ca un patrat din carton poate mpartit, fara pierdere de material,
n 31 de patrate.
(Clasa a III-a)
Andreea Simona Simion, elev
a, Iasi

P.263. Tat
al termina o lucrare n 10 ore, iar ul n 15 ore. In cate ore termina
lucrarea cei doi, daca lucreaza mpreuna?
(Clasa a III-a)
Paula Balan, elev
a, Iasi
P.264. Daca extragem o coala dintr-un ziar, observam ca suma celor patru numere
care indica paginile este 50. Puteti spune cu ce numere este paginata coala din mijloc?
(Clasa a III-a)
Codruta Filip, elev
a, Iasi
P.265. Fie numerele nenule a, b, c, d astfel ncat a + b = 18, b + c = 14, c + d = 10.
Calculati suma 5a + 6b + 8c + 7d.
(Clasa a IV-a)
Nicolae Iv
aschescu, Craiova
P.266. Maria are astazi de sase ori varsta pe care o avea cand fratele ei, Alexandru, avea varsta ei actuala. Cand ea va avea varsta de azi a fratelui ei, cei doi vor
avea mpreun
a 27 de ani. Ce varsta are acum Maria?
(Clasa a IV-a)
Irina C
apraru, Iasi
1 Se

primesc solutii p
an
a la data de 15 iunie 2013.

76

P.267. Pentru un grup de copii s-au adus de trei ori mai multe mandarine decat
portocale. Fiecare copil trebuia sa primeasca 2 portocale si 5 mandarine, nsa din
greseala doi copii au primit cate o mandarina n plus. Stiind ca au ramas 19 portocale
si 92 mandarine, sa se ae cati copii sunt n grup.
(Clasa a IV-a)
Mariana Nastasia, elev
a, Iasi
P.268. Pe o foaie sunt scrise numerele de la 1 la 20. Sa se arate ca nu pot
formate, din cele 20 de numere, grupe de cate patru numere astfel ncat n ecare
grupa suma a doua numere sa e triplul sumei celorlalte doua.
(Clasa a IV-a)
Andreea Bzdg
a, elev
a, Iasi

Clasa a V-a
V.158. Determinati restul mp
artirii numarului 201220132014 la 36 (fara a efectua
mpartirea).
Iulian Oleniuc, elev, Iasi
V.159. Determinati numerele xyz cu proprietatea ca (x + y + z)3 = xyz.
Nicolae Iv
aschescu, Craiova
0
V.160. Determinati ultimele doua cifre ale numarului A = 3 +31 +32 +. . .+32013 .
Mirela Marin, Iasi
V.161. Se considera numerele rationale (nenegative) a, b, c, d si e cu proprietatea
ca a2 + b2 + c2 + d2 + e2 = 4. Demonstrati ca a3 + b3 + c3 + d3 + e3 8.
Mihai Cr
aciun, Pascani
V.162. Vom spune ca un num
ar este istet daca cifra unitatilor sale este egala cu
suma cuburilor celorlalte cifre. Daca ordonam crescator numerele istete, determinati
al cincisprezecelea termen al sirului obtinut.
Silviu Boga, Iasi
V.163. Putem aseza pe un cerc numerele 1, 2, 3, . . . , n (n N, n 3) astfel ncat
suma oricaror trei numere alaturate sa e numar impar?
Gheorghe Iurea, Iasi
V.164. Scriem toate numerele naturale de trei cifre pe cate un cartonas si introducem cele 900 de cartonase ntr-o cutie. Care este numarul minim de cartonase pe
care trebuie sa le extragem, far
a a ne uita la ele, pentru a siguri ca vom obtine cel
putin sapte numere cu aceeasi suma a cifrelor?
Sergiu Prisacariu, Iasi

Clasa a VI-a
VI.158. Determinati perechile de numere ntregi (x, y) care verica simultan
conditiile x + y = 6 si 2xy 3x 3y + 2 = 0.
Eugeniu Bl
ajut, Bac
au
VI.159. Determinati numerele ntregi a, b, c, d, daca 3a + 3b + 3c + 3d = 10.
Vasile Chiriac, Bac
au

VI.160. Impartind 2013 la a, obtinem catul b si restul c. Determinati numerele

77

naturale a, b si c, stiind ca reprezinta lungimile laturilor unui triunghi isoscel.


Ioana Maria Popa, elev
a, Iasi
VI.161. Determinati valorile naturale ale lui n pentru care exista numarul natural
x astfel nc
at (2x + 9, 3x + 1) = 5n .
Gheorghe Iacob, Pascani
VI.162. Un calculator defect mai face doar patru operatii: poate nmulti un
num
ar cu 2 sau cu 5 sau poate mparti un numar la 2 sau la 5, daca mpartirea se
efectueaza exact. Un copil pleaca de la numarul 20 si face aleator 2013 astfel de
operatii. Este posibil ca rezultatul nal sa e tot 20?
Petre B
atr
anetu, Galati
VI.163. Fie M si N mijloacele laturilor AB, respectiv AC ale triunghiului ABC.
Cercul de diametru AB intersecteaza dreapta M N n punctele D si E. Demonstrati

ca BD si BE sunt bisectoarele (interioara si exterioara) ale unghiului ABC.


Ioan S
ac
aleanu, H
arl
au
VI.164. Pe laturile AC si AB ale triunghiului ABC se considera punctele E,
< m(CBE)
si m(ACF
) < m(BCF
). Fie D un
respectiv F astfel nc
at m(ABE)
punct pe latura BC si G (AB), H (AC) pentru care DG BE si DH CF.
Demonstrati ca AG + AH > AB + AC BC.
Petru Asaftei, Iasi

Clasa a VII-a
VII.158. Determinati numerele naturale m cu proprietatea ca m(m+17) se poate
scrie ca produs de doua numere naturale consecutive.
Lucian Tutescu si Petrisor Rocsoreanu, Craiova
2
2
2
VII.159. Dac
reale
a m, n, p sunt

astfel ncat m +n +p = 3, demonstrati


p numere
2
2
2
ca 2m + 5 + 2n + 5 + 2p + 5 3 7.
Mihai Cr
aciun, Pascani

VII.160. Determinati triunghiurile dreptunghice cu lungimile laturilor exprimate


prin numere naturale, care au aria egala cu 24.
Neculai Stanciu, Buz
au
VII.161. Se considera trapezul ABCD cu baza mare AB egala cu diagonala AC.
Fie {E} = AD BC, {O} = AC BD, iar P (AD) este astfel ncat OP AB.
Demonstrati ca CP si CE sunt bisectoarele interioara, respectiv exterioara ale un
ghiului ACD.
Claudiu-S
tefan Popa, Iasi
VII.162. Demonstrati ca diagonala unui trapez isoscel este mai lunga decat linia
mijlocie a acestuia.
Nicolae Bourb
acut, Sarmizegetusa
VII.163. Fie D un punct pe latura BC a triunghiului ABC. Paralela prin D la
AC taie AB n E, iar paralela prin D la AB taie AC n F . Daca {M } = CE BF,
aratati ca suprafetele AEM F si BM C sunt echivalente.
Dan Popescu, Suceava
78

VII.164. Fie ABCD un patrulater convex, ale carui diagonale se intersecteaza n


O. Daca oricare ar punctele E (AB si F (CD cu AE = CF , avem ca O EF ,
demonstrati ca ABCD este paralelogram.
Gheorghe Iurea, Iasi

Clasa a VIII-a

27x3 + 54x + 15
147x2 126x + 24
=
.
98x2 84x + 10
18x2 + 36x + 16
Constantin Dragomir, Pitesti
1
2
3
VIII.159. Demonstrati ca 1 + 3x + 6x + 9x + 9x4 > , x R.
4
Ionel Tudor, C
alug
areni
VIII.160. Se considera dreapta xa d, punctul x A
/ d si planul variabil care
contine dreapta d. Notam cu M proiectia punctului A pe planul . Determinati locul
geometric al lui M .
Aida-Andreea Iacob, Iasi
VIII.161. Se dau zece cutii cubice cu muchiile de 1 cm, 2 cm, . . ., 10 cm, ecare
ind umpluta cu cuburi de muchie 1 cm. Spunem ca p cutii formeaza un bicub daca
una dintre ele contine tot atatea cuburi cate contin celelalte p 1 la un loc. Putem
forma un bicub folosind cateva dintre cutiile date?
Geanina H
av
arneanu, Iasi
VIII.162. Determinati ultimele trei cifre ale numarului natural A = 3733 +3743 +
5
375 + . . . + 6283 .
Mihai Haivas, Iasi

4m + 3
VIII.163. Determinati valorile ntregi ale lui m pentru care numarul
m5
este rational.
Bogdan Chiriac, Bac
au
VIII.164. Fie a, b, c, d patru numere reale strict pozitive astfel ncat ab(c + d)
(a + b)cd si ab + cd (a + b)(c + d). Comparati numerele a + b si c + d.
Dan Nedeianu, Drobeta Tr. Severin
VIII.158. Rezolvati n R ecuatia

Clasa a IX-a
IX.136. Fie a, b, c (0, ) cu a b c = 1. Demonstrati ca
ab
bc
ca
9
+ 2
+ 2
3
.
b2 + c2
c + a2
a + b2
a + b3 + c3 + a + b + c
Sven Cortel si Kinga Ratiu, elevi, Satu Mare
IX.137. Fie ABCD un patrulater inscriptibil. Notam cu H1 , H2 , H3 si H4
ortocentrele triunghiurilor DAB, ABC, BCD respectiv CDA si cu G1 , G2 , G3 si
G4 centrele de greutate ale triunghiurilor AH1 B, BH2 C, CH3 D respectiv DH4 A.
Demonstrati ca patrulaterele ABCD si G1 G2 G3 G4 au acelasi centru de greutate daca
si numai daca ABCD este dreptunghi.
Florin St
anescu, G
aesti
79

IX.138. Fie M un punct variabil n interiorul sau pe laturile triunghiului ascutitunghic ABC. Ar
atati ca M B 2 + M C 2 M A2 AB 2 + AC 2 .
Ovidiu Pop, Satu Mare

b = 90 si m(C)
= 30 .
IX.139. Se considera triunghiul dreptunghic ABC cu m(A)
Fie O si F picioarele medianei, respectiv naltimii din A, iar E este un punct astfel
nc
at F (AE), F E = F B. Notam cu M simetricul lui A fata de E si cu D intersectia
dreptelor M O si AC. Demonstrati ca CO = CD.
Eugeniu Bl
ajut, Bac
au
IX.140. Determinati numerele naturale nenule n pentru care
(x + y)n (xn + y n ) = nxy(x + y)(x2 + xy + y 2 )

n3
2

, x, y R.

Ionel Tudor, C
alug
areni

Clasa a X-a
X.136. Fie numerele naturale nenule a1 , a2 , . . . , an , n 2. Determinati numerele
n
P

n
P

complexe z1 , z2 , . . . , zn , de modul 1, cu proprietatea ca


|1 ziai | +
|1 + ziai | =
i=1
i=1

2n 2.
Sven Cortel, elev, Satu Mare
X.137. Fie a, b N. Rezolvati n C ecuatia |z + a| + |z b| + |a b| = 2.
Gheorghe Iurea, Iasi
X.138. Rezolvati ecuatia
log25 (24x + 22x ) + 24x + 22x = 4 + log5 (28x + 23x+3 + 242x ).
Dan Nedeianu, Drobeta Tr. Severin
X.139. Rezolvat
i ecuatiile:

n
a) [ n n + n2
] = n;

n1
b) [ n1 n +
n2 ] = n.
Ionel Tudor, C
alug
areni
X.140. Daca n triunghiul ABC, cu notatiile uzuale, are loc egalitatea a2 + b2 +
c = p2 + 9r2 , atunci triunghiul este echilateral.
C
at
alin Calistru, Iasi
2

Clasa a XI-a
XI.136. Fie A, B M2 (R) astfel ncat det A = 9, det B = 4 si det (A + B) = 1.
Determinati x R pentru care numarul det(A + xB) este minim.
R
azvan Ceuc
a, student, Iasi

XI.137. Fie a (0, 1) [2,


) si sirul (xn )n1 denit prin x1 = a, xn+1 =

2a xn+1
a + xn , n 1. Notam yn =
, n N .
a xn
80

2
a) Demonstrati ca yn >
, n N .

2 2a + 1 + 1 + 4a
1
b) Daca a (0, 1), aratati ca, n plus, yn <
, n N .
2a + a
Gheorghe Costovici, Iasi
XI.138. Fie (xn )n1 un sir de numere reale monoton si convergent. Aratati ca
sirul (yn )n1 , yn =

n
P

(xk+1 xk )2013 are aceeasi monotonie cu (xn )n1 si este, de

k=1

asemenea, convergent.

Silviu
Boga, Iasi
1
1
1
XI.139. Dac
a p N, p 2, calculati lim n e n+1 + n+2 +...+ pn p .
n
Lucian Tutescu si Liviu Smarandache, Craiova
XI.140. Daca A, B M2 (R), aratati ca 2(det A)2 +det (AB +BA)+2(det B)2
det (A2 B 2 ) + 4det AB.
Mih
aly Bencze, Brasov

Clasa a XII-a
XII.136. Calculati

R x2013 + ax1006
(x1007 + b)2013

dx, x (0, ), a, b > 0.

C
at
alin Cristea, Craiova
cos 2011x
XII.137. Determinati primitivele functiei f : 0,
R, f (x) =
.
2
cos2013 x
Liviu Smarandache si Lucian Tutescu, Craiova
XII.138. Aratati ca rad
acinile polinomului X 3 + aX 2 + bX + c sunt n progresie
geometric
a daca si numai daca a3 c = b3 .
Temistocle Brsan, Iasi
XII.139. Pe multimea nevida G se considera operatia asociativa ,, n raport cu
care are loc regula de simplicare la stanga si astfel ncat exista a G cu axa = x3 ,
x G. Demonstrati ca (G, ) este grup abelian.
D.M. B
atinetu-Giurgiu, Bucuresti si Neculai Stanciu, Buz
au
XII.140. Fie a > 0 si cercurile de ecuatii
C1 : (x a)2 + y 2 = a2 si
C2 : (x + a)2 + y 2 = a2 . Determinati aria minima a unei elipse care are drept axe de
simetrie axele de coordonate si este tangenta exterior n cate doua puncte la ecare
dintre cercurile date.
Adrian Corduneanu, Iasi

h 

Probleme pentru preg


atirea concursurilor
A. Nivel gimnazial
G236. Determinati numerele naturale a, b, c, d si e, strict mai mari ca 1, cu
proprietatea ca a + b + c + d + e = abcde 95.
Titu Zvonaru, Com
anesti
G237. Ar
atati ca exista n numere naturale distincte a1 , a2 , . . . , an pentru care
suma a1 + a1 + . . . + an este patrat perfect, iar a21 + a22 + . . . + a2n este cub perfect.
Gheorghe Iurea, Iasi
81

G238. Se considera numerele reale x, a1 , a2 , . . . , a100 . Daca 51 dintre numerele


a1 + a2 + . . . + a100
a1 + a2
,...,
sunt egale cu x, aratati ca macar doua dintre
a1 ,
2
100
numerele ai , i = 1, 100, sunt egale cu x.
C
at
alin Budeanu, Iasi
G239. Determinati valorile numarului real k, daca
a31 + . . . + a32013 + 4026 k(a1 + . . . + a2013 ), ai [2, ).
Lucian Tutescu, Craiova si Marian Voinea, Bucuresti
G240. Se considera E(x1 , x2 , . . . , xn ) =
R, n 16, cu

n
P

n
P

i=1

xi +2

xi xj , unde x1 , x2 , . . . , xn

1i<jn

x2i = 1. Determinati valorile extreme ale acestei expresii.

i=1

Petru Asaftei, Iasi

G241. In triunghiul ABC se considera cevienele concurente AA , BB si CC



astfel nc
at A B = A C, iar dreptele
BC si B
C se intersecteaza n M .


1
2
1
a) Demonstrati ca
= .
M A
AB
AC
b) Determinati lungimea segmentului M A n functie de laturile triunghiului,
atunci cand AA este bisectoare, respectiv naltime.
Neculai Roman, Mircesti (Iasi)
G242. In triunghiul ABC latura AB este xa, iar lungimea laturii AC este

cu A BC. Aratati ca dreapta


constant
a. Fie AA bisectoarea unghiului BAC,

perpendiculara n A pe AA trece printr-un punct x.


Claudiu-S
tefan Popa, Iasi
G243. Fie O intersectia diagonalelor trapezului ABCD, cu baza mare CD.
Punctele M si N sunt astfel ncat AD separa M si O, BC separa N si O, iar
M AD N CB OAB. Aratati ca BD AC > AB M N.
Cosmin Manea si Dragos Petric
a, Pitesti
G244. Fie V ABC un tetraedru, iar M, N, P mijloacele muchiilor V A, V B respectiv V C. Demonstrati ca
2(AM BC + AN CA + AP AB ) < AV BC + AV CA + AV AB + 3AABC .
Mih
aly Bencze, Brasov

G245. Un triunghi echilateral are varfurile n interiorul sau pe laturile unui


hexagon regulat de latura 1 si nu contine centrul hexagonului n interiorul sau. Care
este lungimea maxima posibila a laturii triunghiului?
Marian Tetiva, B
arlad

B. Nivel liceal
L236. Pe sfera de centru si raza 7 se considera punctele A, B, C astfel ncat
BC = 4, CA = 5 si AB = 6. Perpendiculara pe planul triunghiului ABC n centrul
82

cercului nscris n acest triunghi intersecteaza sfera n punctele M si N . Determinati


lungimea segmentului M N .
Temistocle Brsan, Iasi
L237. Fie A1 , B1 , C1 mijloacele laturilor BC, CA respectiv AB ale triunghiului
ascutitunghic ABC. Coarda comuna a cercurilor de diametre BB1 si CC1 intersecteaza B1 C1 n A2 ; construim analog punctele B2 si C2 . Demonstrati ca dreptele
A1 A2 , B1 B2 si C1 C2 sunt concurente.
Neculai Roman, Mircesti (Iasi)
L238. Fie R un punct n interiorul triunghiului ABC, iar {M } = AR BC,

{N } = BR AC, {P } = CR AB. Notam cu , si masurile unghiurilor AM


B,

BN C, respectiv CP
Daca + C +A, + + B +C si + A+B se
h A.
i
aa n intervalul 0, , aratati ca R este centrul cercului nscris n triunghiul ABC.
2
Marius Dr
agan, Bucuresti
L239. Fie ABCD patrulater circumscriptibil cu ABCD, iar A , B , C si D
sunt punctele de tangent
a ale cercului nscris cu laturile AB, BC, CD respectiv DA.
Se noteaza cu A , B , C , D simetricele punctelor A , B , C respectiv D fata de
mijloacele laturilor pe care se aa. Demonstrati ca:
a) SA B C D SA B C D ;
b) SA B C D + SA B C D SABCD .
Marius Olteanu, Rm. V
alcea
L240. Demonstrati ca n orice triunghi are loc inegalitatea

X c
2r X bc
+
3.
R
ab + ac
a+b
Marian Tetiva, B
arlad

sin3 x
cos3 x
2 2
leg
L241. Ar
atati ca
+

, x R. (In
atur
a cu
(1 + cos2 x)2
9
(1 + sin2 x)2
problema L211 din RecMat-2/2011.)
Dumitru Barac, Sibiu
L242. Un paralelipiped dreptunghic are dimensiunile x, y, z si diagonala d. Aratati
ca
d4
d4
d4
27
+ 4
+ 4

,
4
4
4
ad + bx
ad + by
ad + bz 4
9a + b
leg
oricare ar a, b > 0, 6a 5b. (In
atur
a cu L231 din RecMat-2/2012.)
Titu Zvonaru, Com
anesti
L243. Pentru m, n N si a, b, c R+ , demonstrati inegalitatea
am bm cm (an + bn + cn )2
3.
+ b3m+2n + c3m+2n

a3m+2n

leg
(In
atur
a cu problema VIII.149 din RecMat-1/2012.)
Neculai Stanciu, Buz
au
83

L244. Fie a, b, c R+ , a b c. Demonstrati ca are loc inegalitatea


(a2 + c2 )(ab + ac + bc) 2ac(a2 + b2 + c2 ) 2c(a b)(a c)(b c).
Gabriel Dospinescu, Paris si Marian Tetiva, B
arlad


k
k

L245. Fie f, g : [0, 1] [0, 1] doua functii continue astfel ncat f
g

n
n

k
1
f g
, k, n N , k n. Demonstrati ca cele doua functii sunt egale.
k
n
Florin St
anescu, G
aesti

Training problems for mathematical contests


A. Junior highschool level
G236. Determine the natural numbers a, b, c, d and e, strictly larger than 1, with
the property that a + b + c + d + e = abcde 95.
Titu Zvonaru, Com
anesti
G237. Prove that n distinct natural numbers a1 , a2 , . . . , an such that the sum
a1 + a1 + . . . + an is a perfect square and a21 + a22 + . . . + a2n is a perfect cube.
Gheorghe Iurea, Iasi
G238. The real numbers x, a1 , a2 , . . . , a100 are given. If 51 numbers among
a1 + a2
a1 + a2 + . . . + a100
a1 ,
,...,
are equal to x, prove that at least two among
2
100
the numbers ai , i = 1, 100, are equal to x.
C
at
alin Budeanu, Iasi
G239. Determine the values of the real number k, knowing that
a31 + . . . + a32013 + 4026 k(a1 + . . . + a2013 ), ai [2, ).
Lucian Tutescu, Craiova si Marian Voinea, Bucuresti
G240. Consider the expression E(x1 , x2 , . . . , xn ) =
x1 , x2 , . . . , xn R, n 16, with

n
P

n
P

i=1

xi + 2

xi xj , where

1i<jn

x2i = 1. Determine the extreme values of this

i=1

expression.
Petru Asaftei, Iasi
G241. In the triangle ABC, the concurrent cevian lines AA , BB and CC are

given, such that A B = A C, and the lines
BC aand B C meet at point M .
1

2
1
a) Prove that
= .
M A
AB
AC
b) Determine the length of the segment M A as a function of triangles side lengths,
in the cases when AA is an angle-bisector line, respectively the altitude from A.
Neculai Roman, Mircesti (Iasi)
84

G242. In the triangle ABC, the side AB is xed, and the length of the side AC
with A BC. Show that the
is constant. Let AA be the bisector of angle BAC,

perpendicular line at A on AA passes through a xed point.


Claudiu-S
tefan Popa, Iasi
G243. Let O be the intersection of the diagonals of trapezium ABCD, with
the big base CD. The points M and N are placed such that AD separates M
from O, BC separates N from O, and M AD N CB OAB. Show that
BD AC > AB M N.
Cosmin Manea si Dragos Petric
a, Pitesti
G244. Let V ABC be a tetraahedron, and let M, N, P be the mid-points of the
edges V A, V B and respectively V C. Prove that
2(AM BC + AN CA + AP AB ) < AV BC + AV CA + AV AB + 3AABC .
Mih
aly Bencze, Brasov
G245. An equilateral triangle has its vertices inside or on the sides of a regular
hexagon of side length = 1. The triangle does not contain the centre of the hexagon
in its interior. Which is the maximum possible length of triangles side length?
Marian Tetiva, B
arlad

B. Highschool Level
L236. The points A, B, C are considered on the sphere of centre and radius
7 such that BC = 4, CA = 5 and AB = 6. The perpendicular line on the plane of
triangle ABC in the centre of the circle inscribed in this triangle intersects the sphere
at the points M and N . Determine the length of the line segment M N .
Temistocle Brsan, Iasi
L237. Let A1 , B1 , C1 be the mid-points of the sides BC, CA and respectively AB
of the acute-angled triangle ABC. The common chord of the circles of diameters BB1
and CC1 intersect the line B1 C1 at A2 ; we build in a similar way the points B2 and
C2 . Prove that the lines A1 A2 , B1 B2 and C1 C2 are concurrent.
Neculai Roman, Mircesti (Iasi)
L238. Let R be a point in the interior of triangle ABC, and {M } = AR BC,
{N } = BR AC , {P } = CR AB. We note with , and the measures of the

angles AM
B, BN
C, respectively CP
A. If h i+ C + A, + + B + C

and + A + B fall in the interval 0, , show that R is the centre of the


2
circle inscribed in the triangle ABC.
Marius Dr
agan, Bucuresti
L239. Let ABCD be a circumscrible quadrilateral with ABCD, while A , B , C
and D are the tangency (or contact) points of the inscribed circle with the sides
AB, BC, CD, respectively DA. Let A , B , C , D be the symmetric points of A , B , C ,
repectively D with respect to the mid-points of the sides they lie on. Show that:
a) SA B C D SA B C D ;
b) SA B C D + SA B C D SABCD .
Marius Olteanu, Rm. V
alcea
85

L240. Prove that the following inequality holds in any triangle:

X c
2r X bc
+
3.
R
ab + ac
a+b
Marian Tetiva, B
arlad

sin x
cos x
2 2
, x R. (Related to the
L241. Show that
+

2
2 x)2
2
(1
+
cos
9
(1 + sin x)
problem L211 of RecMat-2/2011.)
Dumitru Barac, Sibiu
L242. A right-angled parallelepiped has the dimensions (edge lengths) x, y, z and
the diagonal length d. Show that
3

ad4

d4
d4
27
d4
+ 4
+ 4

,
4
4
+ bx
ad + by
ad + bz 4
9a + b

any would be a, b > 0, 6a 5b. (Related to the problem L231 of RecMat-2/2012.)


Titu Zvonaru, Com
anesti
L243. For m, n N and a, b, c R+ , prove the inequality
am bm cm (an + bn + cn )2
3.
+ b3m+2n + c3m+2n

a3m+2n

(Related to the problem VIII.149 of RecMat-1/2012.)


Neculai Stanciu, Buz
au
L244. Let a, b, c be positive real numbers with a b c. Prove that the following
inequality holds :
(a2 + c2 )(ab + ac + bc) 2a(a2 + b2 + c2 ) 2c(a b)(a c)(b c).
Gabriel Dospinescu, Paris si Marian Tetiva, B
arlad
Let
L245.
f,
g : [0, 1][0, 1]
be two continuous functions with the property that
k
k
1
k

f
n g n k f g n , k, n N , k n. Prove that the two functions
are equal.
Florin St
anescu, G
aesti

Primul numar al Colectiei ,,Recreatii Matematice


1. D. Br
anzei, Al. Negrescu

Probleme de pivotare,

Ed. ,,Recreatii Matematice, Iasi, 2011 (208 pag.)


poate procurat printr-o simpla cerere la adresa: t birsan@yahoo.com si indicarea
adresei postale proprii. Cartea va trimisa cu plata ramburs la adresa indicata contra
sumei de 25 lei (inclusiv taxe postale).
86

Pagina rezolvitorilor
CRAIOVA
S
coala nr. 24 ,,Sf. Gheorghe. Clasa a VII-a (prof. GURGUI Dan). NICOLAESCU Andrei: VII(152,154,155), VIII(153,154), IX(132,135), XII.132. Clasa a
VIII-a (prof. TUT
ESCU Lucian). TURCU Andrei George: VI.151, VII(152,155),
VIII(151,153,154), IX(131,132), XII.132.
Colegiul National ,,Carol I. Clasa a IX-a (prof. CIULCU Claudiu). ENE
Cristina Elena: VII(154,155), VIII(153-155), IX(131,135), X.132, XI.131, XII.132.
Colegiul National ,,Fratii Buzesti. Clasa a IX-a (prof. POPA Marius). VIRLAN
Leonard: VIII(151,156), G(229,230,231). Clasa a XI-a (prof. TUT
ESCU Lucian).
ENUCA Andreea Ramona: IX.131, X.132, XI(131,132), XII.132, G(229,230); GOLEA
Monica: IX.131, X(132,135), XI(131,132), XII.132, G(229,230).
GRAJDURI (IAS
I)
AnaS
coala Gimnazial
a ,,Valea Satului. Clasa a IV-a (prof.nv.primar URMA

Lacramioara). AMARIUT
EI Alin-Petronel-Florentin: P(244-246,249,253); RUSU
Alexandra: P(244-246,249,253); STOICA Mariana-Ionela: P(244-246,249,253).
IAS
I
S
coala nr. 3 ,,Al. Vlahut
a. Clasa a VI-a (prof. MARIN Mirela). ENEA
Codrut: P(251,252), V(151,152), VI.151; HUHU Paula: P(251,252), V(151,152),
VI.151; POPOVICI Teodor-Andrei: P(251,252), V(151,152), VI.151. Clasa a VII-a
(prof. MARIN Mirela). CIOCOIU Alexandra: V(151,152), VI(151,156,157); CONDURACHE Alexandra: V(151,152), VI(151,156,157); MARIN Marius: V(151,152),

VI(151,156,157); MANZAT
Rebecca: V(151,152), VI(151,156,157); TROFIN ANA:
V(151,152), VI(151,156,157); VLAD Ioana: V(151,152), VI(151,156,157).
S
coala Gimnazial
a nr. 26 ,,G. Cosbuc. Clasa a III-a (inst. RUSU Eugenia). AVRAM Elena: P(245-248,250); BARGHI Sara: P(245-248,250); CAZACU
Costin: P(245-248,250); CIRDEI Dan: P(245-248,250); CRISTEA Alexandru: P(245
248,250); FRONEA Bogdan: P(245-248,250); IACOB Anca: P(245-248,250); LAZAR

Alexandra: P(245-248,250); PARA Gabriel: P(245-248,250). Clasa a IV-a (prof.nv.


Sebastian-Andrei: P(240-242,244-246); GROSU
primar RACU MARIA). CIOPEICA
Victor-Alessandru: P(240,242,244-246,250); LUCHIAN Maria-Clara: P(240-242,244246,250); MANOLE Alexandra-Georgiana: P(240-242,244-246,250); PASNICU Cosmin-Constantin: P(240-242,244-246); POPESCU Andrei-Eduard: P(240-242,244-246);

RAILEANU
Ana-Maria: P(240-242, 244-246); RAILEANU
Razvan-Constantin: P(240242,244-246,250); VASILE Raluca-Andreea: P(240-242,244-245,250).
S
coala nr. 33 ,,M. Kog
alniceanu. Clasa a III-a (nv. SIRBU Lenuta). CIOCOIU Alexandru Boris: P(244-246,248,250,254).
Colegiul National ,,Emil Racovita
. Clasa a V-a (prof. BUDEANU Catalin).
OLENIUC Iulian: P(250,251,253), V(151,153,156).
Colegiul National ,,C. Negruzzi. Clasa a V-a (prof. CIANGA Mihaela). ANDRONACHE Andreia Irina: P(249,250,252-254). Clasa a VII-a (prof. PRECUPANU Codrin). PANAITE Beatrice: V(151,152,157), VI(155,156).
Loredana). LUCHIAN
Liceul Teoretic ,,Miron Costin. Clasa a IV-a (nv. T
ARA
Denisa Alexandra: P(244,246-254).
87

Cristian). POPA Ioana


Colegiul National Iasi. Clasa a VI-a (prof. LAZAR
Maria: P(252-254), V(151-157), VI(151-157), VII(152-155), G(226,227,231).

ROS
IORI (BACAU)
S
coala Gimnazial
a nr. 1. Clasa a VIII-a (prof. CICEU Nela). ONOFREI
Adina: VI(151-157), VII.152.

TARGOVIS
TE
S
coala ,,Vasile C
arlova. Clasa I. (nv. ION Daniela). BACIU Maria Briana:
P(240-244).

T
IGANAS
I (IAS
I)
S
coala Gimnazial
a cu clasele I-VIII ,,M. Kog
alniceanu. Clasa a III-a
(nv. SAMSON Daniel-Mihai). DUCA Daria-Catalina-Stela: P(246,248-253). Clasa
A Aurica). DUCA Ema-Stefania: P(246,248-254);
a IV- a (prof.nv.primar BADIT
DUCA Anamaria: P(246,248-254); DUCA Roxana-Georgiana: P(246,248-254);
Alexandra-Ionela: P(246,248-254); SANDU Marta: P(246,248-254);
GHIOANCA
STEFANACHE Anamaria: P(246,248-254); TICAN David-Petru: P(246,248-254).
Clasa a VI-a (prof. IACOB Aida Andreea). CAZADOI Ioana Cristina: P(250,253),
V(151,152), VI(155,156), VII.152; DUCA Mihaela Cristina: P(250,253,254), V(154,
156); SANDU Ana Paula: P(250,253), V(151,152), VII.156. Clasa a VII-a (prof.

IACOB Aida Andeea). DUCA Malina: V(151,152,154), G(226,229); GHIOANCA


Bianca-Maria: V(151,152,154), VI.152, G.232; TOMA Evelina: V(151,152,154),
VI.152, G.229.

Elevi rezolvitori premiati


S
coala Gimnazial
a ,,G. Cosbuc, Iasi

CIOPEICA Sebastian-Andrei (cl. a IV-a): 1/2012(6pb), 2/2012(5pb), 1/2013(6pb).


GROSU Victor-Alessandru (cl. a IV-a): 1/2012(7pb), 2/2012(5pb), 1/2013(7pb).
LUCHIAN Maria-Clara (cl. a IV-a): 1/2012(6pb), 2/2012(8pb), 1/2013(7pb).
MANOLE Alexandra-Georgiana (cl. a IV-a): 1/2012(6pb), 2/2012(6pb), 1/2013(7pb).
PASNICU Cosmin-Constantin (cl. a IV-a): 1/2012(5pb), 2/2012(5pb), 1/2013(6pb).

RAILEANU
Ana-Maria (cl. a IV-a): 1/2012(5pb), 2/2012(5pb), 1/2013(6pb).

RAILEANU
R
azvan-Constantin (cl. a IV-a): 1/2012(6pb), 2/2012(5pb), 1/2013(7pb).
S
coala Gimnazial
a cu clasele I-VIII ,,M. Kog
alniceanu, T
ig
anasi (Iasi)
CAZADOI Ioana Cristina (cl. a VI-a): 1/2011(5pb), 2/2011(7pb), 1/2013(7pb).
DUCA Mihaela Cristina (cl. a VI-a): 1/2011(5pb), 2/2011(7pb), 1/2013(5pb).
SANDU Marta (cl. a IV-a): 1/2012(6pb); 2/2012(11pb); 1/2013(8pb).
Colegiul National Iasi
POPA Ioana-Maria (cl. a VI-a): 2/2011(14pb), 2/2012(22pb), 1/2013(24pb).
88

Revista semestrial RECREAII MATEMATICE este editat de


ASOCIAIA RECREAII MATEMATICE. Apare la datele de 1 martie i
1 septembrie i se adreseaz elevilor, profesorilor, studenilor i tuturor celor
pasionai de matematica elementar.
n atenia tuturor colaboratorilor
Materialele trimise redaciei spre publicare (note i articole, chestiuni de
metodic, probleme propuse etc.) trebuie prezentate ngrijit, clar i concis; ele
trebuie s prezinte interes pentru un cerc ct mai larg de cititori. Se recomand ca
textele s nu depeasc patru pagini. Evident, ele trebuie s fie originale i s
nu fi aprut sau s fi fost trimise spre publicare altor reviste. Rugm ca materialele tehnoredactate s fie nsoite de fiierele lor (trimise la adresa de mai jos).
Problemele destinate rubricilor: Probleme propuse i Probleme pentru
pregtirea concursurilor vor fi redactate pe foi separate cu enun i demonstraie/rezolvare (cte una pe fiecare foaie) i vor fi nsoite de numele autorului, coala i localitatea unde lucreaz/nva.
Redacia va decide asupra oportunitii publicrii materialelor primite.
n atenia elevilor
Numele elevilor ce vor trimite redaciei soluii corecte la problemele din
rubricile de Probleme propuse i Probleme pentru pregatirea concursurilor
vor fi menionate n Pagina rezolvitorilor. Elevii vor ine seama de regulile:
1. Pot trimite soluii la minimum cinci probleme propuse n ultimul numr aprut al revistei. Pe o foaie va fi redactat soluia unei singure probleme i
vor fi menionate datele: numele i prenumele, clasa, profesorul, coala i
localitatea.
2. Elevii din clasele VI-XII au dreptul s trimit soluii la problemele
propuse pentru clasa lor, pentru orice clas mai mare, din dou clase mai mici i
imediat anterioare. Elevii din clasa a V-a pot trimite soluii la problemele propuse
pentru clasele a IV-a, a V-a i orice clas mai mare, iar elevii claselor I-IV pot
trimite soluii la problemele propuse pentru oricare din clasele primare i orice clas mai mare. Orice elev poate trimite soluii la problemele de concurs (tip G i L).
3. Oricine poate trimite soluii la problemele de tip L; la publicare, soluiile
acestora vor fi urmate de numele tuturor rezolvitorilor.
4. Plicul cu probleme rezolvate se va trimite prin pot (sau va fi adus
direct) la adresa Redaciei:
Prof. dr. Temistocle Brsan
Str. Aurora, nr. 3, sc. D, ap. 6,
700 474, Iai
Jud. IAI
E-mail: t_birsan@yahoo.com

CUPRINS
Academicianul RADU MIRON la 85 de ani (prof.dr. Mihai Anastasiei) .............................1
Profesorul TRAIAN COHAL la aniversarea a 80 de ani (conf.dr. Mircea Brsan) ..........4
Acad. Viorel BARBU Matematica ntre mituri culturale i certitudine ...............................6

ARTICOLE I NOTE
T. BRSAN Cercuri, valori medii i diviziuni armonice ...................................................... 9
C.-L. BEJAN O proprietate a patrulaterelor convexe ..........................................................13
N. ROMAN Observaii asupra cercurilor seminscrise .........................................................15

CORESPONDENE
H. STEPHAN Multiplicative duality in triangles ...............................................................18
A. REISNER Autour de lquation X2 = A dans Mn(R) ....................................................22

CHESTIUNI METODICE
C. DRAGOMIR Reguli de tip Sarrus pentru calculul determinanilor de ordin 3 ..............25
M. BENCZE Utilizarea injectivitii funciilor n rezolvarea ecuaiilor ..............................27
Problema L222 din nr. 1/2012 revizitat ................................................................................29

CUM CONCEPEM... CUM REZOLVM


M. TETIVA O identitate misterioas ..................................................................................32

CHESTIUNI COMPLEMENTARE MANUALELOR


M. TRNUCEANU Ireductibilitate n inele de polinoame..............................................36

COLI I DASCLI
C. DIMITRIU, M. CRCIUN Colegiul Naional M. Sadoveanu din Pacani ..............42

CONCURSURI I EXAMENE
Concursul Recreaii Matematice, ed. a X-a, Muncel, 2012.................................................45
Concursul interjudeean Sperane Olimpice, ed. a XII-a, Pacani, 2012 ..............................47

PROBLEME I SOLUII
Soluiile problemelor propuse n nr. 2/2012 ..........................................................................49
Soluiile problemelor pentru pregtirea concursurilor propuse n nr. 2/2012 ......................64
Probleme propuse..................................................................................................................... 76
Probleme pentru pregtirea concursurilor .............................................................................. 81
Training problems for mathematical contests ....................................................................... 84
Pagina rezolvitorilor .............................................................................................................. 87
Elevi rezolvitori premiai........................................................................................................ 88
ISSN 1582 1765

8 lei

S-ar putea să vă placă și